NCLEX - review

¡Supera tus tareas y exámenes ahora con Quizwiz!

A nurse is preparing to administer haloperidol 3 mg IM to a client. Available is haloperidol solution 5 mg/mL. How many mL should the nurse plan to administer? ___________________ mL

0.6 mL

A nurse is preparing to administer phytonadione 7 mg subcutaneously to the client who has an INR of 4. Available is phytonadione 10 mg/mL. How many mL should the nurse administer? ________________________ mL

0.7 mL

A nurse is preparing to administer 0.9% sodium chloride at 100 mL/hr. the drop factor of the manual IV tubing is 15 gtt/mL. The nurse should set the manual IV infusion to deliver how many gtt/min? ______________________________ gtt/min

25 gtt/min

A nurse is caring for a school-age child who has been admitted to the facility in sickle cell crisis. The nurse is measuring the child's oral intake for the shift. The child consumed 4 oz of juice at breakfast. For lunch, the child consumed 6 oz of milk, 6 oz of gelatin, and drank 7 oz of water. What is the child's oral intake for this shift in mL _____________________ mL

690 mL

A nurse is collecting data from a infant who has severe dehydration. Which of the following findings should the nurse expect? A - Capillary refill of 2 seconds B - Decreased heart rate C- Weight loss of 10% D - Bulging anterior fontanel

C- Weight loss of 10%

A nurse is preparing to administer clindamycin 450 m PO to a client who has endometritis. The amount available is clindamycin 150 mg/capsule. How many capsules should the nurse administer? _________________ capsules

3 capsules

A nurse is to preparing to administer cefazolin 1 in dextrose 5% in water 100 mL to infuse over 30 min. The drop factor of the manual IV tubing is 10 gtt/mL. The nurse should set the manual IV infusion to deliver how many gtt/min? ________________________________ gtt.min

33 gtt/min

A nurse is caring for a client whose NG tube has 475 mL drainage in the suction canister. Previously, the nurse installed 30 mL normal saline through the NG tube twice during the shift without withdrawing it. Calculate the amount of NG output the nurse should document on the client's medical record. ________________________________ mL

415 mL

A nurse is reinforcing discharge teaching with the parent of school-age child who is being treated for nephrotic syndrome. The parent asks the nurse why it is necessary to check the child's urine for protein. Which of the following explanations should the nurse offer? A - "A decrease in urine protein indicates that treatment is effective." B - "Protein in the urine indicates your child's protein intake is adequate." C - "Protein in the urine indicates a need to begin dialysis." D - "An increase in urine protein indicates your child has a secondary infection."

A - "A decrease in urine protein indicates that treatment is effective."

A nurse is collecting data from a newly admitted client. Which of the following questions should the nurse include to gather psychosocial information? A - "Do you have a hobby that you enjoy?" B - "What is the date and where are you living?" C - "How are an apple and an orange alike?" D - "Can you take this pencil and put it in the cup?"

A - "Do you have a hobby that you enjoy?"

A nurse is reinforcing teaching with a client who has a history of pulmonary embolism about reducing the risk of developing deep vein thrombosis (DVT) when traveling. Which of the following statements should the nurse include in the teaching? A - "Get up and walk every 2 hours when traveling by car." B - "Reduce fluid intake while traveling by plane." C - "Plan to wear knee-high hose when traveling." D - "You can cross your legs after walking for 10 minutes when traveling by train."

A - "Get up and walk every 2 hours when traveling by car." Rational A - The client should stop and walk every 1 to 2 hr when traveling by car to promote circulation and decrease venous stasis, which can cause a DVT. A client who has a DVT is at risk for pulmonary emboli because the thrombus can dislodge and migrate to the lung. B - The client should increase fluid intake when traveling by plane to reduce the risk of dehydration, which can lead to clot formation. C - A client who has a history of pulmonary embolism from a DVT should avoid wearing knee-high hose that can cause constriction around the knee, restrict blood flow, cause venous stasis, and lead to a DVT, which places the client at risk for a pulmonary embolism. The nurse should instruct the client to wear support hose or compression stockings when standing for extended periods to promote venous return. D - A client who has a history of pulmonary embolism from a DVT should avoid leg crossing. This action can restrict blood flow, cause venous stasis, and lead to a DVT, which places the client at risk for a pulmonary embolism.

A nurse is speaking with a guardian of a child who has just died following a terminal illness. Which of the following statements should the nurse make? A - "I am sorry for your loss." B - "I can imagine how you must feel." C - "You will feel better in no time." D - "Your child isn't suffering anymore."

A - "I am sorry for your loss." Rational A - This statement is an example of expressing empathy for the loss and is a therapeutic statement. The most supportive measure the nurse can perform is to listen. B - This statement is an example of artificial consolation and is not a therapeutic statement. C - This statement is an example of false reassurance and is not a therapeutic statement. D - This statement is an example of rationalization and is not a therapeutic statement. During the bereavement process, the nurse should be supportive and caring.

A nurse is reinforcing teaching to a client who has a new prescription for losartan. Which of the following statements by the client indicates understanding of the teaching? A - "I should avoid sudden changes in position until I know how my body will react to the medication." B - "I might develop a chronic cough while taking this medication." C - "When my blood pressure returns to normal, I can stop taking my medicine." D - "I may experience constipation while taking this medication."

A - "I should avoid sudden changes in position until I know how my body will react to the medication." Rational A - Orthostatic hypotension is an adverse effect of losartan. The client should be reminded to change positions slowly to decrease the risk of orthostatic hypotension. B - Chronic cough, or intractable cough, is an adverse effect of angiotensin-converting enzyme (ACE) inhibitors such as enalapril. C - The client should be instructed to take the medication at the same time every day and not to stop taking the medication unless directed by a provider. D - Diarrhea, not constipation, is an adverse effect of losartan.

A nurse is reinforcing teaching with a client who is taking insulin glargine. Which of the following information should the nurse include in the teaching? A - "This type of insulin should be given at the same time every day." B - "This insulin can be mixed with short-acting insulin in a single syringe." C - "This type of insulin can be used in a pump." D - "This insulin has an increased risk for hypoglycemia."

A - "This type of insulin should be given at the same time every day."

A nurse is reinforcing teaching with a client about breast self-examination (BSE). Which of the following statements by the client indicates an understanding of the teaching? A - "I should begin my BSE by looking at my breasts while standing in front of the mirror." B - "I should perform my BSE each month on the first day of my menstrual cycle." C - "I should expect a small amount of white discharge when I gently squeeze my nipples." D - "I should feel each of my breasts at the same time to check for any differences."

A - "I should begin my BSE by looking at my breasts while standing in front of the mirror." Rational A - The client should begin their BSE by standing in front of the mirror and inspecting the appearance of each breast. The client should observe for symmetry and changes in appearance. The nurse should instruct the client to report to the provider any indications of dimpling, puckered skin, rashes or scaling of the skin, or nipple discharge. These findings, or any other changes, warrant further assessment by the provider. B - The client should perform a BSE each month 4 to 7 days after menstruation ends. During this time of the menstrual cycle, the client's breasts are less tender than at the beginning of the cycle. Performing the BSE when the breasts are less tender allows the client to perform a more thorough self-examination and increases the likelihood that they will detect changes or abnormalities. C - The client should gently squeeze each nipple during the BSE to check for any discharge. The nurse should instruct the client to report any discharge from their nipples to the provider. After childbirth, the client might have clear yellow discharge from the nipples. Otherwise, this finding warrants further assessment by the provider. D - The client should palpate one breast thoroughly in a vertical strip, circular, or wedge pattern, checking for any lumps or masses in the breast. After completing the palpation of one breast, the client should repeat the process on the other breast. The nurse should instruct the client to report any lumps, masses, or changes to the provider for further assessment.

A nurse is reinforcing teaching about aseptic precautions with a newly licensed nurse. Which of the following statements by the newly licensed nurse indicated an understanding of the teaching? A - "I should maintain a closed urinary drainage system placed below the level of the client's bladder." B - "I should change a client's IV tubing every 48 hours." C - "I should consider the sterile area of a gown to be from the shoulders to the knees." D - "I should remove my eyewear before my gloves when removing personal protective equipment."

A - "I should maintain a closed urinary drainage system placed below the level of the client's bladder." Rational A - The nurse should maintain a closed urinary drainage system with a downhill flow of urine to prevent urinary tract infections. The collection bag is kept below the level of the client's bladder to prevent backflow of contaminated urine into the bladder from the drainage bag. Urine should flow freely into the drainage bag. If the tubing is kinked, obstructed, or twisted, urine that becomes static in the tubing can cause infection. B - The nurse should change intravenous tubing no more often than every 96 hr to minimize the risk of contamination from micro-organisms. C - The nurse should understand that a gown is sterile from the shoulders to the waist area. Only the front portion of the gown is sterile. Anything below the waist is considered contaminated. D - The nurse should remove gloves first because they are the most contaminated piece of personal protective equipment. This measure will prevent the soiled areas of the gloves from touching the skin. The nurse should perform hand hygiene prior to removing protective eyewear.

A nurse is reinforcing teaching with a client who is at 36 weeks of gestation and is scheduled for a nonstress test. Which of the following statements by the client indicates an understanding of the teaching? A - "I will sit in a reclining chair during the test." B - "I will be rubbing my nipples during the test." C - "I will have an IV started prior to the test." D - "I will not eat or drink for 2 hours prior to the test."

A - "I will sit in a reclining chair during the test." Rational A - The nurse should instruct the client that she will be seated in a reclining chair and have a wedge under the right hip during the test. This positioning promotes uterine perfusion and prevents supine hypotension from the uterus compressing the maternal vena cava. B - The nurse should instruct the client that an external fetal monitor will be applied to her abdomen during the test to monitor for fetal activity and the fetal heart rate response. A client undergoing a contraction stress test might need to perform nipple stimulation. C - The nurse should instruct the client that an external fetal monitor will be applied to her abdomen during the test to monitor for fetal activity and the fetal heart rate response. A client undergoing a contraction stress test might need an IV for the administration of oxytocin. D - The nurse should instruct the client that she can eat and drink as usual prior to the test. The test is noninvasive and does not require the client to fast.

A nurse is caring for a client who has cancer and is advised to begin extensive radiation therapy immediately. Which of the following statements by the client indicates the use of the defense mechanism of rationalization? A - "I won't be able to begin radiation treatment right now because my house is being completely remodeled." B - "I'm not planning to begin radiation treatment because I don't believe my condition is serious." C - "I know my sister is worried about me starting radiation treatment because she thinks it will be painful for me." D - "I have been reading about the type of cancer I was diagnosed with and am studying about how it is treated."

A - "I won't be able to begin radiation treatment right now because my house is being completely remodeled." Rational A - The nurse should identify this statement as an example of the use of the defense mechanism of rationalization. Rationalization occurs when a client uses an acceptable explanation to justify an illogical or unreasonable action. B - The nurse should identify this statement as an example of the use of the defense mechanism of denial. Denial occurs when a client refuses to acknowledge an unpleasant or anxiety-producing situation or feeling. C - The nurse should identify this statement as an example of the use of the defense mechanism of projection. Projection occurs when a client attributes their own feelings as coming from another person. D - The nurse should identify this statement as an example of the use of the defense mechanism of intellectualization. Intellectualization occurs when a client focuses on facts in an analytical manner without including an emotional response.

A nurse is assisting with the care of a client who has a terminal cancer and is receiving chemotherapy. The client tells the nurse that she is only continuing treatment for her family's sake. Which of the following responses should the nurse make? A - "Let's talk about your reasons for continuing treatment." B - "You should talk to a social worker about your situation." C - "I'll get the chaplain to come speak with you about your thoughts and feelings." D - "I know you are tired of this treatment, but you are right to think of your family first.

A - "Let's talk about your reasons for continuing treatment."

A nurse is caring for a school-age girl who is being treated for frequent, severe urinary tract infections (UTIs). The nurse should recognize that which of the following statements bub the parent indicates a possible cause of the UTIs? A - "My daughter has bowel movements every 4 to 5 days." B - "I taught her to wipe from front to back after going to the bathroom." C - "She urinates every 2 to 3 hours during the day." D - "I don't let her wear nylon underwear."

A - "My daughter has bowel movements every 4 to 5 days."

A nurse is reinforcing teaching about a provider's treatment plan with a client. The client refuses the plan. Which of the following statements should the nurse make? A - "Tell me about your personal treatment wishes." B "Tell me why you're refusing treatment that can improve your health." C - "Your provider's treatment decision is what is best for you." D - "If I were you, I would accept this treatment plan."

A - "Tell me about your personal treatment wishes." Rational A - The nurse should ensure that the client has the right to state their personal wishes regarding treatment decisions, including the refusal of treatment. B - The nurse should not ask a client to explain why they think or act a certain way because it causes resentment, insecurity, and mistrust. C - The nurse should not impose personal values, beliefs, and attitudes on the client because they have the right to make their own informed decisions. D - The nurse is avoiding the client's concern and offering personal advice

A nurse is reinforcing teaching with a client who has a new diagnosis of celiac disease. Which of the following statements should the nurse make first? A - "Tell me some foods that you like to eat." B - "Many uncooked foods are gluten-free." C - "You can only eat certain types of grain." D - "You might need to eliminate dairy temporarily."

A - "Tell me some foods that you like to eat." Rational A - The first action the nurse should take using the nursing process is to collect data from the client. Therefore, the nurse should ask the client about food preferences to help plan appropriate food choices. It also involves the client in decision-making, which promotes effective teaching and adherence to new dietary requirements. B - The nurse should teach the client about food choices that are naturally gluten-free, such as milk, cheese, fruits, and vegetables. However, there is another statement the nurse should make first. C - The nurse should tell the client that gluten is mainly found in grains and provide the client with information on selection of appropriate grain sources. Providing the client with a handout can help promote understanding. However, there is another statement the nurse should make first. D - The nurse should inform the client that acute celiac disease can cause lactose intolerance. Lactose restriction might be required until the intestinal tract heals. However, there is another statement the nurse should make first.

A nurse is reinforcing teaching with a client who has schizophrenia and a new prescription for chlorpromazine. Which of the following statements should the nurse include in the teaching? A - "The voices you have been hearing should decrease." B - "You will likely have more energy while on this medication." C - "You should now be able to spend more time in the sun." D - "Call your provider immediately if you develop a dry mouth."

A - "The voices you have been hearing should decrease."

A nurse is reinforcing teaching with a newly licensed nurse about the action of ACE inhibitors in clients who are experiencing heart failure. Which of the following statements should the nurse include? A - "This medication decreases aldosterone secretion to reduce sodium and water resorption." B - "This medication relaxes the coronary vessels to improve circulation to the heart muscle." C - "This medication relaxes smooth muscle by blocking the binding of calcium to its receptors." D - "This medication blocks sympathetic nervous system stimulation to the heart's conduction system."

A - "This medication decreases aldosterone secretion to reduce sodium and water resorption." Rational A - The nurse should reinforce that ACE inhibitors treat heart failure by inhibiting aldosterone secretion, which reduces sodium and water resorption. This causes diuresis, which decreases blood volume and cardiac output. B - Nitrates treat angina by relaxing the coronary vessels, which improves circulation to the heart muscle. C - Calcium channel blockers treat hypertension by blocking the binding of calcium to its receptors, which causes smooth muscle relaxation. D - Beta blockers treat heart failure by reducing or blocking sympathetic nervous system stimulation to the heart and the heart's conduction system, which slows the heart rate and increases blood flow to the cardiac muscle

A nurse is reinforcing teaching about butorphanol tartrate with a client who is in labor. Which of the following client statements indicates an understanding of the teaching? A - "This medication might make me dizzy." B - "This medication might cause me to breathe very fast." C - "This medication will last for 10 to 12 hours." D - "This medication will cause my stools to be loose and watery."

A - "This medication might make me dizzy."

A nurse is caring for a client who is 2 days postoperative following a hip arthroplasty. When a news report about military action comes on the television, the client says to the nurse, "My youngest child died 6 months ago while serving in the military." Which of the following responses should the nurse make? select all A - "This must be a very difficult time for you." B - "Your child's death must be a terrible loss." C - "It's just awful what is going on in the world." D - "You need to focus on getting better." E - "Tell me something you remember about your child."

A - "This must be a very difficult time for you." B - "Your child's death must be a terrible loss." E - "Tell me something you remember about your child."

A nurse in an antepartum clinic is reinforcing teaching about the purpose of testing for blood type, Rh factor, and irregular antibodies with a client who is at 8 weeks of gestation. Which of the following information should the nurse include in the teaching? A - "This test identifies if your fetus is at risk for developing erythroblastosis fetalis." B - "This test identifies clients who are at risk for developing gestational hypertension." C - "This test identifies if your fetus might develop hypoglycemia after birth." D - "This test identifies clients who might develop polyhydramnios."

A - "This test identifies if your fetus is at risk for developing erythroblastosis fetalis." Rational A - The client should undergo blood testing to determine any unexpected findings. Testing for maternal blood type, Rh factor, and antibodies will identify if the fetus is at risk for developing erythroblastosis fetalis or hyperbilirubinemia. B - Testing for blood type, Rh factor, and antibodies does not identify clients who are at risk for developing gestational hypertension. Routine blood pressure monitoring at each visit will assist with identifying risks for hypertension. C - Testing for blood type, Rh factor, and antibodies does not identify if the fetus is at risk for developing hypoglycemia following birth. Tests such as a glucose tolerance test will assist with identifying risks for hypoglycemia. D - Testing for blood type, Rh factor, and antibodies does not identify clients who are at risk for developing polyhydramnios. An ultrasound is indicated for identifying polyhydramnios.

A nurse is collecting data from a newborn who is 8 hr old. Which of the following findings should the nurse report to the provider? A - Vernix in the skin folds B - Positive Moro reflex C - Apneic episode of 10 seconds D - Apical heart rate of 90/min while crying

D - Apical heart rate of 90/min while crying

A nurse is reinforcing teaching with a client who has asthma and a new prescription for an albuterol inhaler. Which of the following instructions should the nurse include in the teaching? select all A - "Use this medication first, before other asthma medications." B - "Rinse your mouth after using the inhaler." C - "Dizziness is an expected response when you first start this medication." D - "Use the inhaler after you finish exercising." E - "Limit your intake of caffeine-containing products while taking this medication."

A - "Use this medication first, before other asthma medications." B - "Rinse your mouth after using the inhaler." E - "Limit your intake of caffeine-containing products while taking this medication." Rational A - "Use this medication first, before other asthma medications" is correct. The nurse should instruct the client to use the albuterol, which is a bronchodilator, at least 5 min prior to administering other asthma medications. Albuterol will open the client's airways, allowing better absorption of the medications. B - "Rinse your mouth after using the inhaler" is correct. The nurse should instruct the client to rinse their mouth after administering albuterol because it can cause dryness. C - "Dizziness is an expected response when you first start this medication" is incorrect. The nurse should instruct the client to report dizziness to the provider because this is an adverse effect of the medication. D - "Use this inhaler after you finish exercising" is incorrect. The nurse should instruct the client to use albuterol prior to exercising to prevent exercise-induced bronchospasms. E - "Limit your intake of caffeine-containing products while taking this medication" is correct. The nurse should reinforce with the client to limit intake of caffeine while taking albuterol because it can cause increased nervousness and restlessness.

A nurse is reinforcing teaching with a newly admitted client who has generalized anxiety disorder. Which of the following statements should the nurse make? A - "We will demonstrate for you how to use relaxation techniques." B - "Someone will be here to work with you when you experience flashbacks." C - "Aversion therapy will be used to decrease your anxiety level." D -"Response prevention therapy will help you control your impulses."

A - "We will demonstrate for you how to use relaxation techniques."

A nurse is preparing to reinforce teaching with a client who is 1 day postoperative following a bilateral mastectomy with drains. Which of the following information should the nurse plan to include in the teaching? A - "Wear protective gloves if working in the garden." B - "Begin range-of-motion exercises of the shoulder while the drains are still in place." C - "Someone else should brush your hair for you for 2 weeks after surgery." D - "Perform wall exercises by standing 18 inches away while pushing yourself away from the wall."

A - "Wear protective gloves if working in the garden." Rational A - The nurse should instruct the client how to limit the risk of developing complications, such as infection, that could cause lymphedema. One way to do this is to wear protective gloves while washing dishes and working in a garden. B - The nurse should instruct the client to begin range-of-motion exercises of the shoulder after the drains are removed. The client can begin range-of-motion exercises of the elbow, wrist, and hand within 24 hr after surgery. C - The nurse should instruct the client to perform ADLs, such as combing hair, brushing teeth, and face washing to promote independence. The nurse should caution the client not to raise their elbow above the shoulder when performing ADLs until the drains are removed. D - The nurse should instruct the client to avoid exercises that cause heavy lifting or straining of the surgical sites. The client should perform wall exercises such as front- and side-wall climbing, in which hands are moved up and down the wall while the client stands 15.24 to 30.48 cm (6 to 12 in) away from the wall.

A charge nurse is talking with two assistive personnel (AP) who are angry about the way lunch breaks are scheduled on the unit. Which of the following statements bub the charge nurse demonstrates the use of compromise? A - "You can take turns going to lunch first every other week." B - "Whoever has seniority should go to lunch first." C - "You should try to work out the lunch schedule between the two of you." D - "Can we discuss this tomorrow? I need some time to think about the schedule."

A - "You can take turns going to lunch first every other week."

A nurse is reinforcing teaching with the guardian of a school age child who has tonsillectomy. Which of the following information should the nurse include in the education? A - "You should discourage your child from coughing." B - "You should limit your child's activity for 2 days." C - "You should encourage your child to use a straw while drinking fluids." D - "You should give your child citrus juice to drink after the procedure."

A - "You should discourage your child from coughing." Rational A - The nurse should instruct the guardian to discourage their child from coughing or clearing their throat because it can cause hemorrhage. B - The nurse should instruct the guardian to limit their child's activity for 10 days following the procedure because hemorrhage can occur. C - The nurse should instruct the guardian to avoid allowing their child to use a straw for drinking fluids following a tonsillectomy because placing objects into their mouth can cause hemorrhage. D - The nurse should instruct the guardian to avoid giving their child citrus juice to drink because it can cause discomfort and irritation.

A nurse is reinforcing discharge teaching to a client who has a new prescription for sublingual nitroglycerin. Which of the following instructions should the nurse include in the teaching? A - "You will need to store your tablets at room temperature." B - "You may place your tablets in a pill box with your other medications." C -"You should keep the tablets in your pocket when you are away from home." D - "You should count your tablets at least twice a week to avoid running out of them."

A - "You will need to store your tablets at room temperature." Rational A - Nitroglycerin tablets lose their potency if exposed to air, heat, or moisture; therefore, the nurse should reinforce to the client that tablets should be stored at room temperature. B - Nitroglycerin tablets lose their potency if exposed to air, heat, moisture, or if they are mixed with other capsules and tablets. The nurse should reinforce to the client that the tablets should be stored alone in their original glass container. C - Nitroglycerin tablets lose their potency if exposed to air, heat, or moisture. To prevent exposure to body heat, the nurse should reinforce to the client that tablets should not be kept close to the body. D - Nitroglycerin tablets lose their potency if exposed to air, heat, or moisture; therefore, the nurse should reinforce to the client that the tablets should be kept in their original glass container with the lid closed tightly.

A nurse is caring for a client who is at 30 weeks of gestation. Which of the following findings should the nurse report to the provider? A - 2+ urinary protein B - Leukorrhea C - Spider nevi D - 30 cm fundal height

A - 2+ urinary protein

A nurse enters a client's room at the beginning of a shift. Which of the following findings requires intervention by the nurse? A - A capped bottle of sterile water that was opened 36 hr ago B -A urinary catheter drainage bag that is hanging on the lower portion of the bed frame C - A peripheral IV catheter that was inserted 2 days ago D - A wound dressing that requires frequent changes and is secured using Montgomery straps

A - A capped bottle of sterile water that was opened 36 hr ago

A nurse is preparing an in-service for a group of newly licensed nurse about child physical maltreatment. Which of the following children should the nurse identify as having a risk factor for maltreatment? A - A child who has a congenital disability B - A child who has acute bronchitis C - A child who has otitis media D - A child who was born at 39 weeks of gestation

A - A child who has a congenital disability Rational A - The nurse should include that a child who has a disability is at an increased risk for child maltreatment. Other risk factors include hyperactivity, prematurity, young children, and infants. B - Acute illness is not a risk factor for child maltreatment. Risk factors include disability, hyperactivity, prematurity, young children, and infants. C - Acute illness is not a risk factor for child maltreatment. Risk factors include disability, hyperactivity, prematurity, young children, and infants. D - Prematurity, or birth before 38 weeks of gestation, is a risk factor for child maltreatment. Other risk factors include disability, hyperactivity, prematurity, young children, and infants.

A nurse on a pediatric unit is assisting with the admission of four children. For which of the following children should the nurse initiate droplet precautions? A - A child who has pertussis B - A child who has roseola C - A child who has molluscum contagiosum D - A child who has Rocky Mountain spotted fever

A - A child who has pertussis Rational A - The nurse should initiate droplet precautions for a child who has pertussis to decrease the risk of transmitting the infection to others on the unit. Pertussis, or whooping cough, is a bacterial infection that is transmitted via direct contact with or exposure to respiratory secretions from an infected person. Manifestations include fever, sneezing, and a cough that becomes more severe. B - Roseola is a herpes virus infection that causes a rash and high fever. Other manifestations include lymphadenopathy, sore throat, and cough. Standard precautions are used when caring for a child who has roseola. C -Molluscum contagiosum is an infection caused by the poxvirus. Manifestations include flesh-colored papules on the face, trunk, or extremities. Molluscum contagiosum is transmitted via direct skin to skin contact. Contact precautions are used when caring for a child who has molluscum contagiosum. D - Rocky Mountain spotted fever is a bacterial infection that is most commonly transmitted via a tick bite. Manifestations include fever, myalgia, and a rash that primarily appears on the extremities. Standard precautions are used when caring for a child who has Rocky Mountain spotted fever.

A nurse is preparing to provide care for a group of clients. Which of the following clients should the nurse see first? A - A client who has a blood glucose of 68 mg/dL and reports mild sweating B - A client who has a calcium level of 10 mg/dL and reports a headache C - A client who has acute glomerulonephritis and reports output of reddish-brown urine D -A client who has macular degeneration and reports difficulty seeing shapes

A - A client who has a blood glucose of 68 mg/dL and reports mild sweating Rational A - When using the urgent vs. nonurgent approach to client care, the nurse should see the client who has a blood glucose of 68 mg/dL first. A blood glucose of less than 70 mg/dL indicates hypoglycemia. Early findings include diaphoresis, anxiety, and hunger, which can progress to confusion and seizures. The nurse should provide the client with a quick-acting carbohydrate, such as 6 oz of juice, to increase the client's blood glucose. B - A calcium level of 10 mg/dL is nonurgent because it is within the expected reference range of 9 to 10.5 mg/dL, and a headache is usually a non-urgent finding. Therefore, there is another client that the nurse should see first. C - Reddish-brown urine is nonurgent because it is an expected finding for a client who has glomerulonephritis. Therefore, there is another client the nurse should see first. Acute glomerulonephritis is an inflammation of the glomerular capillaries that can also cause chills and flank pain. D - Difficulty seeing shapes is nonurgent because it is an expected finding for a client who has macular degeneration. Therefore, there is another client the nurse should see first. The client might also report blurred vision and loss of central vision.

A nurse is assisting in planning care for a group of clients. Which of the following clients should the nurse recommend for an interprofessional client care conference? A - A client who has cystic fibrosis B - A client who has appendicitis C - A client who has pyelonephritis D - A client who has a kidney stone

A - A client who has cystic fibrosis Rational A - The nurse should recommend interprofessional care for a client who has a chronic disease, such as cystic fibrosis (CF). CF is a genetic disease that affects many organs, including the lungs, pancreas, liver, salivary glands, and reproductive system. Potential complications of CF include respiratory infections, intestinal obstruction, poor growth, malnourishment, cirrhosis, osteoporosis, and diabetes mellitus. Management of CF is complex and requires a lifelong multidisciplinary approach. B - Appendicitis is an acute inflammation of the appendix. Manifestations include pain in the lower right quadrant of the abdomen, fever, nausea, and vomiting. This client has an acute illness and does not require an interprofessional client care conference. C - Acute pyelonephritis is an infection in the kidneys. Manifestations include fever, flank pain, chills, nausea, vomiting, and fatigue. This client has an acute illness and does not require an interprofessional client care conference. D - Manifestations of a kidney stone can include flank pain, nausea, and vomiting. Treatment includes increased fluid intake, analgesics, insertion of a urinary stent, and extracorporeal shock waves to break up the stone. This client has an acute illness and does not require an interprofessional client care conference.

A nurse in a prenatal clinic is caring for a group of clients. Which of the following clients should the nurse recommend the provider see first? A - A client who is at 37 weeks of gestation and reports a persistent headache B - A client who is at 38 weeks of gestation and reports irregular uterine contractions C - A client who is at 12 weeks of gestation and reports abdominal cramping D - A client who is at 26 weeks of gestation and reports periodic numbness in the fingers

A - A client who is at 37 weeks of gestation and reports a persistent headache

A nurse is reinforcing discharge teaching to prevent dumping syndrome for a client following a partial gastrectomy for ulcers. Which of the following information should the nurse include in the teaching? A - Avoid liquids at mealtimes. B - Exclude eating starchy vegetables. C - Avoid eating high-protein meals. D - Plan to increase intake of sweetened fruits.

A - Avoid liquids at mealtimes.

A nurse is assisting the charge nurse with the preparation for an inservice about negligence for a group of newly licensed nurses. Which of the following scenarios should the charge nurse use as an example to identify negligence? A - A nurse does not notify the provider of a change in condition for a client who has schizophrenia. B - A nurse delegates an assistive personnel to sit with a client who has bulimia nervosa during mealtimes. C - A nurse administers an antianxiety medication to a restless client who has given implied consent. D - A nurse does not document completion of an incident report about a recent fall in the client's medical record.

A - A nurse does not notify the provider of a change in condition for a client who has schizophrenia.

A nurse is collecting data from a client who is postoperative. Which of the following findings should the nurse identify as an indication that the client might be experiencing delirium? A - Abrupt alteration in level of consciousness B - Flat affect C - Slow and deliberate speech D - Anhedonia

A - Abrupt alteration in level of consciousness Rational A - Delirium is characterized by a change in cognition that occurs over a short period of time. It always results from a secondary physiological condition (e.g., infection, surgery, prolonged hospitalization, hypoxia, fever, or an adverse effect of medications) and is a transient disorder. Although delirium can occur with clients in any age group, it is more common in older adults. Delirium is characterized by agitation, restlessness, illusions, hallucinations, or alterations in memory. It is a medical emergency and the nurse should report the finding immediately. B - A flat affect is a manifestation of a client who has depression and is present when the client's facial expression is blank or displays no emotions. This is not an expected finding in a client who is experiencing delirium. A client who has delirium has rapid swings of emotions, hallucinations, and delusions. C - Slow, deliberate speech is a manifestation of a client who has dementia with difficulty finding the appropriate words to convey thoughts and needs. This is not an expected finding in a client who is experiencing delirium. A client who has delirium has rapid, incoherent speech that is rambling in nature. D - Anhedonia is an inability to take pleasure in life, which is a manifestation of clients who have depression. It is not an expected finding in a client who is experiencing delirium.

A nurse is collecting data from a client who received protamine 4 hr ago. Which of the following findings indicates a therapeutic response to the medication? A - Absence of blood in urine B - Decreased heart rate C - Decreased nausea D - Absence of headache

A - Absence of blood in urine Rational A - Protamine is a heparin antidote that binds with heparin and reverses its anticoagulant properties. The absence of blood in the urine indicates a therapeutic response to the medication. B - Protamine reverses the anticoagulation properties of heparin. Therefore, a reduction in heart rate is not a therapeutic response to the medication. C - Protamine reverses the anticoagulation properties of heparin. Therefore, a reduction in nausea is not a therapeutic response to the medication. D - Protamine reverses the anticoagulation properties of heparin. Therefore, the absence of headache is not a therapeutic response to the medication.

A nurse is caring for a client and administers penicillin IM. The client begins exhibiting hives and has severe difficulty breathing. After establishing a patent airway, which of the following actions should the nurse take next? A - Administer epinephrine. B - Monitor the client's vital signs. C - Monitor the client's oxygen saturation level. D - Administer an antihistamine.

A - Administer epinephrine.

A nurse is assisting with the admission of a toddler. Which of the following actions should the nurse take? A - Administer humidified oxygen. B - Place the toddler in contact isolation. C - Examine the toddler's throat for swelling. D - Initiate NPO status.

A - Administer humidified oxygen. Rational A - The nurse should administer humidified oxygen to a toddler who has suspected epiglottitis and has manifestations of respiratory distress, such as decreased oxygen saturation, increased respiratory rate, and substernal retractions. Humidification moistens the airway and relieves discomfort. B - The nurse should implement droplet precautions for a toddler who has suspected epiglottitis. C - The nurse should avoid examining the throat of a toddler who has suspected epiglottitis, because it could compromise the toddler's airway. D - The nurse should provide a toddler who has suspected epiglottitis with plenty of fluids to prevent dehydration.

A nurse is caring for a preschooler who has a new diagnosis of asthma. Which of the following medications should the nurse instruct the parent to administer for an acute asthma attack? A - Albuterol B - Fluticasone C - Cromolyn sodium D - Montelukast

A - Albuterol

A nurse on a pediatric unit is assisting with the care of a group of clients. Which of the following clients would benefit most from an interprofessional care conference? A - An infant who has cystic fibrosis and is continuing to lose weight B - An infant who has pneumonia and is receiving IV antibiotics C - A school-age child who has sickle cell disease and is scheduled to receive a blood transfusion D -A school-age child who has spina bifida and whose parent needs to learn to perform intermittent catheterization

A - An infant who has cystic fibrosis and is continuing to lose weight

A nurse is assisting with the plan of care for a client who has Clostridium difficile (C. difficile). Which of the following supplies should the nurse ensure is available in the room? A - Antimicrobial soap B - N95 respirator mask C - Sterile gloves D - Alcohol-based hand sanitizer

A - Antimicrobial soap Rational A - The nurse must perform hand hygiene by washing their hands with antimicrobial soap and water after each client contact or contact with the surfaces in the client's room. Alcohol-based hand sanitizers are not effective against spore-forming organisms like C. difficile. The nurse should also perform appropriate hand hygiene after removal of personal protective equipment, when hands are visibly soiled, or after caring for clients with known or suspected infectious diarrhea. B - The specially fitted N95 respirator mask has a higher filtration rate than a regular face mask. The nurse should wear this type of mask when caring for a client who requires airborne precautions, such as a client who has tuberculosis. C - The use of sterile gloves is not cost effective or necessary for a client who requires transmission-based precautions. The nurse should use clean gloves when providing care for a client who has C. difficile. D - Alcohol-based hand sanitizers are not effective against spore-forming organisms like C. difficile. The nurse should use an antimicrobial soap and water when performing hand hygiene after caring for a client who has C. difficile.

A nurse is planning to implement droplet precautions for a client who has manifestations of pertussis. Which of the following interventions should the nurse include when contributing to the plan of care? A - Apply a mask on the client if transport is needed. B - Wear a mask when working within 4 feet of the client. C - Don a gown when visiting with the client. D - Wear an N95 mask when entering the client's room.

A - Apply a mask on the client if transport is needed.

A nurse is assisting with the plan of care for a client who is immobile and has urinary incontinence. Which of the following actions should the nurse recommend including in the plan to prevent the development of skin breakdown? A - Apply a moisture barrier ointment to the client's skin. B - Clean the skin with chlorhexidine solution after each episode of incontinence. C - Change the client's position every 4 hr. D - Request a prescription for an indwelling urinary catheter.

A - Apply a moisture barrier ointment to the client's skin. Rational A - Skin that is left in contact with urine for prolonged periods of time is at risk for maceration and breakdown. Cleansing the skin and removing items that are wet (e.g., incontinence pads, sheets, undergarments) is a priority to prevent breakdown. The nurse should apply moisture-barrier ointments and creams to the client's skin after cleansing to prevent urine from coming in contact with the skin. B - The nurse should clean the client's skin with soap and water after each episode of incontinence. Chlorhexidine is used to cleanse hands and can be used for bathing. However, chlorhexidine should only be used on the perineum for thorough cleansing and not after each episode of incontinence. C - The nurse should change the client's position every 2 hr to prevent skin breakdown. D - Catheterization of the bladder can introduce bacteria into the bladder, creating a risk for bacteremia, a life-threatening bacterial infection of the blood. Indwelling retention catheterization results in a high rate of urinary tract infections as compared to intermittent catheterization.

A nurse is reinforcing teaching about joint protection with a client who has an acute exacerbation of rheumatoid arthritis. Which of the following information should the nurse include in the teaching? A - Apply cold packs to the inflamed joints. B - Participate in high-impact exercise. C - Carry a hand purse rather than a shoulder bag. D - Sleep on a soft foam mattress.

A - Apply cold packs to the inflamed joints.

A nurse is preparing to administer erythromycin ophthalmic ointment 0.5% to a newborn. Which of the following actions should the nurse take? A - Apply the ointment in the lower conjunctival sac of each eye. B - Obtain a written consent from the guardian prior to administering the ointment. C - Wipe the excess ointment off immediately to prevent irritation. D - Administer the ointment starting at the outer canthus of the eye to the inner canthus.

A - Apply the ointment in the lower conjunctival sac of each eye. Rational A - The nurse should administer the ointment in the lower conjunctival sac by gently squeezing the tube starting at the inner canthus and moving to the outer canthus. B - Eye prophylaxis is required and mandated in all states. Written consent is not necessary. The guardian can refuse by signing a form. C - The nurse should wait 1 min before wiping off the excess ointment. D - The nurse should administer the ointment in the lower conjunctival sac by gently squeezing the tube starting at the inner canthus and moving to the outer canthus.

A nurse is preparing to apply a belt restraint for a client. Which of the following actions should the nurse plan to take? A - Apply the restraint over the client's clothing. B - Attach the restraint to the side rails of the bed. C - Allow three fingerbreadths between the restraint and the client. D - Position the restraint across the chest at the client's nipple line.

A - Apply the restraint over the client's clothing. Rational A - The nurse should apply the restraint over the client's clothing. This helps to decrease the client's risk of skin irritation. The nurse should also ensure that wrinkles in the client's clothing are smoothed out. B - The nurse should attach the restraint to a moveable part of the frame of the client's bed. Securing a restraint to a side rail of the bed increases the risk for injury if the head of the bed is adjusted. C - The nurse should allow no more than two fingerbreadths of space between the restraint and the client to allow for adequate restraint while not compromising circulation. The nurse should monitor a client in restraints for manifestations of compromised circulation, including pallor, cool skin, or client report of pain or numbness. D - The nurse should place the belt restraint across the client's waist, not across the chest or abdomen. The nurse should apply the restraint while the client is in a sitting position and place it over the client's clothing.

A nurse is contributing to the plan of care for a client who is newly admitted to a rehabilitation facility. Which of the following actions should the nurse take first? A - Ask the client to identify his goals for recovery. B - Select interventions to match the priority client needs. C - Reinforce information with the client about expected treatment outcomes. D - Recommend referrals to address the client's needs.

A - Ask the client to identify his goals for recovery.

A nurse is reviewing the plan of care for a client who is at risk for increased intracranial pressure following a closed-head injury. Which of the following interventions should the nurse identify as contraindicated for this client? A - Assist the client to cough and deep breathe. B - Elevate the head of the client's bed to 30°. C - Perform passive arm range-of-motion exercises for the client. D - Log roll the client slowly when repositioning.

A - Assist the client to cough and deep breathe. Rational A - A closed-head injury is caused by blunt trauma to the head. As a result, edema can occur within the brain and lead to increased intracranial pressure (ICP). The client should avoid deep breathing, coughing, performing Valsalva maneuvers, and blowing their nose because these actions can increase ICP. An initial sign of increased ICP is often a decrease in the level of consciousness. B - The nurse should position the client in a neutral position with the head of the bed elevated. This allows the cerebral spinal fluid to flow freely through the brain and spinal cord and minimize pressure within the central nervous system. C - Passive range-of-motion exercises of the arms promotes adequate circulation and reduces complications of immobility. These exercises do not cause an increase in intracranial pressure. The nurse should avoid flexion of the client's neck and hips because these positions could potentially increase ICP by restricting the free flow of cerebral spinal fluid through the brain and spinal cord. D - Treatment of increased ICP focuses on decreasing intracranial pressure. The nurse should slowly log roll the client to avoid flexing the client's hips and neck or turning their head when repositioning. This allows the cerebral spinal fluid to flow freely through the brain and spinal cord, and decreases the risk of a further increase in the ICP. Sudden movement can also cause an increase in ICP.

A nurse is contributing to the plan of care for a client who has a prescription for a 24 hr urine specimen. Which of the following interventions should the nurse plan to include? Select all A - Begin the timed collection by discarding the first specimen. B - Post the times for urine collection above the toilet in the client's bathroom. C - Document volume estimations of missed voids. D - Obtain a clean specimen collection container for use during the test. E - Remove feces or toilet paper that is in the specimen collection container.

A - Begin the timed collection by discarding the first specimen. B - Post the times for urine collection above the toilet in the client's bathroom. D - Obtain a clean specimen collection container for use during the test.

A nurse is reinforcing teaching with the parents of a child who has cystic fibrosis and si taking pancrelipase as a pancreatic enzyme replacement. The nurse should plan to inform the child's parents that the therapeutic effects of this medication can be evaluated by which of the following? A - Blood glucose levels B - Amount and consistency of stools C - Chloride sweat tests D - BUN and creatinine clearance tests

A - Blood glucose levels Rational A - Blood glucose levels do not indicate the therapeutic effects of pancrelipase. However, cystic fibrosis-related diabetes mellitus results from insulin resistance and deficiency. B - Recording the amount and consistency of the child's stools will help determine the effectiveness of pancrelipase, which is taken to decrease the bulk of feces. C - Chloride sweat tests do not indicate the therapeutic effects of pancrelipase. However, the results of these tests can be used to determine a diagnosis of cystic fibrosis. D - BUN and creatinine clearance tests do not indicate the therapeutic effects of pancrelipase.

A nurse is collecting data from a client who is at 36 weeks of gestation during a prenatal examination. Which of the following findings should the nurse report to the provider? A - Blurred vision B - Nonpitting ankle edema C - 10 fetal movements in 2 hr D - Leg cramps

A - Blurred vision

A nurse is collecting data from a client who has a new prescription for metoprolol. The nurse should identify that which of the following findings is an adverse effect of this medication? A - Bradycardia B - Epistaxis C - Decreased urinary output D - Increased blood pressure

A - Bradycardia Rational A - Bradycardia is a common adverse effect of metoprolol and therefore is contraindicated for clients who have sinus bradycardia. The nurse should take the client's apical pulse before administering metoprolol and notify the charge nurse for a pulse rate less than 50/min. B - Metoprolol can cause sinus congestion rather than epistaxis. C - Metoprolol can cause urinary frequency rather than decreased urinary output. D - Metoprolol can cause hypotension rather than hypertension.

A nurse in a provider's office is reinforcing teaching with a client who has been taking omeprazole for the past 2 years. The nurse should remind the client about the importance of maintaining an adequate intake of which of the following minerals? A - Calcium B - Iron C - Phosphorus D - Chromium

A - Calcium Rational A - Long-term use of proton pump inhibitors can result in osteoporosis. It is important for the client to maintain an adequate intake of calcium and vitamin D to help prevent fractures resulting from bone loss. Calcium is the most plentiful element in the body. It is found in many foods, such as dairy and foods that are fortified with calcium, including cereals and orange juice. B - Iron intake is especially important for female clients of childbearing age and for clients who have had heavy or chronic blood loss. Clients can increase absorption of iron by consuming iron with foods that are high in vitamin C, such as orange juice or tomatoes. Omeprazole does not cause iron-deficiency anemia. C - Phosphorus is abundant in many foods, such as animal proteins, dairy, and legumes. Therefore, phosphorus deficiency is very rare. Omeprazole does not cause a phosphorus deficiency. D - Chromium intake is important for clients who have insulin resistance and poor glucose tolerance. It is found more often in foods that are unrefined because processing depletes chromium. Omeprazole does not cause chromium deficiency.

A nurse is assisting with the care of a client who has preeclampsia and is receiving magnesium sulfate via continuous IV infusion. The client has a respiratory rate of 10/min. Which of the following medications should the nurse expect a charge nurse to administer to the client? A - Calcium gluconate B - Naloxone C - Protamine sulfate D - Diphenhydramine

A - Calcium gluconate

A nurse is preparing to administer scheduled medication to a client. Which of the following prescriptions should the nurse verify with the provider? A - Ceftriaxone B - Diltiazem C - Pioglitazone D - Hydrocodone 5 mg/acetaminophen 500 mg

A - Ceftriaxone Rational A - Clients who have a severe sensitivity to penicillin can have a cross-sensitivity reaction to ceftriaxone, a cephalosporin. Therefore, the nurse should contact the provider to clarify the prescription. B - The nurse should administer diltiazem because the client's heart rate and blood pressure are within the expected reference ranges. C - The nurse should administer pioglitazone because the client's blood glucose level is within the expected reference range. D - The nurse should administer hydrocodone and acetaminophen to manage the client's pain because the client's respiratory rate is within the expected reference range.

A nurse is reinforcing teaching about perineal care with a client who is 24 hr postpartum following a vaginal delivery. Which of the following instructions should the nurse include select all A - Change the pad after each voiding. B - Clean from back to front. C - Spray a povidone-iodine solution on the area twice daily. D - Apply ice packs several times daily. E - Wash the area using a squeeze bottle of warm water after each voiding.

A - Change the pad after each voiding. D - Apply ice packs several times daily. E - Wash the area using a squeeze bottle of warm water after each voiding. Rational A - Change the pad after each voiding is correct. The client should change the perineal pad after each voiding to prevent infection. B - Clean from back to front is incorrect. Wiping from front to back decreases the chances of transmitting fecal organisms to other areas, such as the urinary meatus. C - Spray a povidone-iodine solution on the area twice daily is incorrect. Povidone-iodine solution is an antimicrobial agent primarily used preoperatively to rid the site of germs. Use of a povidone-iodine solution on the perineum will cause unnecessary irritation to the tissue. The client should wash the perineal area with a mild soap and water once daily. D - Apply ice packs several times daily is correct. The client should apply ice packs to the perineal area to decrease edema, increase comfort, and provide an anesthetic effect. E - Wash the area using a squeeze bottle of warm water after each voiding is correct. Washing the perineal area using a squeeze bottle of warm water after each voiding can decrease the risk for infection.

A nurse is preparing to reinforce preoperative teaching with a client. Which of the following actions is the nurse's priority? A - Check the client's comfort level. B - Determine the client's motivation to learn. C - Check the client's preferences for learning methods. D - Determine the client's health literacy.

A - Check the client's comfort level. Rational A - When using Maslow's hierarchy of needs, the nurse should recognize that checking the physiological needs of the client, such as pain, is the priority. B - Determining the client's motivation to learn is important because the client's level of motivation determines attention and adherence to the teaching. According to Maslow's hierarchy of needs, motivation to learn is at the level of self-actualization. C - Checking the client's preferences for learning methods is important because learning is enhanced when the client's learning style is incorporated in the teaching. According to Maslow's hierarchy of needs, learning methods are at the level of self-actualization. D - Determining the client's health literacy is important because health literacy determines the client's ability to understand the teaching. According to Maslow's hierarchy of needs, health literacy is at the level of self-actualization.

A nurse is caring for a newborn who is large for gestational age and is jittery. Which of the following actions should the nurse take first? A - Check the newborn's blood glucose level. B - Place the newborn under a radiant warmer. C -Provide nonnutritive sucking. D - Swaddle the newborn.

A - Check the newborn's blood glucose level.

A nurse is collecting data from a client who has schizophrenia. Which of the following is the priority finding for the nurse to report? A - Command hallucinations B - Delusions C - Clang associations D - Neologisms

A - Command hallucinations Rational A - Command hallucinations are auditory hallucinations instructing the client to take some form of action. The client's report of command hallucinations indicates that they are at the greatest risk for harming themselves or others. This is the priority finding for the nurse to report to the provider. B - Delusions are persistent, false beliefs. The client's report of delusions is important for the nurse to report so that all members of the interprofessional team know about the client's delusion. However, another finding is the priority. C - Clang associations are a positive symptom of schizophrenia in which the client selects to communicate by using words that rhyme rather than convey real meaning. This often results in nonsensical statements. Clang associations are an important finding for the nurse to report so that all members of the interprofessional team understand how the client is communicating. However, another finding is the priority. D - Neologisms are made-up words that have meaning only to the client. This is a positive symptom of schizophrenia. Neologisms are created by the client's altered thought processes and can be a communication barrier. Communicating using neologisms is an important finding for the nurse to report so that all members of the interprofessional team know how the client is communicating. However, another finding is the priority.

A nurse is assisting with the admission of a client who has schizophrenia. Which of the following actions should the nurse take first? A - Conduct an abnormal involuntary movement scale test. B - Discuss behavioral expectations with the client. C - Orient the client to unit routines. D - Encourage the client to attend group art sessions.

A - Conduct an abnormal involuntary movement scale test. Rational - The first action the nurse should take when using the nursing process is to collect data. The abnormal involuntary movement scale (AIMS) test is a data collection tool used to guide medication therapy for clients who are prescribed antipsychotic medications. Therefore, the first action the nurse should take is to conduct the AIMS test.

A nurse is assisting with the care of a client who has a right-sided chest tube to closed drainage. The nurse notes a rise in fluid level of the water seal chamber when the client inhales. Which of the following actions should the nurse take? A - Continue to monitor the client. B - Retape the chest tube connections. C - Reposition the client toward the left side. D - Clamp the chest tube near the water seal.

A - Continue to monitor the client. Rational A - The fluid in the water seal chamber rises 5 to 10.2 cm (2 to 4 in) during inhalation and falls during exhalation. This is a process called tidaling. Tidaling in the water seal chamber shows an effective connection between the pleural space and the drainage chamber, indicating a patent drainage system. An absence of tidaling can indicate a fully expanded lung or an obstruction in the chest tube. There is no action required by the nurse at this time other than to continue to monitor the client. B - The nurse should ensure the chest tube connections are taped and monitor the client for a leak in the system. Continuous bubbling in the water seal chamber, rather than fluctuation with respirations, is an indication of an air leak and the charge nurse should be notified. The chest tube connections might need to be retaped if a loose connection occurs. C - Repositioning the client can promote comfort and reduce the risk for pressure injuries as well as promote full expansion of the non-injured lung. However, repositioning is not indicated in this situation. D - The nurse should not clamp the chest tube as this can cause damage to the tube and might result in client injury.

A nurse is collecting data from a client who is receiving enteral tube feedings. Which of the following findings should the nurse identify as an indication that the tube is displaced? A - Coughing B - Diarrhea C - Peripheral edema D - Abdominal cramping

A - Coughing Rational A - The nurse should identify that coughing or vomiting can be an indication that the client's tube feeding is displaced. The nurse should replace the tube and confirm placement prior to restarting the tube feeding formula. B - Diarrhea can be caused by medications, hyperosmolar formula, antibiotics, or a possible bacterial infection due to contamination of the tube feeding formula. C - Peripheral edema can be caused by fluid overload related to excess fluids. D - Abdominal cramping can be caused by cold tube feeding formula or a rapid rate of infusion.

A nurse is collecting data from a child who has acute appendicitis. Which of the following findings should the nurse expect? A - Decreased bowel sounds B - Decreased heart rate C - Hematemesis D - Rectal bleeding

A - Decreased bowel sounds Rational A - The nurse should expect a child who has acute appendicitis to exhibit decreased bowel sounds due to possible peritonitis and partial obstruction of the small bowel by the inflamed appendix. B - The nurse should expect a child who has acute appendicitis to have an increased heart rate due to pain and possible infection. C - The nurse should expect a child who has peptic ulcer disease to exhibit hematemesis. D - The nurse should expect a child who has irritable bowel syndrome to exhibit rectal bleeding.

A nurse is assisting in the plan of care regarding bowel retraining for a client who has a cervical spinal cord injury. Which of the following interventions should the nurse plan to implement first? A - Determine the client's daily elimination habits. B - Administer a suppository to the client 30 min prior to defecation time. C - Offer the client 4 oz of warm prune juice to promote elimination. D - Provide dietary bulk to the client to ease the passage of stool.

A - Determine the client's daily elimination habits.

A nurse is caring for a client who is scheduled for surgery and is experiencing anxiety. Which of the following interventions should the nurse identify as the priority? A - Determine the client's understanding of the procedure. B - Encourage the client to express his feelings. C - Allow the client's family to stay with him. D - Provide music as a distraction.

A - Determine the client's understanding of the procedure.

A nurse is reviewing a client's medical record at the beginning of the shift. Which of the following provider prescriptions should the nurse expect? A - Discontinue clozapine. B - Transfuse 2 units of packed RBCs. C - Limit sodium to 1,850 mg daily. D - Administer potassium 40 mEq PO twice daily.

A - Discontinue clozapine. Rational A - The nurse should identify a WBC count of 2,900/mm3 as below the expected reference range of 5,000 to 10,000/mm3. The client's low WBC count, in addition to the client's report of influenza-like findings, such as sore throat, fever, malaise, and lethargy are associated with agranulocytosis, an adverse effect of clozapine. The nurse should place the client in protective isolation and expect the provider to discontinue the clozapine. B - The nurse should identify a hemoglobin (Hgb) of 15 g/dL and a hematocrit (Hct) of 45% are within expected reference ranges of 12 to 18 g/dL and 37% to 52% respectively. Therefore, there is no reason to anticipate the provider will prescribe a blood transfusion. Blood transfusions are administered to increase circulating volume following blood losses, to correct severe anemia, or to replace cellular components such as clotting factors. C - The nurse should identify a sodium level of 136 mEq/L is within the expected reference range of 136 to 145 mEq/L. Therefore, there is no reason for the nurse to anticipate that the provider will prescribe sodium restrictions for the client. Sodium restrictions might be prescribed for clients who have heart failure. D - The nurse should identify that a potassium level of 3.5 mEq/L is within the expected reference range of 3.5 to 5.0 mEq/L. Therefore, there is no reason for the nurse to anticipate that the provider will prescribe additional potassium supplementation. Potassium might be prescribed for clients who take a loop diuretic such as furosemide.

A nurse is caring for a client during the postpartum period. Which of the following findings should the nurse expect during the first 24 hr following delivery? Select all A - Diuresis B - Soft, boggy uterus upon palpation C - Discharge of clear, yellow fluid from the breasts D - Lochia serosa E - Lower abdominal cramping

A - Diuresis C - Discharge of clear, yellow fluid from the breasts E - Lower abdominal cramping Rational A - Diuresis is correct. This is an expected finding that results from the loss of excess fluid that is retained during pregnancy. b - Soft, boggy uterus upon palpation is incorrect. This is not an expected finding in the postpartum period and can cause excessive bleeding. C - Discharge of clear, yellow fluid from the breasts is correct. This fluid, called colostrum, is an expected finding in the postpartum period. Colostrum is present for 3 to 5 days until the mother's milk appears and can leak from the breasts beginning in the third trimester of pregnancy. D - Lochia serosa is incorrect. Lochia serosa is vaginal discharge that is pink or brown, which occurs 3 to 4 days after delivery. E - Lower abdominal cramping is correct. This is an expected finding and results from the contraction of the uterus as it decreases in size.

A nurse is making client care assignments at the beginning of a shift. Which of the following tasks should the nurse assign to an assistive personnel (AP) A - Document a client's output from a nasogastric tube. B - Update a client's plan of care following a minor surgical procedure. C - Determine the effectiveness of a client's pain medication. D - Administer milk of magnesia to a client who has constipation

A - Document a client's output from a nasogastric tube. Rational A - The nurse can assign the task of measuring and documenting a client's output to an AP because this task does not require interpretation of data or decision making. Tasks within an AP's range of function include taking vital signs, feeding, bathing, ambulation, and providing postmortem care. B - A nurse is required to update a client's plan of care as this task requires interpretation of the client's needs. The nurse should assign an AP tasks that have a low potential for client harm. C - A nurse is required to evaluate the effectiveness of care provided to a client as this requires assessment or data collection, which are outside the range of function for an AP. The nurse should assign an AP tasks that are predictable and repetitive. D - A nurse should administer medications to a client as this requires assessment or data collection, which are outside the range of function for an AP, before and after administration. The nurse should assign an AP tasks that are generally noninvasive.

A nurse in a pediatric clinic is collecting data from an infant who was recently exposed to pertussis. The nurse should recognize which of the following as a manifestation of pertussis? A - Dry cough B - Abdominal pain C - Muscle stiffness D - Swollen eyelids

A - Dry cough

A nurse is caring for a client who has chronic bronchitis and reports shortness of breath. Which of the following actions is the priority for the nurse to take? A - Elevate the head of the client's bed. B - Encourage the client to recall past calming mechanisms. C - Instruct the client to perform purse-lipped breathing exercises. D - Remind the client to limit speaking until breathing is regulated.

A - Elevate the head of the client's bed. Rational A - The priority action the nurse should take when using the airway, breathing, circulation approach to client care is to elevate the head of the client's bed. This action immediately improves the client's airway by increasing lung expansion, relaxing the chest muscles, and placing the diaphragm in the proper position to contract for increased ventilation. This position also promotes mobilization of secretions. B - The nurse should encourage the client to recall past calming mechanisms to reduce the client's anxiety; however, another action is the priority. C - The nurse should instruct the client to perform purse-lipped breathing exercises to provide the client with a sense of control over breathing and slow respirations; however, another action is the priority. D - The nurse should remind the client to limit speaking until breathing is regulated to help calm the client and reduce the client's oxygen demand; however, another action is the priority.

A nurse is contributing to the plan of care for a client who has chronic obstructive pulmonary disease (COPD) and is dyspneic. Which of the following interventions should the nurse include in the plan? A - Encourage abdominal breathing. B - Direct the client to inhale with pursed lips. C - Set the oxygen therapy at 5 L/min. D - Instruct the client to lean back when coughing.

A - Encourage abdominal breathing.

A nurse is reinforcing teaching about tracheostomy care with the parent of a toddler who has a temporary tracheostomy. Which of the following instructions should the nurse include in the teaching? A - Ensure one finger fits between the ties and the neck. B - Clean around the stoma with isopropyl alcohol. C - Perform necessary tube changes immediately after meals. D - Change the tracheostomy tube every other week.

A - Ensure one finger fits between the ties and the neck.

A nurse is assisting in the care of a client who is receiving medications via intravenous infusion. Which of the following findings at the IV site should the nurse report as manifestations of phlebitis? select all A - Erythema B - Damp dressing C - Bleeding D - Warmth E - Streak formation

A - Erythema D - Warmth E - Streak formation Rational A - Erythema is correct. Erythema is a reddened area at the insertion site and can be accompanied by throbbing, burning, and increased skin temperature. Erythema is a manifestation of phlebitis. B - Damp dressing is incorrect. A damp dressing can indicate infiltration where fluid is entering the subcutaneous tissue around the site of the venipuncture. Other indicators of infiltration include pallor, localized swelling, and decreased skin temperature. C - Bleeding is incorrect. Bleeding indicates the IV site is not intact or the tubing has become disconnected. Therefore, the nurse should notify the charge nurse to determine if the IV site needs to be secured or discontinue the IV infusion. D - Warmth is correct. Warmth and redness are indications of inflammation as a response to irritated tissue caused by the IV infusion. Warmth at the infusion site is a manifestation of phlebitis. E - Streak formation is correct. Streaks in the client's skin are a manifestation of phlebitis. Therefore, the nurse should discontinue the IV infusion. A cold compress should be applied first to minimize the blood flow at the infusion site followed by a warm compress to increase circulation.

A nurse is contributing to the plan of care for a client who has borderline personality disorder and exhibits manipulative behaviors. Which of the following interventions should the nurse include in the plan to address limit setting? A - Establish and explain consequences for the client's behavior. B - Teach the client to use reaction formation for behavior control. C - Recommend the client attend assertiveness training. D - Encourage the client to increase socialization.

A - Establish and explain consequences for the client's behavior.

A nurse is caring for a client who has had a bilateral mastectomy. The client states, "I client states, "I can't even look at my scars." Which of the following actions should the nurse take? A - Establish eye contact with the client. B - Use a closed position when listening to the client. C - Inform the client that most people feel this way after this procedure. D - Reassure the client that everything will be okay now that the surgery is over.

A - Establish eye contact with the client. Rational A - The nurse should establish eye contact with the client when listening. This promotes a therapeutic environment and conveys to the client that the nurse is interested and actively listening to what the client is saying. B - The nurse should use an open position with their body when listening to the client to promote a therapeutic environment and convey that the nurse is interested and actively listening to what the client is saying. The nurse should be facing the client with arms at the sides and legs uncrossed. C - This response is nontherapeutic because the nurse is providing false reassurance to the client. This can minimize the client's feelings and can block the line of communication between the nurse and client. D - This response is nontherapeutic because the nurse is providing false reassurance to the client. This can minimize the client's feelings and can block the line of communication between the nurse and client.

A nurse on a pediatric unit is teamed with an assistive personnel (AP) for the upcoming shift. Which of the following assignments is within the range of function for the AP? A - Feeding formula to an infant B - Evaluating an adolescent's understanding of dietary needs C - Instructing a school-age child how to use crutches D - Interpreting a urine report of a toddler

A - Feeding formula to an infant Rational A - When determining who can perform a certain task, the nurse should assign clients based on the legal scope of practice of available personnel. Formula-feeding an infant is an appropriate task for the nurse to delegate to the AP. B - When determining who can perform a certain task, the nurse should assign clients based on the legal scope of practice of available personnel. Evaluation is part of the nursing process and requires professional education. It is not an appropriate task to delegate to an AP. C - When determining who can perform a certain task, the nurse should assign clients based on the legal scope of practice of available personnel. Providing instruction requires assessment of the client's ability and readiness to learn. It is not an appropriate task to delegate to an AP. D - When determining who can perform a certain task, the nurse should assign clients based on the legal scope of practice of available personnel. Interpreting data is part of the nursing process and requires professional education. It is not an appropriate task to delegate to an AP.

A nurse is contributing to the plan of care for a client who has paranoid personality disorder. Which of the following interventions should the nurse recommend? A - Give the client a written daily schedule each day. B - Mix prescribed medications in the client's food. C - Use therapeutic touch during episodes of client agitation. D - Assign a new nurse each day to the client's care.

A - Give the client a written daily schedule each day. Rational A - The nurse should recommend providing the client with a written daily schedule of planned care and activities for each day. A written schedule can decrease paranoia by informing the client of what to expect each day at specific times. The nurse should avoid changes to the planned schedule whenever possible and should communicate any necessary changes to the client. This communication between the nurse and client promotes trust and works to decrease the client's anxiety. B - Clients who have paranoid personality disorder are distrustful of others and might suspect that the nurse is trying to hide medication by mixing it in food. This paranoia can result in the client's refusal to eat. The nurse should promote honesty and trust in the nurse-client relationship by providing the client with medications that are not mixed in food along with an explanation of the benefits of the medication. Furthermore, mixing the medication in the client's food infringes on the client's right to refuse medication. The client can refuse medication except in the case that the medication is court ordered or it prevents the client from causing self-harm or harm to others. C - Clients who have paranoid personality disorder are distrustful of others and might respond to touch with increased anxiety and agitation. The nurse should avoid touching the client when the client is agitated. The nurse should provide adequate personal space and use therapeutic communication to calm the client during episodes of agitation. D - Clients who have paranoid personality disorder are distrustful of others and need consistency in nursing staff. The nurse should recommend assigning the same nurses to the client's care each day to promote trust and decrease client anxiety.

A nurse is preparing to administer a prescribed medication to a client via IV piggyback. Which of the following actions should the nurse take? A - Hang the piggyback medication bag above the level of the primary fluid bag. B - Cleanse the needleless port on the primary infusion line with 0.9% sodium chloride. C - Adjust the primary infusion rate to 30 mL/hr. D - Connect the piggyback IV line to the port closest to the IV site.

A - Hang the piggyback medication bag above the level of the primary fluid bag. Rational A - The nurse should hang the piggyback medication bag above the level of the primary fluid bag to allow the medication to infuse. B - The nurse should cleanse the needleless port on the primary infusion line with alcohol, rather than 0.9% sodium chloride. C - The nurse should identify that the primary infusion should not continue while the piggyback medication is infusing. D - The nurse should connect the piggyback IV line to the upper port, rather than the port closest to the IV site.

A nurse is collecting data for a school-age child who has sickle cell and is experiencing a vasoocclusive crisis. Which of the following manifestations should the nurse expect? A - Hematuria B - Hemoglobin 10 g/dL C - Reports of tingling in fingers D - Total bilirubin 1.0 mg/dL

A - Hematuria Rational A - Hematuria is a manifestation of a vasoocclusive crisis because of kidney ischemia. B - Children who have chronic sickle cell anemia have a hemoglobin level of 6 to 9 g/dL. C - Children who have a vasoocclusive crisis can have painful swelling of the hands and feet. D - A total bilirubin of 1.0 mg/dL is within the expected reference range of 0.3 to 1.0 mg/dL. Children who are experiencing a vasoocclusive crisis can have an elevated bilirubin level because of hepatomegaly or cirrhosis of the liver.

A nurse is assisting with the care of an infant who has spina bifida and recently had a ventriculoperitoneal shunt placed for hydrocephalus. Which of the following findings should the nurse identify as an indication of increased intracranial pressure? A - High-pitched cry B - Ataxia C - Depressed anterior fontanel D - Nuchal rigidity

A - High-pitched cry

A nurse is collecting data about a 4-year-old preschooler's gross motor skill. The nurse should expect the preschooler to be able to perform which of the following activities? A - Hopping on one foot B - Skipping on alternate feet C - Jumping rope D - Roller skating

A - Hopping on one foot

A nurse is preparing to auscultate the bowel sounds of a client who has a mechanical bowel obstruction in the descending colon. When listening in the left upper quadrant, the nurse should identify this sound as which of the following (audio clip) A - Hyperactive bowel sounds B - Friction rub C - Normal bowel sounds D - Abdominal bruit

A - Hyperactive bowel sounds Rational A - A mechanical bowel obstruction prevents a portion or all of the bowel contents from moving forward through the bowel. The nurse should expect to auscultate high-pitched, hyperactive bowel sounds above the point of the intestinal obstruction as the intestines attempt to propel the blockage forward. B - The nurse should expect to auscultate a pericardial friction rub, a high-pitched scratchy sound over the heart, for a client who has pericarditis. C - When auscultating normal bowel sounds, the nurse should expect to hear 5 to 35 gurgles and clicks in 1 min. D - When auscultating an abdominal bruit, the nurse should expect to hear a whooshing sound that indicates impaired blood flow through an artery.

A nurse in an urgent care clinic is collecting data from a client whose friend reports a suspicion of cocaine use. The nurse should identify that which of the following manifestations is an indicator of the client's use of this substance? A - Hypertension B - Drowsiness C - Bradycardia D - Constricted pupils

A - Hypertension

A nurse is reviewing the medical record of a female adolescent client who has primary amenorrhea. Which of the following findings should the nurse identify as a risk factor for this disorder? Select all A - Hypothyroidism B - Obesity C - Cannabis use D - Oral contraceptive use E - Emotional stress

A - Hypothyroidism C - Cannabis use D - Oral contraceptive use E - Emotional stress

A nurse is reinforcing teaching about management of constipation with a client who has hypothyroidism. Which of the following should the nurse include in the teaching? A - Increase intake of fiber-rich foods. B - Take a laxative every morning. C - Maintain a fluid intake of 1200 mL per day. D - Limit activity to preserve energy.

A - Increase intake of fiber-rich foods.

A nurse is reviewing the laboratory results for a client who has been receiving epoetin alpha for chronic kidney failure. The nurse should identify which of the following findings as a therapeutic effect of the medication? A - Increased hemoglobin B - Increased potassium C - Decreased calcium level D - Decreased WBC count

A - Increased hemoglobin Rational A - Epoetin alpha stimulates red blood cell production. Loss of kidney function can lead to decreased production of erythropoietin and cause anemia. The goal of treatment is a hemoglobin level of 10 to 11 g/dL. Hbg for clients receiving epoetin should not exceed 12 g/dL due to an increased risk of adverse effects. The lowest possible dose should be used. B - An increase in potassium level is a manifestation of chronic kidney failure and is not an indication that epoetin alpha has been effective. C - A decreased calcium level is a manifestation of chronic kidney failure and is not an indication that epoetin alpha has been effective. D - A decreased WBC count is a manifestation of autoimmune diseases or a viral infection and is not an indication that epoetin alpha has been effective.

A nurse observes an assistive personnel (AP) taking a picture of a client who has not given consent. The nurse should identify the AP has committed which of the following torts? A - Invasion of privacy B - Negligence C - Defamation of character D - Battery

A - Invasion of privacy

A nurse is reinforcing teaching about adverse effects with a client who has a subdermal implant for contraception. Which of the following potential adverse effects should the nurse include in the teaching? A - Irregular menstrual bleeding B - Anxiety C - Bone density loss D - Weight loss

A - Irregular menstrual bleeding Rational A - The nurse should inform the client that subdermal implants for contraception are effective for up to 3 years and contain progestin hormone, which prevents ovulation and thickens the cervical mucus. The most common adverse effect is irregular menstrual bleeding. Other adverse effects include nausea, vertigo, and headaches. B - Depression, rather than anxiety, is a potential adverse effect of a subdermal implant for contraception. C - Bone density loss is an adverse effect of injectable progestins, such as medroxyprogesterone acetate. A subdermal implant for contraception has no effect on bone density. D - Weight gain, rather than weight loss, is a potential adverse effect of a subdermal implant for contraception.

A nurse is caring for a client who is in Buck's traction. Which of the following interventions should the nurse perform to reduce skin breakdown? A - Keep the skin dry and free of perspiration. B - Use hot water and antibacterial soap to bathe the client. C - Massage the skin over bony prominences to promote circulation. D - Limit the use of moisturizers on the skin over bony prominences.

A - Keep the skin dry and free of perspiration.

A nurse is caring for a client who is at 11 weeks of gestation and reports frequent vomiting. Which of the following findings should the nurse identify as an indication that the client has hyperemesis gravidarum? A - Ketonuria B - Bradycardia C -Bradypnea D - Proteinuria

A - Ketonuria

A nurse is reviewing the medication administration record for a client who is receiving nifedipine for gestational hypertension. The nurse should identify that which of the following medications is contraindicated for use with nifedipine? A - Magnesium sulfate B - Acetaminophen C - Promethazine D - Oxytocin

A - Magnesium sulfate

A nurse is caring for an older adult client who has a reddened area over the sacrum. Which of the following actions should the nurse take? A - Minimize the time the head of the bed is elevated. B - Apply a sterile gauze dressing to the site. C - Massage the site with moisturizing lotion. D - Place a donut-shaped cushion under the client's sacral area.

A - Minimize the time the head of the bed is elevated.

A nurse in a oncology clinic is reinforcing teaching about Mohs surgery with a client who has skin cancer. Which of the of following information should the nurse include in the teaching? A - Mohs surgery is a horizontal shaving of thin layers of the tumor. B - Mohs surgery uses liquid nitrogen to destroy the cancerous tissue. C - Mohs surgery is the preferred treatment for melanoma skin cancer. D - Mohs surgery is a palliative treatment for metastatic skin cancer.

A - Mohs surgery is a horizontal shaving of thin layers of the tumor. Rational A - Mohs surgery is performed to treat basal and squamous cell carcinoma. The procedure, which involves a horizontal shaving of thin layers of a tumor, has a high treatment rate. B - Cryosurgery, rather than Mohs surgery, uses liquid nitrogen to destroy cancerous tissue. C - Mohs surgery is the preferred treatment for basal and squamous cell carcinoma. The preferred treatment for melanoma is a wide, full thickness surgical excision. D - Radiation, rather than Mohs surgery, can be used as a palliative treatment for metastatic skin cancer.

A nurse is assisting with the care of a client who is in active labor and requests hydrotherapy in the whirlpool tub. Which of the following actions should the nurse take? A - Monitor the fetal heart rate by Doppler. B - Assist with the placement of an internal scalp electrode. C - Remind the client that therapy is not recommended during this stage of labor. D - Verify membranes are ruptured.

A - Monitor the fetal heart rate by Doppler. Rational A - A Doppler, fetoscope, or wireless external monitoring device is used to monitor the fetal heart rate during hydrotherapy. B - The use of an internal scalp electrode is contraindicated during hydrotherapy in a whirlpool tub because of the force of the water pressure coming from the jets and the risk for infection. C - Hydrotherapy is an effective nonpharmacological intervention and is recommended for use during labor. D - Rupture of membranes is not required to implement hydrotherapy in a whirlpool tub. Studies show no increased risk of infection if therapy is used and membranes have been ruptured.

A nurse is assisting with the care of a client who has a cardiac catheterization via the right femoral artery. Which of the following actions should the nurse take to prevent post procedure complication? select all A - Monitor the insertion site for bleeding. B - Position the affected extremity at a 45° angle. C - Restrict the client's fluid intake. D - Maintain the pressure dressing. E - Check the client's peripheral pulses.

A - Monitor the insertion site for bleeding. D - Maintain the pressure dressing. E - Check the client's peripheral pulses.

A nurse in an outpatient clinic is reviewing the medical record of a school-age child who has a negative rapid strep test 2 days ago. The nurse notes a positive throat culture result for group A streptococcus b-hemolytic. Which of the following actions is the nurse's priority? A - Notify the guardian of the need to start antibiotic therapy. B - Instruct the guardian to replace the child's toothbrush. C - Tell the guardian to have the child gargle with warm saline several times daily. D - Instruct the guardian to increase the child's daily fluid intake.

A - Notify the guardian of the need to start antibiotic therapy. Rational A - The greatest risk to this client is injury due to complications from a streptococcal infection, including rheumatic fever and glomerulonephritis. Therefore, the priority action is to initiate antibiotic therapy. The nurse should also reinforce the importance of completing the entire course of antibiotic therapy, even if manifestations subside. The child is no longer infectious to others 24 hr after initiating antibiotic therapy. Manifestations of group A streptococcus ß-hemolytic infection include swollen and red tonsils, exudate on the pharynx, nasal discharge, fever, arthralgia, and an enlarged local lymph node. B - The nurse should instruct the guardian to replace the child's toothbrush after receiving antibiotic therapy for 24 hr to prevent reinfection; however, this is not the priority action. C - The nurse should instruct the guardian to have the child gargle several times per day with warm saline to decrease the discomfort caused by a throat infection; however, this is not the priority action. Acetaminophen and NSAIDs are effective at relieving pain or inflammation. D - The nurse should instruct the guardian to increase the child's daily fluid intake to reduce the risk for dehydration and to prevent irritation to the tonsils; however, this is not the priority action.

A nurse is monitoring a client who is receiving IV fluids via an infusion pump and notes the pump is malfunctioning. Which of the following actions should the nurse take? A - Place a tag on the IV pump. B - Report the malfunctioning IV pump to the risk manager. C - Calculate the manual IV drip rate. D - Call housekeeping to pick up the IV pump.

A - Place a tag on the IV pump.

A nurse is assisting with the administration of a nasogastric enteral feeding for an infant. Which of the following actions should the nurse take? A - Place the infant in semi-Fowler's position for 1 hr after the feeding. B - Flush the tube with 30 mL of normal saline before the feeding. C - Warm the feeding in the microwave immediately prior to administration. D - Auscultate over the infant's epigastric area to ensure proper tube placement.

A - Place the infant in semi-Fowler's position for 1 hr after the feeding.

A nurse is caring for a client who has manifestations of tuberculosis and a prescription for sputum specimen collection to test for acid-fast bacillus (AFB). Which of the following actions should the nurse plan to take? A - Obtain a sputum specimen on 3 consecutive days. B - Schedule collection of the sputum specimen just before bedtime. C - Ensure the collected specimen is kept at room temperature. D - Collect the specimen 30 min before performing postural drainage.

A - Obtain a sputum specimen on 3 consecutive days. Rational A - The nurse should plan to obtain an early-morning sputum specimen on 3 consecutive days when testing for AFB. When obtaining a sputum specimen, the nurse should provide oral care to decrease the risk of specimen contamination. B - The nurse should plan to collect the sputum specimen early in the morning when the client awakens. It is often easier for the client to expectorate a specimen at this time of day due to the accumulation of sputum while sleeping. C - The nurse should immediately transport the sputum specimen to the laboratory or place it in a refrigerator until transport. This action decreases the risk of a false positive due to growth of organisms within the specimen. D - The nurse can perform postural drainage to assist the client in expectorating a sputum specimen. If postural drainage is needed, it is performed prior to specimen collection. Postural drainage improves the client's oxygenation by improving drainage and expectoration of secretions through appropriate positioning and turning of the client.

A nurse i s organizing care for a group of clients. According to Maslow's hierarchy of needs, which of the following interventions should the nurse plan to perform first? A - Offer finger foods to a client who is in the manic phase of bipolar disorder B - Document the effect of a client who had light therapy for seasonal affective disorder 2 days ago. C - Assist a client who has a depressive disorder with decision making regarding group activities D - REinforce teaching about a new prescription of clozapine with a client who has schizophrenia.

A - Offer finger foods to a client who is in the manic phase of bipolar disorder

A nurse is reinforcing teaching with a client who has heart failure and a new prescription for hydrochlorothiazide. Which of the following findings should the nurse instruct the client to report to the provider? A - Onset of nausea B - Increased urinary output C - Weight loss of 0.9 kg (2 lb) per week D - Missed dose of the medication

A - Onset of nausea Rational A - The nurse should instruct the client to report a new onset of nausea, which can be an indication of hyponatremia or hypokalemia resulting from the diuretic effects of the hydrochlorothiazide. B - The nurse should remind the client that an increase in urinary output is a desired effect of hydrochlorothiazide. C - The nurse should remind the client to report weight gain of 0.9 kg (2 lb) or more per week to the provider. D - The nurse should instruct the client to take a missed dose of the medication as soon as the client remembers. However, the client should not take a double-dose of the medication.

A nurse is reinforcing teaching with the parent of a child who has a new prescription for ferrous sulfate. The nurse should reinforce that the parent should administer the medication with which of the following fluids to enhance medication absorption? A - Orange juice B - Water C - Milk D - Unsweet tea

A - Orange juice

A nurse is assisting in planning an inservice about time-management strategies with a group of newly licensed nurses. Which of the following information should the nurse plan to include? A - Organize client care tasks based on data from change-of-shift report. B - Perform simple tasks before performing more complex tasks. C - Fulfill client requests as soon as they are made. D - Plan to multitask several client care activities.

A - Organize client care tasks based on data from change-of-shift report. Rational A - The nurse should use data from change-of-shift report to help determine priorities of care. The nurse should delegate tasks to assistive personnel to allow more time for tasks that cannot be delegated. B - The nurse should prioritize tasks according to the needs of the clients. C - The nurse should use client goals to guide care delivery. Meeting requests as they are made can waste the nurse's time and can prevent the nurse from meeting needs that relate to risks or urgent issues. D - The nurse should plan to complete one client care activity at a time. Multitasking divides the nurse's attention and increases the risk for errors.

A nurse is discussing contraceptive options with a client who has expressed interest in depot medroxyprogesterone. The nurse should reinforce with the client that they can expect which of the following outcomes from this medication? A - Ovulation suppression B - Contraceptive protection for 12 months with each dose C - Spermicidal environment in the uterus D - Endometrial involution

A - Ovulation suppression Rational A - Depot medroxyprogesterone blocks the secretion of gonadotropins, suppressing ovulation. It also produces thick cervical mucus that inhibits sperm motility and causes thinning of the endometrium, making implantation unlikely. B - Each dose of depot medroxyprogesterone offers protection against pregnancy for 3 months. The client will need to return to the provider quarterly for another IM injection of the medication. C - Intrauterine devices (IUDs) create a harmless inflammatory response that has a spermicidal effect. An IUD does not prevent ovulation, unlike depot medroxyprogesterone. D - Subdermal etonogestrel implants produce thick mucus and involution of the endometrium. This prevents implantation of fertilized eggs on the uterine wall.

A nurse on a quality improvement team is assisting in the development of a step-by-step guide for interpreting ECG rhythms. Which of the following parts of the cardiac cycle should the nurse recommend to include for determination of the rate of atrial contractions? A - P waves B - P-R intervals C - S-T segments D - T waves

A - P waves Rational A - The P wave shows depolarization of the atria, which occurs with atrial contraction. The nurse should measure the distance between sequential P waves to determine rhythm regularity, and count the number of P waves to determine the atrial rate. B - The P-R interval represents the length of time it takes for electricity to travel across the atria. This interval should be 0.12 to 0.20 seconds. A number greater than the expected reference range indicates a client is experiencing first-degree heart block. C - The S-T segment measures the time from the end of ventricular contraction until the ventricle is repolarized and ready to contract again. Elevation or depression of the S-T segment shape can indicate myocardial infarction. D - The T wave measures the time required for the ventricle to repolarize. Potassium levels and myocardial injury can alter the shape of T waves.

A nurse is caring for a client who has a compound fracture of the femur and was placed in balanced suspension skeletal traction 4 days ago. Which of the following actions should the nurse take? A - Perform pin site care daily. B - Remove the overbed trapeze. C - Remove the boot every 2 hr. D - Keep the weights on a stable, flat surface.

A - Perform pin site care daily.

A nurse is caring for a client who is receiving internal radiation therapy and discovers a radioactive pellet on the client's bed. Which of the following actions should the nurse take? A - Place the pellet in a lead container. B - Remove the pellet with a gloved hand. C - Contact environmental services. D - Notify the infection control officer.

A - Place the pellet in a lead container. Rational A - The nurse should use tongs and place the pellet in a lead container due to radioactive elements the pellet contains. This action protects the nurse and other clients from radiation exposure. When caring for a client receiving internal radiation, the nurse should limit time spent in the room and maintain distance from the client when possible. The nurse should also wear a lead apron and a dosimeter badge during client care. B - The pellets are radioactive and require specific safety measures for removal. The nurse should use tongs, rather than gloves, to remove the pellet and place it in a lead container. C - The nurse should contact the radiology department to replace or dispose of the radioactive pellet. There is no need to contact environmental services. For a client receiving internal radiation therapy, the nurse should ensure a sign is on the door warning other staff of possible exposure to radiation when in the room. D - There is no need for the nurse to notify the infection control department. The radioactive pellet is not a source of infectious disease. However, the nurse should use special precautions to dispose of client secretions and when handling dressings and linens that contain secretions, because they are radioactive.

A nurse is caring for a client who is at risk for developing pressure ulcers. Which of the following actions should the nurse take? A - Position pillows between the bony prominences. B - Check for incontinence every 3 hr. C - Massage reddened areas of the skin. D - Elevate the head of the bed to 45°.

A - Position pillows between the bony prominences.

A nurse is contributing to the plan of care for a child who has sickle cell anemia and is experiencing a vaso-occlusive crisis. Which of the following is the priority intervention for the nurse to recommend to include in the plan? A - Promote oxygen utilization. B - Administer antibiotics. C - Encourage fluid intake. D - Apply a warm compress to the joints.

A - Promote oxygen utilization.

A nurse on a postpartum unit is assisting with the care of a client who has a hypotonic uterus and excessive vaginal bleeding. Which of the following actions should the nurse take first? A - Provide fundal massage for the client. B - Insert an indwelling urinary catheter for the client. C - Administer methylergonovine IM to the client. D - Administer oxygen via nonrebreather face mask to the client.

A - Provide fundal massage for the client. Rational A - The nurse should identify that the greatest risk to this client is postpartum hemorrhage. Therefore, the first action the nurse should take is to provide fundal massage to increase uterine muscle tone and express blood clots from the uterus, which will decrease bleeding. B - Inserting an indwelling urinary catheter is important to eliminate bladder distention and monitor urinary output. However, this is not the first action the nurse should take. C - Administering methylergonovine to enhance uterine contractions is an action the nurse should take to manage postpartum hemorrhage. However, this is not the first action the nurse should take. D - Administering oxygen via nonrebreather face mask is an action the nurse should take to enhance oxygenation to the cells. However, this is not the first action the nurse should take.

A nurse is assisting in the plan of care for a client who may have meningitis and is scheduled for a lumbar puncture. Which of the following actions should the nurse plan to include following the procedure? Select all A - Provide pain medication as needed. B - Administer a sedative. C - Maintain lateral recumbent position. D - Check level of consciousness. E - Check sensation in the toes.

A - Provide pain medication as needed. D - Check level of consciousness. E - Check sensation in the toes. Rational A - Provide pain medication as needed is correct. Following a lumbar puncture procedure, the client might experience moderate pain at the puncture site and headache. Prescribed analgesia medication, such as acetaminophen, manages the pain effectively. B - Administer a sedative is incorrect. Sedation can inhibit accurate examination of the client's neurological status. The nurse should monitor for changes in vital signs, level of consciousness, headache, and nuchal rigidity, as well as drainage, redness, or swelling at the puncture site. C - Maintain lateral recumbent position is incorrect. Following a lumbar puncture, the client should be kept flat and still for 4 to 24 hr as prescribed by the provider to decrease leakage of cerebrospinal fluid (CSF) from the lumbar puncture site. The client should maintain the lateral recumbent position during the procedure to open the spaces between the vertebrae. The nurse can assist the client to hold this position. D - Check level of consciousness is correct. A change in the client's level of consciousness can indicate meningitis or a loss of CSF. Additional manifestations of meningitis include headache, increased temperature, infection at the lumbar puncture site, or nuchal rigidity. E - Check sensation in the toes is correct. A lumbar puncture can cause injury to the spinal cord. The nurse should monitor the client's sensation, position, and movement in both lower extremities. The nurse should modify the neurological exam to maintain the client in a flat, still position and report any neurologic deficits.

A nurse is contributing to the plan of care for a client who has a halo stabilization device. Which of the following interventions should the nurse include in the plan of care? A - Provide pin site care every shift. B - Turn the client by holding the halo device. C - Remove the jacket to inspect the client's skin integrity. D - Allow three fingers to slide easily between the client and the jacket.

A - Provide pin site care every shift. Rational A - The nurse should provide pin site care, which includes cleaning and inspection for manifestations of infection, every shift, or more often as indicated by condition or facility protocol. B - The nurse should avoid using the halo device to reposition the client. Pulling on the device can alter the stabilization of the client's cervical vertebrae. C - The nurse should not remove the halo jacket to inspect the client's skin because this can alter the stabilization of the cervical vertebrae. When providing skin care, the nurse should loosen the side of the vest, then wash and thoroughly dry the client's skin. D - The nurse should ensure the jacket fits snugly, so that only one finger can slide easily between the client and the jacket, to stabilize the cervical vertebrae without causing injury to the client's skin.

A nurse is preparing to delegate assignments after receiving change-of-shift report. Which of the following tasks should the nurse assign to an assistive personnel (AP) A - Provide postmortem care. B - Insert a nasogastric tube. C - Obtain a specimen for a wound culture. D - Instruct a client on the use of an incentive spirometer.

A - Provide postmortem care.

A charge nurse is demonstrating actions to take when using a fire extinguisher to contain a small first to a group of staff. Which of the following actions should the nurse take? A - Pull the pin out of the extinguisher. B - Rotate the handles a quarter turn until they click into place. C - Aim the hose of the extinguisher at the top of the fire. D - Keep the hose steady while dousing the central portion of the fire.

A - Pull the pin out of the extinguisher. Rational A - When in place, the pin locks the handles. The nurse must remove the pin to use the extinguisher. B - The nurse should squeeze the handles together to discharge the extinguishing material onto the fire. C - The nurse should aim the hose at the base of the fire. D - The nurse should sweep the hose from side-to-side while attempting to douse the fire.

A nurse is contributing to the plan of care for a preschooler who is hospitalized and has the measles. Which of the following activities should the nurse recommend for the child? A - Putting together a puzzle with large pieces B - Playing a video game in the playroom C - Pulling a wagon with toys in the hallway D - Constructing a model airplane

A - Putting together a puzzle with large pieces Rational A - After initiating airborne precautions for a preschooler, the nurse should recommend putting together a puzzle with large pieces. Other recommended activities for preschoolers confined to a small room include playing dress up, role playing, finger painting, and looking at picture books. B - Preschoolers who are on airborne precautions should not be allowed in the playroom with other children, because this would increase the risk of spreading the infection. C - Preschoolers who are on airborne precautions should be confined to their room unless they are going for testing in another area of the facility. Pulling a wagon in the hallway can spread the infectious disease by dispersing particles in the air containing infectious agents, exposing them to others. The child should wear a mask when leaving the room for any reason. D - Constructing a model airplane requires an extended attention span, fine motor skills, and the ability to follow detailed directions. This activity is more appropriate for a much older child.

A nurse is caring for a client who has bipolar disorder and is experiencing a manic episode. Which of the following interventions should the nurse take first? A - Remove harmful objects from the client's room. B - Decrease the client's environmental stimuli. C - Administer an antipsychotic medication to the client. D - Provide physical activities for the client.

A - Remove harmful objects from the client's room.

A nurse is measuring the length and weight of a 6-month-old infant during a well-baby visit. Which of the following actions should the nurse take? select all A - Remove the infant's diaper before weighing. B - Use a stadiometer to measure the infant. C - Place a disposable covering on the scale before weighing. D - Measure the infant from the forehead to the heel. E - Zero the scale prior to use.

A - Remove the infant's diaper before weighing. C - Place a disposable covering on the scale before weighing. E - Zero the scale prior to use. Rational A - Remove the infant's diaper before weighing is correct. The nurse should weigh the infant without a diaper or clothing to ensure an accurate weight is measured. B - Use a stadiometer to measure the infant is incorrect. The nurse should not use a stadiometer to measure the infant. A stadiometer is a wall-mounted unit for measuring the height of individuals who can stand independently. The nurse should position the infant recumbent and measure him using a secured flat measuring board. The measuring board often has a stationary headrest and an adjustable footboard to enhance measurement accuracy. C - Place a disposable covering on the scale before weighing is correct. The nurse should place a disposable covering on the scale before weighing the infant. Infection control measures are essential when placing an infant on a scale used by numerous clients. A thin, disposable cover promotes safety and minimizes possible transmission of pathogens. D - Measure the infant from the forehead to the heel is incorrect. To obtain an accurate measurement, the nurse should measure the infant from the crown of the head to the heels of the feet. E - Zero the scale prior to use is correct. The nurse should zero the scale prior to use. If a precise measurement is needed, two nurses should weigh the infant on separate occasions.

A nurse is reinforcing discharge teaching with a client who has hearing loss. Which of the following actions should the nurse take when communicating with the client? A - Rephrase client instructions when not understood. B - Cup hands around the mouth and direct speech toward the client. C - Accentuate vowel sounds by using a higher pitch when speaking. D - Sit to the side of the client and speak instructions into her best ear.

A - Rephrase client instructions when not understood.

A nurse is caring for a school-age child who has skeletal traction applied to the right lower leg to repair a femur fracture. Which of the following findings is the priority for the nurse to report to the provider? A - Report of tingling in the right foot B - Pain rating of 7 on a scale of 0 to 10 C - Decrease in food intake D - Increase in crusting at pin sites

A - Report of tingling in the right foot

A nurse is collecting data from a client who is receiving gentamicin. Which of the following findings is an adverse effect of gentamicin for which the nurse should withhold the medication? A - Report of tinnitus B - Ecchymosis C - Report of blurred vision D - Stomatitis

A - Report of tinnitus Rational A - Gentamicin is an aminoglycoside antibiotic that can cause ototoxicity. The nurse should monitor the client for early manifestations of ototoxicity such as tinnitus and hearing loss. B - Gentamicin is an aminoglycoside antibiotic that can cause nephrotoxicity; however, it does not cause bleeding or bruising. The nurse should monitor the client's weight, BUN, and creatinine levels for manifestations of nephrotoxicity. C - Gentamicin is an aminoglycoside antibiotic that can cause vestibular dysfunction, not blurred vision. The nurse should monitor the client for manifestations of vestibular damage, such as ataxia, vertigo, nausea, and vomiting. D - Gentamicin is an aminoglycoside antibiotic that can cause vestibular damage, not stomatitis. The nurse should monitor the client for manifestations of vestibular damage, such as headache.

A nurse is caring for a client in a day treatment program. Which of the following actions should the nurse take (exhibit) A - Request transport for the client to an emergency department. B - Place a hypothermia blanket on the client. C - Discontinue the client's fluoxetine therapy immediately. D - Implement droplet precautions for the client.

A - Request transport for the client to an emergency department. A - The nurse should request transport for the client to the nearest emergency department because the client has manifestations of serotonin syndrome. Serotonin syndrome is a life-threatening syndrome and is caused by an over activation of the central serotonin receptors. This is related to interactions with taking an SSRI and trazodone along with St. John's wort. Manifestations of serotonin syndrome include hypertension, tachycardia, vomiting, abdominal pain, and mental status changes b- There is no indication that a hypothermia blanket is necessary for this client. Hypothermia blankets are used for clients who have excessively high body temperatures, such as a client who has experienced a heat stroke. C -Fluoxetine should not be discontinued abruptly because this can cause the client to exhibit manifestations of withdrawal. D - There is no indication for the implementation of droplet precautions for this client. Droplet precautions are used for clients who have diseases that are transmitted by large droplets that are expelled into the air.

A nurse is caring for a client who experienced angioedema in response to newly prescribed losartan. Which of the following findings indicates a therapeutic response to treatment of the angioedema? A - Respirations are unlabored. B - Client reports decreased groin pain. C - Blood pressure is decreased. D - Client tolerates prescribed dose of losartan with no peripheral edema.

A - Respirations are unlabored. Rational A - Losartan is an angiotensin receptor blocker (ARB). Both ARBs and angiotensin converting enzyme (ACE) inhibitors have the adverse effect of angioedema. Manifestations of angioedema include edema of the tongue, glottis, and pharynx. This results in limitation or blockage of the airway. Angioedema causes the capillaries to become more permeable, resulting in fluid shifting into the subcutaneous tissues. Untreated angioedema can result in death. B - Although edema can occur in any area, the groin is not affected specifically by this disorder. Angioplasty and angiograms most often use the femoral vessels, but the prefix "angio" is a general term for blood vessel rather than a reference to the femoral area. C - An anaphylactic reaction results in tachycardia, decreased pulses, and hypotension due to vasodilation. A decreased blood pressure level would not indicate a therapeutic response to treatment. D - Peripheral edema is not an expected adverse effect of angioedema. The edema that is significant in this client occurs in the lips, mouth, and throat, causing airway obstruction. Once the client has this response, the client should never again take this medication.

A nurse on a facility's performance improvement team is assisting to develop practice guidelines for performing bladder scans. Which of the following actions should the nurse take prior to developing a policy and procedure for this task? A - Review evidence-based practice data related to bladder scanner use. B - Compare the cost of indwelling urinary catheters with that of a bladder scanner. C - Conduct a chart audit to determine previous outcome trends in bladder scanner use. D - Gather a consensus of provider opinions about the use of bladder scanners at the facility.

A - Review evidence-based practice data related to bladder scanner use.

A nurse is preparing to reinforce discharge teaching with a client who does not speak the same language as the nurse. Which of the following actions should the nurse plan to take? Select all A - Select an interpreter who is the same gender as the client. B - Ask the client's family members to interpret the information. C - Ensure interpreters provided by the facility have knowledge of medical terminology. D - Obtain informed consent from the client prior to requesting an interpreter. E - Choose an interpreter from the same ethnic background as the client.

A - Select an interpreter who is the same gender as the client. C - Ensure interpreters provided by the facility have knowledge of medical terminology. E - Choose an interpreter from the same ethnic background as the client.

A nurse is collecting data from a client who is receiving erythromycin to treat primary atypical pneumonia. Which of the following findings should the nurse report to the provider as an adverse effect of the medication? A - Severe diarrhea B - Photosensitivity C - Stomatitis D - Muscle cramps

A - Severe diarrhea Rational A - The nurse should identify that Clostridium difficile-associated diarrhea is a potential complication of erythromycin. Manifestations include severe diarrhea, fever, and blood or pus in the stool. Therefore, the nurse should report this finding to the provider. B - Photosensitivity is not an adverse effect of erythromycin. C - Stomatitis is not an adverse effect of erythromycin D - Muscle cramps are not an adverse effect of erythromycin.

A nurse in a long-term care facility is assisting with an in-service about legal torts. The nurse should explain that communicating false information that damages the client's reputation is an example of which of the following types of torts. A - Slander B - Battery C - Malpractice D - Assault

A - Slander Rational A - Slander occurs when a nurse uses false oral communication about a client that is harmful to the client's reputation. Libel refers to written or print communication that defames the client's character. B - Battery occurs when a nurse touches a client without consent. A nurse performing a procedure on a client without consent is an example of battery. C - Malpractice occurs when a nurse commits professional negligence. A nurse failing to check blood prior to a transfusion and administering the incorrect type of blood to a client is an example of malpractice. D - Assault occurs when a nurse intentionally threatens a client. A nurse threatening to restrain a client who is not participating with the plan of care is an example of assault.

A nurse is assisting with discharge planning for a client who needs a say treatment center and has limited community and financial support. Which of the following referrals should the nurse recommend for inclusion in the client's discharge plan? A - Social worker B - Recreational therapist C - Psychologist D - Occupational therapist

A - Social worker

A nurse is reinforcing teaching with the parents of a 7-year-old female child about behavioral expectations. Which of the following behaviors should the nurse include in the teaching? A - Spends a lot of time by herself B - Exhibits a decline in self-esteem C - Selectively chooses a best friend D - Shows a competitive nature with others

A - Spends a lot of time by herself

A nurse is assisting with the medication reconciliation of a client who has a new prescription for venlafaxine. The nurse should recognize that which of the following complementary therapies, when taken with venlafaxine, increases the client's risk for developing serotonin syndrome? A - St. John's wort B - Glucosamine C - Probiotics D - Feverfew

A - St. John's wort Rational A -St. John's wort, an herbal preparation, promotes the activation of serotonin receptors. When taken concurrently with venlafaxine, a serotonin/norepinephrine reuptake inhibitor, the client can develop serotonin syndrome, causing life-threatening manifestations. B - Glucosamine can be taken to treat osteoarthritis. It does not affect serotonin levels. However, glucosamine does interact with anticoagulants, increasing the client's risk for bleeding. C - Probiotics can be taken to treat irritable bowel disease, ulcerative colitis, and Clostridium difficile-associated diarrhea. It does not affect serotonin levels. However, antibacterial and antifungal medications can reduce the effectiveness of probiotics. D - Feverfew can be taken to prevent migraine headaches. It does not affect serotonin levels. However, feverfew does interact with anticoagulants, increasing the client's risk for bleeding.

A nurse is caring for a client who is preoperative and is receiving an IV infusion of cefazolin. Ten minutes after beginning the infusion, the client reports intense itching. Which of the following actions should the nurse take first? A - Stop the medication infusion. B - Notify the charge nurse. C - Administer a PRN dose of diphenhydramine. D - Follow facility policy for appropriate reporting of the adverse reaction.

A - Stop the medication infusion.

A nurse is caring for a group of clients on a medical-surgical unit. The nurse should take which of the following actions to protect client confidentiality? select all A - Store clients' charts in a secure location. B - Ensure that emails containing client health information are encrypted. C - Give a verbal report at change of shift outside the clients' rooms. D - Remove client information from fax machines after use. E - Dispose of written report sheets into the trash container.

A - Store clients' charts in a secure location. B - Ensure that emails containing client health information are encrypted. D - Remove client information from fax machines after use. Rational A - Store clients' charts in a secure location is correct. A client's medical record or chart is a legal document and contains information describing the care that is delivered to a client. Medical records are required to be stored in a locked, secured area in all departments within the facility. The client's current records on the unit should be kept in a secured location at the nurses' station or in cabinets out of reach of visitors or family members. B - Ensure that emails containing client health information are encrypted is correct. Emails that contain client information should be encrypted to protect client confidentiality. C - Give a verbal report at change of shift outside the clients' rooms is incorrect. Although the practice of a walking change-of-shift report is a common practice in many facilities, it can contribute to a breach in client confidentiality. Family members, visitors, or other hospital employees can overhear information related to a client's health care. D - Remove client information from fax machines after use is correct. It is imperative to remove client medical information immediately from fax and copy machines to prevent other people who are not involved in the client's care from viewing the information. Fax machines should be placed in areas that do not provide access to anyone other than health care providers. E - Dispose of written report sheets into the trash container is incorrect. Disposing of written report sheets into the trash container is inappropriate and can contribute to a breach in confidentiality. Anyone could retrieve the report sheet and view confidential health care information about clients. The nurse should shred the report sheet in a locked receptacle within the facility and then incinerate the contents of the receptacle.

A nurse is assisting with the discharge of a client who has a new permanent colostomy. The client expresses concern about learning to care for the appliance and obtaining supplies discreetly. Which of the following actions should the nurse take? select all A - Suggest that the client join an ostomy support group. B - Arrange a follow-up appointment with an enterostomal therapy nurse. C - Provide the client with the name and number of an ostomy supply delivery service. D -Recommend that the client's discharge be postponed until concerns are resolved. E - Request a social work referral for the client to discuss financial concerns.

A - Suggest that the client join an ostomy support group. B - Arrange a follow-up appointment with an enterostomal therapy nurse. C - Provide the client with the name and number of an ostomy supply delivery service. E - Request a social work referral for the client to discuss financial concerns.

A nurse is observing an assistive personnel (AP) providing care for a group of clients. For which of the following actions by the AP should the nurse intervene? A - The AP offers a client a drink of cool water 1 hr before a scheduled liver biopsy. B - The AP weighs a client who has heart failure before the client eats breakfast. C - The AP assists a client who is 36 hr postoperative from an above-the-knee amputation into the prone position. D - The AP provides a high-protein snack for a client who has AIDS.

A - The AP offers a client a drink of cool water 1 hr before a scheduled liver biopsy. Rational A- A client who is scheduled for a liver biopsy should have nothing to eat or drink for at least 4 hr before the procedure to reduce the risk for aspiration in case the client requires general anesthesia. Therefore, the nurse should intervene with the AP's action. B - A client who has heart failure is at risk for fluid volume overload. Weight is an accurate indication of fluid volume status. The AP should weigh the client at the same time each day, preferably before breakfast. C - To reduce the risk for a contracture, a client who is 24 hr postoperative from an above-the-knee amputation should lie in a prone position for 20 to 30 min every 3 to 4 hr. The AP can assist a client to change positions. D - A client who has AIDS has an increased metabolic need and requires an increase in protein and caloric intake. The AP can provide the client with prescribed nutritional meals and supplements.

A nurse in an outpatient clinic is caring for a client who has schizophrenia. For which of the following client actions should the nurse recommend transfer to an acute care facility? A - The client develops command hallucinations. B - The client displays transference toward the nurse. C - The client reveals a family history of schizophrenia. D - The client expresses feelings of low self-esteem.

A - The client develops command hallucinations.

A nurse is caring for a client who is postoperative following a below-the-knee amputation. Which of the following client actions indicates that the client is struggling to accept their body image change? A - The client looks away during dressing changes. B - The client asks how other people live without their legs. C - The client requests to wear their pants from home. D - The client reports pain to the affected extremity.

A - The client looks away during dressing changes. Rational A - Looking away from the amputation during dressing changes indicates to the nurse that the client might not be accepting their body image change. The nurse should encourage the client to look at the amputation and be involved in their wound care to promote acceptance of their body image change. B - Asking questions about how other people live without a leg should indicate to the nurse that the client is looking toward the future and accepting their body image change. C - The nurse should encourage the client to wear clothing from home to promote comfort and acceptance of the body image change. D - A report of pain is not an indication that the client has not accepted their body image change. Phantom limb pain can occur following an amputation.

A nurse is reinforcing teaching with a client who is at 26 weeks of gestation and is Rh-negative. The client is scheduled for an amniocentesis. Which of the following information should the nurse include in the teaching? A - The client will receive Rho(D) immune globulin immediately after the procedure. B - The client should not consume foods or liquids for 12 hr prior to the procedure. C - The client should not empty their bladder until after the procedure. D - The client should receive the hepatitis B vaccine at least 1 month prior to the procedure.

A - The client will receive Rho(D) immune globulin immediately after the procedure. Rational A - The nurse should reinforce that Rh-negative clients will receive Rho(D) immune globulin following an amniocentesis to prevent isoimmunization, which can occur as a result of fetomaternal hemorrhage. B - There is no indication to restrict intake prior to having an amniocentesis. C - The nurse should reinforce that the client should empty their bladder prior to the amniocentesis to reduce the bladder size, which reduces the risk for puncturing the bladder. D - The client is not required to receive the hepatitis B vaccine prior to an amniocentesis.

A nurse in a skilled nursing facility is caring for a group of clients. Which of the following actions demonstrates the nurse's role as client advocate? A - The nurse assists a client in communicating end-of-life decisions to the provider. B - The nurse personalized client medication information to reinforce client teaching. C - The nurse implements a turn schedule to prevent client skin breakdown. D - The nurse consistently assigns the same staff to a client who has dementia.

A - The nurse assists a client in communicating end-of-life decisions to the provider.

A nurse is obtaining informed consent from a client who is scheduled for a procedure. Which of the following actions should the nurse take? A - The nurse should witness the client signing the informed consent document. B - The nurse should ensure that the client understands the benefits of the procedure. C - The nurse should inform the client that once consent is given, it cannot be changed. D - The nurse should inform the client of alternative treatment.

A - The nurse should witness the client signing the informed consent document. Rational A - It is the nurse's role to witness the signature of the client on the informed consent document. B - It is the responsibility of the provider to disclose the risks, benefits, alternatives, and consequences of refusal of the procedure to the client. C - Clients should be informed that consent can be revoked at any time, even after the treatment or procedure has begun. D - It is the responsibility of the provider to disclose the risks, benefits, alternatives, and consequences of refusal of the procedure to the client.

A nurse is providing change-of-shift report for a client who is 3 days postoperative following a transurethral resection of the prostate. Which of the following information should the nurse include? A - The provider changed the client's morphine prescription from IV to PO. B - The client's partner visited for the first time since the surgery. C - The client requires sterile asepsis when the nurse irrigates their bladder. D - The client becomes demanding when they don't get their meal tray on time.

A - The provider changed the client's morphine prescription from IV to PO. Rational A - The nurse should include significant changes to the client's plan of care, such as new medication prescriptions. Using a standard communication method such as the situation-background-assessment-recommendation (SBAR) tool can help the nurse communicate priorities about the client's care. B - Unless there is a specific issue or concern about visitation, it is not necessary to report that the client had a visitor or to convey any opinions about the frequency or duration of visitation. C - The nurse should not describe the steps of a procedure during change-of-shift report. Instead, the nurse should report any essential changes or discontinuation of routine procedures. D - The nurse should not offer personal opinions about the client's behavior during change-of-shift report. The nurse should keep the report factual and objective.

A nurse in a long-term care facility is reveiwing ventilator settings with a newly licensed nurse. Which of the following information should the nurse use to describe the assist-control ventilator mode? A - The ventilator delivers a set tidal volume that helps the client's breathing pattern. B - The ventilator provides a set number of breaths to synchronize with the client's breathing pattern. C - The ventilator controls the rate and depth of all breathing. D- The ventilator maintains positive pressure throughout the breathing cycle.

A - The ventilator delivers a set tidal volume that helps the client's breathing pattern. Rational A - Assist-control ventilation uses the client's own breathing effort to trigger a response by the ventilator. When the client breathes, the ventilator delivers a set tidal volume of air to promote oxygenation and provide rest for the client's lungs. This mode includes setting a minimum respiratory rate and will provide breaths if the client's rate falls below the prescribed number. B - This describes synchronized intermittent mandatory ventilation (SIMV), which delivers a set number of breaths per minute, following the client's own respiratory effort. The client can still breathe independently. These breaths do not affect the number delivered by the ventilator. C - This describes controlled-mode ventilation, which is used when the nervous system is unable to control ventilation, as with medication overdose and anesthetic sedation. D - This describes positive end-expiratory pressure (PEEP), which is used in conjunction with other modes on the ventilator to prevent atelectasis of small airways. This setting is useful with respiratory failure and acute respiratory distress syndrome.

A nurse is examining a client's IV site and notes a red line up his arm. The client reports a throbbing, burning pain at the IV site. The nurse should identify that the client's manifestations indicate which of the following complications of IV therapy? A - Thrombophlebitis B - Infiltration C - Hematoma D - Venous spasms

A - Thrombophlebitis Rational A - The nurse should identify pain, warmth, and a red streak up the arm as indications of thrombophlebitis. b - The nurse should identify swelling and cool skin at the IV site as indications of infiltration. c - The nurse should identify swelling and bruising as indications of a hematoma that can develop by not holding enough pressure after discontinuing the IV. d - The nurse should identify cramping at or above the insertion site and numbness as indications of venous spasms.

A nurse is reviewing the laboratory report of a client who is at 33 weeks of gestation and has gestational hypertension. Which of the following laboratory results should the nurse report to the provider? A - Uric acid 9.6 mg/dL B - Platelet count 150,000/mm3 C - Creatinine 0.8 mg/dL D - Hgb 12 g/dL

A - Uric acid 9.6 mg/dL Rational A - The nurse should report a uric acid level of 9.6 mg/dL because it is above the expected reference range of 2.7 to 7.3 mg/dL. An elevated uric acid level can indicate worsening gestational hypertension or the development of preeclampsia. B - The nurse should identify that a platelet count of 150,000/mm3 is within the expected reference range of 150,000 to 400,000/mm3. A platelet count less than 100,000/mm3 is an indication of preeclampsia or HELLP syndrome and should be reported to the provider. C - The nurse should identify that a creatinine level of 0.8 mg/dL is within the expected reference range of 0.5 to 1.1 mg/dL. An elevated creatinine level is an indication of preeclampsia or HELLP syndrome and should be reported to the provider. D - The nurse should identify a hemoglobin of 12 g/dL is within the expected reference range of greater than 11 mg/dL for a client in the third trimester of pregnancy. A hemoglobin of less than 11 mg/dL can indicate anemia and should be reported to the provider.

A nurse is reveiwing the medical record of a client who has a prescription for morphine. Which of the following findings should the nurse report to the provider? A - Urinary retention B - Administration of celecoxib 24 hr ago C - History of immunosuppression D - Administration of levothyroxine 12 hr ago

A - Urinary retention

A nurse is reviewing the medical records of a group of infants and children. Which of the following is a nationally notifiable infectious condition and requires reporting? A - Varicella B - Scarlet fever C - Fifth disease D - Roseola infantum

A - Varicella Rational A - The nurse should report varicella because it is a nationally notifiable infectious condition. B - Scarlet fever is not a nationally notifiable infectious condition. C - Fifth disease is not a nationally notifiable infectious condition. D - Roseola infantum is not a nationally notifiable infectious condition.

A nurse is contributing to the plan of care for a client who is newly admitted and has anorexia nervosa. The client is 20% below ideal body weight. Which of the following interventions should the nurse recommend including in the client's plan of care? A - Weigh the client each morning. B - Allow the client to rest undisturbed in their room for 1 hr after meals. C - Allow the client to exercise for 30 min each day. D - Establish flexible mealtimes according to the client's preferences.

A - Weigh the client each morning. Rational A - The nurse should weigh the client each morning after voiding using the same scale to monitor progress toward their goal weight. A 0.9 to 1.4 kg (2 to 3.1 lb) weight gain per week is recommended. If the client does not want to see the current weight, the nurse can allow the client to look away from the scale. B - The nurse should observe the client during meals and for at least 1 hr after every meal to prevent purging and to ensure that the client does not hide or throw away food. C - The nurse should not allow the client to begin an exercise program until the client reaches the target weight. Many clients who have anorexia nervosa experience a strong urge to exercise to maintain or lose weight. The nurse can allow the client to resume regular exercise once reaching a weight within 10% of the ideal body weight. D - The nurse should establish consistent mealtimes for the client according to the unit's schedule. The nurse should inform the client of these mealtimes and should avoid bargaining or changing the times according to the client's preferences. Consistent mealtimes decrease client anxiety and power struggles with the health care team.

A nurse is reinforcing dietary teaching with the guardian of a school-age child who has celiac disease. Which of the following foods should the nurse recommend including in the child's diet? A - White rice B - Whole wheat bread C - Graham crackers D - French fries

A - White rice

A nurse is caring for a 4-year-old child who is postoperative. Which of the following pain data collection tools should the nurse use? A - Wong-Baker FACES B - CRIES C - COMFORT-neo D - Numeric rating

A - Wong-Baker FACES Rational A - The nurse should use the Wong-Baker FACES for children older than 3 years old. This scale provides picture representations of various pain intensities. The nurse asks the child to identify which of the pictures best describes their pain level. B - The CRIES scale is a pain data collection tool for newborns who were born at 32 to 40 weeks of gestation. This scale is used to assess postoperative pain. C - The COMFORT-neo scale is a pain data collection tool for newborns who were born at 24 to 42 weeks of gestation and is used to assess prolonged pain. D - The numeric rating scale is used for children older than 8 years old. The child must be able to count and have a basic understanding of numbers and number values.

A nurse is assisting with the neuromuscular assessment of a newborn by eliciting primitive reflexes. Which of the following images indicates a characteristic response of the tonic neck reflex. A - baby on laying on back B - baby on prone C - baby with knees to chest D - baby with arms out

A - baby on laying on back Rational A - The nurse should identify this as the tonic neck reflex. When the newborn's head is quickly turned to one side, the arm and leg on the same side extend, while the arm and leg on the opposite side flex. B - The nurse should identify this as the crawling reflex. When the newborn is placed on the abdomen, he will appear to make crawling movements with the arms and legs. C - The nurse should identify this as the magnet reflex. The newborn will push against the examiner's hands when pressure is applied to the soles of the newborn's feet. D - The nurse should identify this as the Moro reflex. When the newborn hears a loud noise, he will abduct then extend the arms with the fingers widely open and the thumb and index finger form a "C" shape. The lower extremities may also extend then abduct toward the abdomen.

A nurse is preparing to administer multiple immunizations to a client who is receiving chemotherapy and is immunocompromised. Which of the following vaccines should the nurse plan to administer? Select All A - ​Inactivated influenza vaccine ​B - Varicella (VAR) vaccine ​C - Hepatitis B (HepB) vaccine D - Measles, mumps, and rubella (MMR) vaccine ​E - Meningococcal conjugate (MCV4) vaccine

A - ​Inactivated influenza vaccine ​C - Hepatitis B (HepB) vaccine ​E - Meningococcal conjugate (MCV4) vaccine Rational A - Inactivated influenza vaccine is correct. The nurse should recognize that the inactivated influenza vaccine is not a live vaccine and is safe to administer to a client who is immunocompromised. Clients who are immunocompromised should not receive live vaccines because of the inability of the client's immune system to protect the client against the disease. B - Varicella (VAR) vaccine is incorrect. The nurse should recognize that the VAR vaccine is a live vaccine, which is contraindicated for a client who is immunocompromised. C - Hepatitis B (HepB) vaccine is correct. The nurse should recognize that the HepB vaccine is not a live vaccine and is safe to administer. Clients who are immunocompromised should not receive live vaccines because of the inability of the client's immune system to protect the client against the disease. D - Measles, mumps, and rubella (MMR) vaccine is incorrect. The nurse should recognize that the MMR vaccine is a live vaccine, which is contraindicated for a client who is immunocompromised. E - Meningococcal conjugate (MCV4) vaccine is correct. The nurse should recognize that the MCV4 vaccine is not a live vaccine and is safe to administer. Clients who are immunocompromised should not receive live vaccines because of the inability of the client's immune system to protect the client against the disease.

A nurse is caring for a client who is postoperative following a total left hip arthroplasty. Which of the following interventions should the nurse implement to prevent hip dislocation? A - ​Maintain a foam wedge between the client's legs. ​B - Encourage the client to use elastic stockings. ​C - Monitor for shortening of the client's affected leg. D - Perform passive range of motion by flexing the client's hip to 120°.

A - ​Maintain a foam wedge between the client's legs. Rational A - The nurse should maintain a foam wedge between the client's legs to keep the affected leg slightly abducted and reduce the risk for hip dislocation. A major complication of total hip arthroplasty is subluxation (partial dislocation) or total dislocation. In some facilities, abduction devices, such as foam wedges and pillows, are placed between the legs. Adduction of the hip should be avoided to prevent dislocation. B - The nurse should apply elastic stockings or a sequential compression device on the client to reduce the risk for venous thromboembolism, but this will not prevent dislocation of the hip. To decrease the risk for venous thromboembolism, the nurse should also reinforce teaching about leg exercises, encourage fluid intake, administer prescribed anticoagulants, and observe for redness, swelling, pain, or changes in mental status. C - The nurse should monitor for shortening of the client's affected leg to identify, rather than prevent dislocation. Manifestations of dislocation include shortening of the extremity, pain, and external rotation of the extremity. The nurse should report the presence of these findings to the provider immediately. D - The nurse should avoid flexing the client's hips more than 90° when performing passive range of motion for the client to reduce the risk for hip dislocation.

A nurse is reinforcing teaching with a client who has a new prescription for crutches with no weight bearing on the left leg. Which of the following statements should the nurse include? A -"Advance your right leg first when going up stairs." B - "Your elbows should be flexed to about 45 degrees." C - "Rest the crutch pads against your axillae." D - "You should utilize the four-point crutch gait when ambulating."

A -"Advance your right leg first when going up stairs." Rational A - When ascending the stairs, the client should advance with the unaffected leg first. When descending the stairs, the client should advance the crutches first, then advance the affected leg. B - The client's elbows should be flexed between 15 and 30 degrees to reduce to the risk of injury to the joints. C - The nurse should instruct the client to bear weight on their arms rather than rest the crutch pads against their axillae. Resting the crutches directly under the arms can cause nerve damage. There should be at least two to three fingerbreadths between the crutch pad and axilla. D - The nurse should instruct the client to use a three-point crutch gait. The four-point crutch gait requires weight bearing on each leg.

A nurse is speaking with a client who is expressing an intense disapproval of the current social worker. When the social worker approaches the nurse and client a few moments later, the client cheerfully states, "Now, here is my favorite social worker!" The nurse should identify the client is using which of the following defence mechanisms? A -Reaction formation B - Dissociation C - Denial D - Projection

A -Reaction formation

A nurse is caring for a client who has major depressive disorder (MDD). The client states, "I have nothing to live for anymore. I just can't go on." Which of the following responses should the nurse make? A- "Are you thinking about ending your life?" B - "Don't you think it's best to let your family know how you feel?" C - "I'm sure you can't mean that. You have everything to live for." D - "I know how you feel right now. Everything will be okay."

A- "Are you thinking about ending your life?"

A nurse is reinforcing teaching about gastroesophageal reflux disease (GERD) with a client. Which of the following statements by the client indicates an understanding of the teaching? A- "I should wait at least 2 hours after eating before going to bed." B - "I should eat three meals a day without eating snacks between meals." C - "I should season my food with garlic." D - "I should drink my liquids through a straw."

A- "I should wait at least 2 hours after eating before going to bed."

A nurse is reinforcing discharge teaching with the guardian of a child who has juvenile idiopathic arthritis (JIA). Which of the following statements by the parent indicates an understanding of the teaching? A- "I will have my child sleep in knee, wrist, and hand splints." B - "I will encourage my child to take an afternoon nap." C - "I will apply topical hydrocortisone to my child's joints as needed." D - "I will administer opioids to my child for the next several months to control the pain."

A- "I will have my child sleep in knee, wrist, and hand splints."

A nurse is caring for a client who is agitated. The client yells, "I hate all the nurses in this place!" Which of the following responses should the nurse make? A- "You seem to have strong feelings about the nurses here." B - "You must immediately stop this behavior." C - "Why are you so angry toward nurses?" D - "It is good you are expressing your real feelings about nurses."

A- "You seem to have strong feelings about the nurses here." Rational A - This response validates the client's emotions and is an open-ended statement. This type of statement encourages further communication and is nonthreatening toward the client, which can promote de-escalation. B - This statement demonstrates disapproval of the client's words and feelings. The nurse should not make this response because it can cause the client to feel defensive, which can cause their aggressive behavior to escalate. The nurse should validate the client's emotions and use open-ended responses to obtain more information about the client's feelings. Limit-setting is appropriate if the client demonstrates harmful behavior, but not for verbal expressions of anger toward staff. C - This statement criticizes the client's feelings. The nurse should avoid asking a "why" question because this type of response can make the client feel defensive. The nurse should implement de-escalation techniques to help calm the client and reduce the risk of harm to others. The nurse should remain calm when responding to a client who is angry. D - This statement demonstrates approval of the client's behavior. The nurse should not make this response because it implies that it is acceptable for the client to communicate through yelling. The nurse should use open-ended responses to obtain more information about the client's feelings.

A nurse is caring for a client who has an acute ischemic stroke 1 day ago. Which of the following actions should the nurse take to reduce the risk for aspiration? A- Allow for 30 min of rest before meals. B - Provide a straw for drinking liquids. C - Serve foods at room temperature. D - Place 2 tsp of food in the client's mouth at a time.

A- Allow for 30 min of rest before meals. Rational A - The nurse should allow the client to rest for 30 min before meals to prevent aspiration. B - The nurse should provide a cup for drinking liquids, rather than a straw. C - The nurse should serve foods that are cold or heated. It is more difficult for the client to swallow food that is lukewarm or at room temperature. D - The nurse should place only 1 tsp of food in the client's mouth at a time.

A nurse is auscultating heart sounds on an infant. The nurse should identify this sound as which of the following? A- Sinus rhythm B - Ventricular septal defect C - Pulmonic stenosis D - Atrial septal defect

A- Sinus rhythm

A nurse is reinforcing teaching about prevention of toxoplasmosis with a client who is at 10 weeks of gestation. Which of the following statements should the nurse include in the teaching? A - "Practice safe sex and use a condom for sexual intercourse." B - "Avoid contact with cat litter." C - "Wear a mask when out in large crowds." D - "Avoid wearing tight-fitting undergarments in the perineal area."

B - "Avoid contact with cat litter." Rational A - Toxoplasmosis is not a sexually transmitted infection. B - Toxoplasmosis is caused by consumption of raw or undercooked meat or the handling of contaminated cat feces. The nurse should reinforce to the client that maternal infection with toxoplasmosis, especially during the first trimester of pregnancy, can result in fetal malformations and fetal loss. The nurse should instruct the client to avoid changing cat litter boxes, working in soil that might be contaminated with cat feces, and handling or consuming raw or undercooked meat. The nurse should also reinforce to the client the importance of proper handwashing after contact with raw meat. C - Toxoplasmosis is not an infection requiring airborne precautions. D - Avoiding tight-fitting undergarments is a recommendation for the prevention of urinary tract infections, not for toxoplasmosis.

A charge nurse is discussing the steps to complete an incident report with a newly licensed nurse. Which of the following statements should the nurse include? A - "Place a copy of the incident report in the client's medical record." B - "Complete the incident report after ensuring the client is not injured." C - "Document the completion of the incident report in the nurse's notes." D - "Include subjective data in the incident report."

B - "Complete the incident report after ensuring the client is not injured." Rational A - The nurse should not place a copy of the report in the client's medical record. The incident report is an internal document and is not part of the medical record, which is a legal document. B - An incident report is the record of an occurrence, such as a client fall or medication error. The first step the nurse should take following an unusual occurrence is to collect data from the client to determine if an injury has occurred. After providing care to the client, the nurse should contact the provider and complete the incident report as soon as possible. C - The nurse should document the facts of the event in the client's medical record. However, they should not document that an incident report was completed in the nurse's notes. D - The nurse should include objective statements that are factual and complete, with a clear description of the incident that occurred.

A nurse is reinforcing discharge teaching with a client who has a new prescription for alprazolam. Which of the following instructions is the priority for the nurse to include? A - "Avoid drinking beverages that contain caffeine." B - "Do not drive until your reaction to the medication is determined." C - "Avoid taking naps in the daytime." D - "Take this medication with a light snack."

B - "Do not drive until your reaction to the medication is determined."

A nurse on a pediatric unit is checking a toddler's vital signs. Which of the following comments should the nurse make? A - "Can I listen to the sounds of your lungs?" B - "I am going to listen to your heart." C - "I am going to take your blood pressure now." D - "Can you stand still while I take your temperature?"

B - "I am going to listen to your heart." Rational A - The nurse should not ask "yes/no" questions in this situation. The toddler will initially resist the nurse's attempt to obtain vital signs and most likely will answer "no." If the nurse asks the question and the toddler responds "no," and the nurse proceeds anyway, it can create an environment of mistrust between the toddler and the nurse. B - The nurse should inform the toddler of exactly what they are going to do before performing the procedure without asking the toddler's permission. C - The nurse should use non-threatening words and phrases when communicating with a toddler. The nurse should avoid using the word "take." D - The nurse should not ask "yes/no" questions in this situation. The toddler will initially resist the attempt at taking vital signs and most likely will answer "no." If the nurse asks the question and the toddler responds "no," and the nurse proceeds anyway, it can create an environment of mistrust between the toddler and the nurse.

A nurse is reinforcing teaching with a client who has asthma. Which of the following client statements indicated an understanding of the use of budesonide and albuterol inhalers? select all A - "I should expect to feel sleepy after using my albuterol inhaler." B - "I never forget to rinse my mouth after using my budesonide inhaler." C - "Between office visits, I keep a record of how many times I use my albuterol inhaler." D - "I use my albuterol inhaler before I go swimming." E - "I should use my budesonide inhaler before using my albuterol inhaler."

B - "I never forget to rinse my mouth after using my budesonide inhaler." C - "Between office visits, I keep a record of how many times I use my albuterol inhaler." D - "I use my albuterol inhaler before I go swimming." Rational A - "I should expect to feel sleepy after using my albuterol inhaler" is incorrect. The client should recognize that albuterol stimulates the sympathetic nervous system, which can cause nervousness and insomnia, along with increased heart rate and blood pressure. B - "I never forget to rinse my mouth after using my budesonide inhaler" is correct. The client should rinse his mouth after using a budesonide inhaler to reduce the risk for oral fungal infection. C - "Between office visits, I keep a record of how many times I use my albuterol inhaler" is correct. The client should record the number of times that he uses his albuterol inhaler. This information can assist the provider to determine the effectiveness of the medication. D - "I use my albuterol inhaler before I go swimming" is correct. The client should use the albuterol inhaler before exercise to prevent exercise-induced bronchospasms. E - "I should use my budesonide inhaler before using my albuterol inhaler" is incorrect. The client should first use the albuterol inhaler, a bronchodilator, to open the airway and enhance the absorption of the budesonide, which is an inhaled corticosteroid.

A nurse is discussing family planning with a client who is requesting information about available contraceptive methods. Which of the following client statement indicates an understanding of the teaching? A - "When I use the diaphragm, I should remove it 2 hours after intercourse." B - "I should use water-soluble lubricant when my partner wears a condom." C - "I should remove the birth control sponge 24 hours after intercourse." D - "When I use the birth control patch, it must be changed once a month."

B - "I should use water-soluble lubricant when my partner wears a condom." Rational A - The diaphragm should be left in place for at least 6 hr following intercourse. B - Only water-soluble lubricants should be used with male condoms, because the use of any other lubricant may compromise the integrity of the condom. C - The sponge should be left in place for at least 6 hr, but less than 24 hr, following intercourse. Leaving the sponge in for 24 hr or longer increases the risk of toxic shock syndrome. D - The patch is changed weekly for 3 weeks, followed by 1 week in which the client does not wear the patch.

A nurse is reinforcing discharge teaching about newborn care with a first-time parent Which of the following statements by the parent indicates and understanding of the teaching? A - "I will use talcum powder to prevent a diaper rash." B - "I will clean the skin around the umbilical cord with warm water." C - "I will clean inside my baby's ears with a cotton-tipped swab." D - "I will keep the water at 110 degrees Fahrenheit for my baby's bath."

B - "I will clean the skin around the umbilical cord with warm water." Rational A - Talcum powder contains zinc which can be distressing to the newborn's respiratory tract and can cause an allergic response if inhaled. Therefore, the nurse should instruct the parent to avoid using talcum powder to prevent aspiration pneumonia. B - The nurse should instruct the parent to clean the base of the newborn's umbilical cord where it joins the skin using warm water. Soap should be avoided because it causes drying of the skin to occur, which can lead to peeling of the epidermis in this area. The umbilical cord should fall off in about 10 to 14 days. C - The nurse should instruct the parent not to use a cotton-tipped swab inside the newborn's ear. This can cause injury because newborns have short ear canals. Parents should clean the ears of a newborn using the edge of a washcloth. D - The nurse should instruct the parent to use water that is about 38° C (100° F) to prevent burns to the newborn's skin. The client should test the water's temperature on their inner wrist to ensure that the temperature feels correct. The nurse should also remind the parent to keep the temperature in the room at about 24° C (75° F) for bathing the newborn to prevent cold stress.

A nurse in a maternal-newborn unit is caring for a newborn in the nursery. The newborn's grandfather asks if he may take the newborn to his daughter's room. Which of the following responses should the nurse make? A - "I'll first need to see your photo ID before I can release the baby to you." B - "Let me wash my hands and then I'll take your grandson to his mother." C -"Please wash your hands first, then I'll allow you to carry the baby to your daughter's room." D - "Have your daughter call the nursery so that the staff can release the baby to you."

B - "Let me wash my hands and then I'll take your grandson to his mother."

A nurse is reinforcing teaching with a client who has dumping syndrome as a complication of gastric bypass surgery. Which of the following client statements indicates an understanding of the teaching? A- "I will take a short walk after each meal." B - "I will eat some type of protein with each meal." C - "I will drink at least 8 ounces of liquid with each meal." D - "I will eat only three meals a day."

B - "I will eat some type of protein with each meal." Rational A - The nurse should instruct the client to lie down for 30 to 60 min after each meal to slow the passage of food into the intestine. B - The nurse should instruct the client to eat protein with each meal to slow gastric emptying. Dumping syndrome can occur following gastric bypass surgery because a large amount of undigested food enters the intestines at a rapid rate. The client should minimize intake of simple carbohydrates and increase intake of fats and proteins. C - The nurse should instruct the client to consume liquids between meals because liquids increase the size of the food bolus. D - The nurse should instruct the client to consume small, frequent meals to reduce the size of the food bolus.

A nurse in an antepartum clinic is reinforcing teaching about how to prevent supine hypotension with a client who is at 16 weeks of gestation. Which of the following responses by the client indicates an understanding of the teaching? A - "I will apply support stockings 30 minutes after getting out of bed." B - "I will lie on my left side with my head elevated on a pillow." C - "I will cross my legs when sitting." D - "I will limit my salt intake."

B - "I will lie on my left side with my head elevated on a pillow."

A nurse is reinforcing teaching about home safety with the parent of a toddler. Which of the following parent statements indicates an understanding of the teaching? A - "I will keep my hearing aid batteries in my bedside table." B - "I will place a screen in front of the fireplace." C - "I will keep my medication in my purse." D - "I will use a steam vaporizer when my child has a cold."

B - "I will place a screen in front of the fireplace."

A nurse is reinforcing teaching about car seat safety with the guardian of an 18-month-old toddler who weights 10.9 kg (24 lb). Which of the following statements by the guardian indicates an understanding of the instructions? A- "I will not allow my child to play with a stuffed animal while in the car seat." B - "I will secure my child's car seat rear-facing in the back seat." C - "I will put a small pillow behind my child's back while in the car seat." D - "I will ensure the retainer clip fits snugly across my child's abdomen."

B - "I will secure my child's car seat rear-facing in the back seat." Rational A - The guardian should allow the toddler to have a favorite blanket, toy, or stuffed animal while riding in the car seat. This decreases boredom and discourages attempts to unfasten the car seat harness. B - The nurse should remind the guardian that the safest place for a toddler to ride in a motor-vehicle is the back seat. It is recommended that a child is placed rear-facing in a back seat until the age of 2 or until the child outgrows the limits specified by the manufacturer of the car seat. C - The guardian should not place anything between the toddler and the car seat or restraints. This affects the positioning and tightness of the harness straps, increasing the risk for injury during a motor-vehicle crash. D - The guardian should not place the retainer clip across the toddler's abdomen, because this increases the risk for injury during a motor vehicle crash. The retainer clip should be placed at the level of the toddler's armpits.

A nurse is reinforcing teaching about thought stopping with a client who has a phobia of rising in automobiles. Which of the following client statements indicates an understanding of the instructions? A - "For the first step of my therapy, I will look at pictures of cars." B - "I will snap a rubber band on my wrist when I feel anxious about riding in a car." C -"My therapist will be with me while we ride in a car together." D -"I will ride in a car for several hours at a time."

B - "I will snap a rubber band on my wrist when I feel anxious about riding in a car." Rational A - This statement describes systematic desensitization, not thought stopping. B - This statement describes thought stopping, which is used to interrupt a client's negative thought with a distraction. C - This statement describes modeling, not thought stopping. D - This statement describes flooding, not thought stopping.

A nurse is reinforcing teaching to a group of clients who have each been prescribed new medications. Which of the following statements by a client indicates an understanding of their medication therapy? A - "I will take what is left from my previous sulfadiazine prescription if my UTI returns." B - "I will take ibuprofen to manage the pain from my rheumatoid arthritis." C - "I will take propranolol to manage my type 2 diabetes." D - "I will take aspirin to help heal my peptic ulcer."

B - "I will take ibuprofen to manage the pain from my rheumatoid arthritis." Rational A - Even though symptoms might resolve before the full prescription is taken, clients should be reminded to complete the prescribed course of antibiotic treatment for any infection. B - Ibuprofen is classified as a nonsteroidal anti-inflammatory (NSAID) and is used to treat rheumatoid arthritis and osteoarthritis, as well as mild to moderate pain and fever. Ibuprofen decreases inflammation caused by arthritis. C - Propranolol is a beta-blocker used to manage cardiac arrhythmias, myocardial infarction, angina pectoris, and hypertension. It is not used to treat type 2 diabetes mellitus. D - The use of aspirin is contraindicated for a client who has peptic ulcer disease due to the increased risk of GI bleeding.

A nurse is reinforcing teaching with a client who has type 1 diabetes mellitus about safe home disposal of insulin syringes and needles. Which of the following statements by the client indicates an understanding of the teaching? A - "I'll recap the needles and discard them in their original wrappers in a metal trash can." B - "I'll collect the needles in a rigid plastic laundry detergent container and take them to a hazardous waste facility." C - "I'll put the needles in a sealed red bag and bring them to the hospital for disposal." D - "I'll collect the needles in a disposable aluminum pie plate and fold it in half before I put it in the trash."

B - "I'll collect the needles in a rigid plastic laundry detergent container and take them to a hazardous waste facility." Rational A - The client risks injury to themself and others by recapping the needles and discarding them in a trash can. B - The client should use an impervious container made of heavy plastic, such as a laundry detergent container, to prevent self-injury. The client can take the container to a community drop-off program or a hazardous waste facility for disposal. C - Used needles can puncture a plastic bag and cause injury to the client and others. A hospital does not provide disposal services for clients. D - A folded pie plate could allow needles and syringes to slip out and cause injury to the client and others. Furthermore, discarded needles cannot be placed in the trash. They must be taken to a hazardous waste facility and incinerated.

A nurse is reinforcing teaching with a client who is using a PCA pump to deliver morphine for postoperative pain management. Which of the following statements by the client indicates an understanding of the teaching? A - "I'll ask my son to push the dose button when I am asleep." B - "I'll push the button about 10 minutes before I request assistance to get out bed." C - "I'll be careful about pushing the button too often so I don't overdose." D - "I will use the PCA for pain control until I go home."

B - "I'll push the button about 10 minutes before I request assistance to get out bed." Rational A - The client should be the only person to operate the PCA pump. The nurse should reinforce with the client's family that the PCA pump should only be activated by the client and only when the client is awake to avoid administering more medication than is required for pain management. B - A PCA device allows the client to self-administer pain medication on an as-needed basis. The client should activate the device prophylactically about 10 min before potentially painful activities, such as ambulation and dressing changes. The analgesic effect will take about 10 min from the time of administration until the client feels relief. The client should plan activities such as getting out of bed and push the button prior to the activity. C - PCA devices have a timing control or lockout mechanism that allows a preset minimum interval between medication doses and also limits the dose per hour. This safety feature prevents overdoses of the analgesic. D - Patient-controlled analgesia is used for pain relief in the immediate postoperative period during the time the client is NPO. Once a diet is restarted, clients are encouraged to use PO opioids for pain control. This ensures that the client can tolerate oral medication and that the client's pain is well controlled.

A nurse is reinforcing teaching about car seat safety with a parent of a newborn. Which of the following statements should the nurse identify as an indication that the client understands teh instructions? A - "My baby should be in a rear-facing car seat until he is 6 months old and 15 pounds." B - "If my baby rides in a car with no back seat, the passenger air bag must be turned off." C - "It is dangerous to secure the car seat using the vehicle's seat belts." D - "I will place my baby's car seat at a 90 degree angle in the back seat."

B - "If my baby rides in a car with no back seat, the passenger air bag must be turned off."

A nurse is reinforcing teaching with a client who is about to begin chemotherapy and is prescribed a medication that causes alopecia. Which of the following statements should the nurse make? A - "Hair loss following chemotherapy is, unfortunately, a permanent change." B - "If you choose to select a head covering, it would be best to do so before hair loss occurs." C - "If you apply a heating pad to your head during chemotherapy it can reduce hair loss." D - "Hair loss is a minor effect of medication compared to the problems cancer can cause."

B - "If you choose to select a head covering, it would be best to do so before hair loss occurs." Rational A - The nurse should instruct the client that chemotherapy-induced alopecia is temporary. Hair growth usually begins about 30 days after the last chemotherapy treatment. B - Hair regrowth after chemotherapy can appear very different from the client's original hair. The nurse should suggest the client select a wig or hair cover before hair is lost, so the client has the opportunity to find something that will match existing hair. C - The nurse should instruct the client that the use of an approved ice cap during chemotherapy might reduce hair loss by decreasing circulation to hair follicles. D - This response by the nurse is nontherapeutic because the nurse is ignoring the client's feelings about body image, and blocking communication. The nurse should communicate in a way that offers acceptance and encourages the client to openly express their feelings.

A nurse is reinforcing teaching with a client who has a new prescription for ferrous sulfate. Which of the following instructions should the nurse include? A - "Take the medication with an 8-ounce glass of milk." B - "Increase your fluid intake with this medication." C - "Avoid eating citrus fruits while taking this medication." D - "Take a double dose of the medication if you miss a dose."

B - "Increase your fluid intake with this medication."

A nurse is reinforcing teaching with a client who has a hearing aid. Which of the following instructions should the nurse include? A - "Avoid cleaning the ear canal with hydrogen peroxide." B - "Inspect the dials for cracks." C - "If you use hair spray, spray around the hearing aid." D - "If static is detected, the device might be too loud."

B - "Inspect the dials for cracks." Rational A - The nurse should instruct the client to cleanse the ear canal daily with hydrogen peroxide or mineral oil to soften the cerumen. B - Cracks are likely to occur on the dials, earmold, earphone, or connection and plugs, and can ruin the integrity of the device. If a client sees a crack on their device, they should contact the audiologist for repairs. C - The nurse should instruct the client to avoid using hair spray, as this can damage the hearing aid. D - The nurse should instruct the client that static can indicate a hearing aid malfunction. The nurse should educate the client on reporting static, distortion of sound, or poor volume quality to the audiologist for further evaluation.

A nurse is reinforcing teaching with a client about managing hearing loss. Which of the following statements by the client indicates an understanding of the teaching? A - "Reading lips is a good alternative to wearing a hearing aid." B - "It can take a several weeks to adjust to a hearing aid when it is first used." C - "I should wait to use hearing aids until other alternatives no longer work." D - "Cleaning my ears daily with a cotton-tipped swab can prevent further hearing loss."

B - "It can take a several weeks to adjust to a hearing aid when it is first used." Rational A - Reading lips can be helpful, but not all words can be understood through reading lips and the client can miss almost half of all communication. It is not recommended that the client rely on reading lips alone. B - After a client is initially fitted for a hearing aid, the client might need to return to the audiologist for repeated fittings and adjustments of the settings. It is important for the client to understand the difficulties associated with adjusting to a hearing aid to prevent the client from getting frustrated and giving up. C - The nurse should encourage the client to use a hearing aid and reinforce that using a hearing aid sooner, rather than later, will make it easier for the client's brain to adjust to the aid. This improves the quality of hearing and increases the client's ability to communicate effectively. D - While impacted cerumen can affect hearing acuity, the nurse should reinforce that inserting any object into the ear can cause damage to the ear. The client can safely remove cerumen by instilling ear drops that loosen the cerumen, or a health care provider can manually remove it.

A nurse is reinforcing teaching with a client who has systemic lupus erythematosus (SLE) and is to begin taking methylprednisolone orally. Which of the following statements should the nurse include in the teaching? A - "Take the medication on an empty stomach." B - "Limit contact with large groups of people." C - "Avoid taking over-the-counter calcium supplements." D - "Follow a low-protein diet."

B - "Limit contact with large groups of people."

A nurse is caring for an older adult client who has early dementia and is about to be admitted to a facility. THe client currently lives in a private home with their adult child and states, "I think my child is stealing my money." Which of the following responses should the nurse make? A - "We're going to make sure all of your needs are met while you're here." B - "Tell me more about what you think about your living situation." C - "Let's find some music or a book you would enjoy to pass the time." D - "You're probably misunderstanding the cost of your health care bills."

B - "Tell me more about what you think about your living situation." Rational A - This statement by the nurse changes the subject from the client's statement. The nurse should respond in a way that promotes a therapeutic relationship with the client, rather than avoiding uncomfortable discussion. B - The nurse should collect further data from the client when there are possible indications of abuse. The nurse should follow ethical principles such as beneficence, doing what is best for the client, in promoting safety for the client. This statement by the nurse will further explore the client's concerns so the nurse can take proper action if there is a legitimate concern. C - Although music or books can be a useful distraction to a client who is confused or in pain, it is inappropriate for the nurse to suggest a distraction at this time. There is no indication that the client is currently confused, and the nurse must promote the client's right to self-expression through therapeutic communication. D - This statement by the nurse challenges the client's point of view, which negates the client's feelings. The nurse should avoid making judgments of client statements. This prevents the client from feeling free to express personal opinions and information, which could hinder the nurse-client relationship.

A nurse is checking the vital sings of a 6-month-old infant. The infant has a temperature of 40°C (104°F) and a heart rate of 180/min. The guardian asks the nurse, "Why is my baby's heart beating so fast?" Which of the following responses should the nurse make? A - "This heart rate is within the expected range for your baby." B - "The fever is causing your baby's heart rate to increase." C - "As your baby begins to fall asleep, his heart rate might increase." D - "Your baby's heart rate is increased due to an increased calcium level."

B - "The fever is causing your baby's heart rate to increase." Rational A - This heart rate is not within the expected reference range for an infant. B - The nurse should inform the guardian that, because the infant's temperature is elevated, the heart rate is increased due to the increase in metabolic rate. The heart rate is affected by factors such as increased physical activity, crying, and elevated temperature. The metabolic rate increases 10% for every 1° C increase in temperature. C - An infant's heart rate might decrease slightly while asleep. However, this infant's heart rate is higher than expected due to the fever. Once the temperature is controlled, the infant's heart rate should decrease to the expected range. D - There is no indication that the increase in the infant's heart rate is due to hypercalcemia. Hypercalcemia can cause bradycardia and cardiac arrest.

A nurse is reinforcing teaching with a 16-year-old client who requests information about the human papillomavirus (HPV) immunization. Which of the following responses should the nurse make? A - "This immunization will provide protection against gonorrhea." B - "This immunization comes in a series of three doses." C - "You are not eligible to receive this immunization until you are 18 years old." D - "You should not receive this immunization if you are allergic to eggs."

B - "This immunization comes in a series of three doses." Rational A - The HPV vaccine does not provide protection against gonorrhea. It is administered to prevent cervical cancers caused by HPV. B - The nurse should inform the adolescent that the HPV vaccine is administered in a series of two or three doses. The first and second doses are administered 6 to 12 months apart. Any adolescent who receives both of these doses within 5 months will require a third dose. C - It is recommended that clients receive the HPV vaccine at the age of 11 to 12; however, it can be administered at a minimum age of 9. D - The HPV vaccine is contraindicated for clients who have had a severe allergic reaction to a previous dose of the vaccine.

A nurse is reinforcing teaching with a client who has a prescription for periodic aPTT testing. Which of the following client statements indicates an understanding of the teaching? A - "I will need to avoid eating or drinking prior to this test." B - "This test measures the effectiveness of clotting factors." C - "I might need to receive vitamin K if my levels are too low." D - "This test will help my provider adjust my warfarin dosages."

B - "This test measures the effectiveness of clotting factors." Rational A - A client who is having an aPTT does not need to remain NPO for any period of time. An aPTT assesses the intrinsic clotting cascade and the action of coagulation factors. It is prolonged whenever any of the coagulation factors are deficient. B - An aPTT test measures the effectiveness of coagulation factors in the clotting cascade. Clients who are receiving heparin therapy should have regular testing of the aPTT. The therapeutic range of heparin therapy is 1.5 to 2.5 times the expected range in seconds. Prolonged times indicate a risk for serious bleeding. C - The client should receive protamine sulfate to reverse heparin; vitamin K is the reversal agent for warfarin. If the client's levels are high, the client is at risk for bleeding. D - The provider will monitor the client's prothrombin time (PT) and INR to adjust warfarin dosages. The nurse should expect the PT and INR values to be above the expected reference range but still within the target values prescribed by the provider.

A nurse in a provider's office is preparing to administer the influenza the influenza vaccine to a group of adult clients. The nurse should identify that which of the following clients has a contraindication to receiving the vaccine? A - A client who has a shellfish allergy B - A client who has Guillain-Barré syndrome C - A client who is taking aspirin therapy D - A client who has a fractured femur

B - A client who has Guillain-Barré syndrome Rational A - A client who has a shellfish allergy has no contraindication to receiving the influenza vaccine. However, a client who is allergic to chicken eggs should not receive the influenza vaccine. B - Receiving either the live or inactivated influenza vaccine is contraindicated for a client who has Guillain-Barré syndrome. This client should not receive the influenza vaccine. C - A client who is on aspirin therapy has no contraindication to receiving the influenza vaccine. D - A client who has a fractured femur has no contraindication to receiving the influenza vaccine.

A nurse is assisting with the admission of a client to a medical-surgical unit. Which of the following statements should the nurse make to the client? A - "We will keep your vital signs on a message board in your room." B - "You need to give written permission for your medical information to be released." C - "We must let you know each time new health care personnel looks at your chart." D - "You can sign a general consent now that will cover all hospital procedures."

B - "You need to give written permission for your medical information to be released." Rational A - The nurse should not include a client's personal health information on a message board in the client's room. This is a violation of confidentiality. B - Under HIPAA privacy laws, client consent is required to release medical information. The nurse should reinforce with the client that the requirement is in place to protect the client's information. Only those directly involved in client care have a right to access the information. C - HIPAA laws allow information to be shared with health care personnel directly involved in client care. This can include providers, nurses, and therapists from various disciplines. It is not a requirement to notify the client each time the medical record is reviewed. D - A general consent form is signed on admission to a facility and covers most routine procedures. A separate consent must be obtained before certain procedures or invasive diagnostic tests are performed.

A nurse is reinforcing teaching with a client who has a new prescription for rifampin. Which of the following instructions should the nurse include? A - "Avoid drinking grapefruit juice." B - "You should expect to experience red-orange discoloration in your tears." C - "You may experience a decreased libido due to this medication." D - "Take this medication with meals."

B - "You should expect to experience red-orange discoloration in your tears." Rational A - The nurse should reinforce that the client should avoid drinking alcohol while taking this medication because rifampin can cause hepatotoxicity. B - The nurse should reinforce that rifampin can cause red-orange discoloration of tears, urine, sweat, and saliva. This is a temporary and harmless adverse effect of the medication. C - Decreased libido is not an expected effect of this medication. D - The nurse should reinforce that rifampin should be taken 1 hr before or 2 hr after meals to increase absorption.

A nurse is reinforcing teaching with a client who has a prescription for calcitonin. Which of the following statements should the nurse include in the teaching? A - "This medication can cause impaired thyroid function." B - "You should not take this medication if you are allergic to salmon." C - "This medication can increase your risk for spinal and hip fractures." D - "You should not drink orange juice when taking this medication."

B - "You should not take this medication if you are allergic to salmon." Rational A - Calcitonin is a hormone secreted by the thyroid gland. It affects the parathyroid gland, serum calcium levels, and bone calcium resorption. Calcitonin does not cause impaired thyroid function. B - Synthesized calcitonin is a hormone replacement medication derived from salmon. Clients who have a pre-existing allergy to salmon should not take this medication. C - Therapeutic effects of calcitonin include increased bone density, which can help to decrease the incidence of bone fractures. D - The nurse should identify that bisphosphonates should not be taken with orange juice because it can hinder the client's ability to absorb the medication, reducing the therapeutic effect of the medication. Calcitonin is a hormone that promotes calcium resorption into the bones and is not a bisphosphonate. Orange juice is not contraindicated when taking this medication.

A nurse is reinforcing teaching with an adolescent female client who has acne vulgaris and a new prescription for isotretinoin. Which of the following should the nurse include? A - "You should apply this medication to the affected skin twice daily." B - "You will need to have two negative pregnancy tests prior to starting this medication." C - "Your provider will monitor your kidney function while you are taking this medication." D - "Your provider will prescribe a vitamin A supplement to take with each dose of this medication."

B - "You will need to have two negative pregnancy tests prior to starting this medication."

A nurse is reinforcing teaching with a client who has preeclampsia and is receiving magnesium sulfate via continuous IV infusion. Which of the following statements should the nurse include in the teaching? A - "We will monitor your blood pressure every 2 hours." B - "Your fluid intake will be limited to no more than 125 milliliters per hour." C - "You might notice that you will begin breathing faster than normal." D - "We will monitor your baby's heart rate once per hour."

B - "Your fluid intake will be limited to no more than 125 milliliters per hour."

A nurse is contributing to a discussion about informed consent during a staff meeting. Which of the following clients should the nurse identify as requiring a guardian to provide consent for general treatment? A - A young adult client who has schizophrenia B - A 17-year-old client who dropped out of high school C - A 16-year-old client who has a newborn D - An older adult client who has brain cancer

B - A 17-year-old client who dropped out of high school Rational A - Mental illness does not make an individual incapable of providing consent. If the client's mental capacity becomes questionable, health care personnel should determine whether the client is still competent. A court ruling might be required to declare incompetence. B - Minors are required to have a parent or guardian provide consent for general medical care. Some states allow minors to give consent for certain treatments, such as for a mental illness or sexually transmitted infection. An emancipated minor can give consent. C - A minor who has a child is considered emancipated and can provide consent. Minors also are able to provide consent for children of whom they are guardians or have custody. In some states a client who is pregnant might be considered emancipated. D - An older adult client who has brain cancer can provide legal consent for care and treatment. The nurse should encourage all clients to complete advance directives, especially in situations where the client's diagnosis could affect judgment in the future.

A nurse is assisting with a sterile dressing change for an adolescent who has partial thickness burn on the right hip. Which of the following actions should the nurse take first? A - Open the sterile dressing tray. B - Administer pain medication to the client. C - Assist the client into the left lateral position. D - Remove the previous dressing to inspect the wound.

B - Administer pain medication to the client.

A nurse is assisting with the care of a child who has tonic-clonic seizures. Which of the following actions should the nurse take? A - Ensure the availability of soft extremity restraints. B - Place a padded tongue blade at the bedside. C - Have a suction canister and tubing available in the room. D - Keep the child's bed in the highest position.

C - Have a suction canister and tubing available in the room.

A nurse is assisting with assigning care for a group of clients. The nurse should instruct the assistive personnel (AP) to assist with ADLs for which of the following clients? Select all A - A client who is newly admitted and is having an episode of status asthmaticus B - A client who has a history of mild chronic heart failure and had a knee arthroplasty 2 days ago C - A young adult client who is 1 day postoperative following a laparoscopic appendectomy D - A client who is being discharged to home and has a new diagnosis of Crohn's disease E - A client who has diabetic ketoacidosis and is receiving regular insulin via a continuous IV infusion

B - A client who has a history of mild chronic heart failure and had a knee arthroplasty 2 days ago C - A young adult client who is 1 day postoperative following a laparoscopic appendectomy D - A client who is being discharged to home and has a new diagnosis of Crohn's disease Rational A - A client who is newly admitted and is having an episode of status asthmaticus is incorrect. Status asthmaticus is a life-threatening episode of bronchoconstriction that can be unresponsive to usual treatment. Manifestations include wheezing and labored respirations, which can lead to cardiac or respiratory arrest. The range of function for an AP includes assisting with ADLs for stable clients; a client experiencing status asthmaticus is not stable. B - A client who has a history of mild chronic heart failure and had a knee arthroplasty 2 days ago is correct. Heart failure occurs when the heart muscle is unable to pump effectively, resulting in inadequate cardiac output. Knee arthroplasty is the replacement of the knee that requires a short hospital stay followed by physical therapy and rehabilitation. The range of function for an AP includes assisting with ADLs for stable clients; a client who has a history of mild chronic heart failure and is 2 days postoperative is stable. C - A young adult client who is 1 day postoperative following a laparoscopic appendectomy is correct. Clients are usually discharged to home 12 to 24 hr following a laparoscopic appendectomy. The range of function for an AP includes assisting with ADLs for stable clients; a client who is 1 day postoperative following a laparoscopic appendectomy is stable. D - A client who is being discharged to home and has a new diagnosis of Crohn's disease is correct. Manifestations of Crohn's disease include fever, fatigue, diarrhea, and abdominal pain. The range of function for an AP includes assisting with ADLs for stable clients; a client who is being discharged to home is stable. E - A client who has diabetic ketoacidosis and is receiving regular insulin via a continuous IV infusion is incorrect. Diabetic ketoacidosis is an acute, life-threatening condition characterized by hyperglycemia and acidosis. The range of function for an AP includes assisting with ADLs for stable clients; a client who is experiencing diabetic ketoacidosis is not stable.

A nurse is discussing the condition of several clients with an assistive personnel (AP) prior to =routine vital sign measurement. The nurse should plan to measure vital signs for which of the following clients rather than delegating this task to the AP? A - A client who has a history of migraine headaches and reports an aura B - A client who has new onset atrial fibrillation and reports lightheadedness C - A client who requires a Doppler for pedal pulse measurement due to poor circulation D - A client who requires droplet isolation precautions for pneumonia

B - A client who has new onset atrial fibrillation and reports lightheadedness

A nurse is administering medication to a group of clients. For which of the following clients should the nurse expect to administer zolpidem? A - A client who reports nausea and vomiting B - A client who reports difficulty falling asleep C - A client who is wheezing and reports shortness of breath D - A client who reports feeling anxious

B - A client who reports difficulty falling asleep Rational A - The nurse should identify that a client who reports nausea and vomiting requires an antiemetic medication rather than a sedative. B - The nurse should identify that zolpidem is a sedative that is prescribed for clients who have difficulty sleeping. C - The nurse should identify that a client who is wheezing and reports shortness of breath requires a bronchodilator rather than a sedative. D - The nurse should identify that a client who is reporting feeling anxious requires an antianxiety medication rather than a sedative.

A nurse is reveiwing a client's medication administration record (MAR). The nurse should identify that which of the following actions noted in the MAR requires the creation of an incident report? A - A nurse placed a nitroglycerin patch on the client's upper arm. B - A nurse crushed a sustained-release metoprolol tablet and mixed it with yogurt. C - A nurse administered citalopram with a glass of grapefruit juice. D - A nurse injected iron dextran IM using the Z-track method.

B - A nurse crushed a sustained-release metoprolol tablet and mixed it with yogurt. Rational A - Nitroglycerin is a nitrate that causes vasodilation and is used in the treatment of angina. Transdermal nitroglycerin patches are used prophylactically to prevent angina. The nurse should apply the patches to a hairless area of the body, such as the trunk and proximal extremities. B - Metoprolol SR is a selective beta-blocker in a sustained/extended-release form, which should be administered intact. Crushing a sustained- (SR) or extended-release (XL) medication is contraindicated because it will increase the absorption rate of the medication, which increases the risk of adverse effects. This could cause harm to the client and should be documented using an incident report. C - Citalopram is a selective serotonin reuptake inhibitor (SSRI) and is used in the treatment of depression. Grapefruit juice inhibits the action of some medications, including statin medications and some SSRIs, such as sertraline. Clients should avoid grapefruit juice when taking these medications. However, grapefruit juice does not affect the action of citalopram. D - Iron dextran is used to treat iron-deficiency anemia by stimulating the production of red blood cells. When administered intramuscularly, it can be irritating to the tissue and cause staining. Iron dextran should be administered into a deep, large muscle using the Z-track method.

A nurse is monitoring a client who is taking acarbose. Which of the following findings should the nurse identify as an adverse effect of the medication? A - Polyuria B - Abdominal cramps C - Renal insufficiency D - Insomnia

B - Abdominal cramps Rational A - Polyuria is an adverse effect of furosemide. B - Acarbose affects the gastrointestinal system. Therefore, the nurse should monitor the client for abdominal cramping, rumbling bowel sounds, and diarrhea as adverse effects of this medication. C - Long-term and high-dose use of acarbose can cause liver dysfunction, not renal insufficiency. D - Insomnia is an adverse effect of methylphenidate.

A nurse is reinforcing teaching with a client who has alcoholic hepatitis about over-the-counter medications. The nurse should instruct the client to avoid taking which of the following medications? A - Calcium carbonate B - Acetaminophen C - Thiamine D - Docusate

B - Acetaminophen Rational A - Calcium carbonate is an antacid used to treat indigestion and a calcium supplement used to treat and prevent hypocalcemia. It does not cause hepatotoxicity and is not contraindicated in clients who have alcoholic hepatitis. It is contraindicated in clients who have hypercalcemia and renal calculi. B - The nurse should instruct the client that acetaminophen is contraindicated in a client who has alcoholic hepatitis. Acetaminophen is metabolized primarily in the liver and can cause hepatotoxicity in clients who have liver disease. The nurse should instruct the client to check the labels on all over-the-counter medications to prevent ingesting a product containing acetaminophen. C - The nurse should instruct the client to increase intake of foods high in thiamine, such as enriched cereals and fresh vegetables, to prevent thiamine deficiency caused by chronic alcohol use. The provider might prescribe a thiamine supplement for this client to treat thiamine deficiency. D - Docusate is a stool softener used to prevent constipation by increasing the absorption of water into stool. It is contraindicated in clients who have abdominal pain, nausea, or vomiting. It is not contraindicated in clients who have alcoholic hepatitis.

A nurse is reveiwing the laboratory findings of a client who has bbipolar disorder and is taking lithium as a mood stabilizer. The client's lithium level is 0.9 mEq/L. Which of the following actions should the nurse plan to take? A - Contact the provider B - Administer the medication. C - Repeat the laboratory test. D - Monitor for weight loss.

B - Administer the medication. Rational A - Lithium is a mood-stabilizing medication used in the treatment of bipolar disorder. It has a narrow therapeutic range. Outside of this range, the client either does not achieve the therapeutic effects of the medication or can develop lithium toxicity. This client's lithium level of 0.9 mEq/L is within therapeutic range of 0.8 to 1.2 mEq/L and would not require the nurse to contact the provider. B - The nurse should review the client's lithium level and ensure that the client's value is within the therapeutic range of 0.8 to 1.2 mEq/L. Because the client's serum lithium level is within the therapeutic range, the nurse should give the medication as prescribed. Even though this serum level is therapeutic, the nurse should continue to monitor the client for expected adverse effects of lithium, such as fine hand tremor, mild nausea, thirst, and weight gain. C - Lithium is a mood-stabilizing medication with a narrow therapeutic range. Serum levels should be monitored once or twice weekly while dose adjustments are being made and then monthly thereafter. The nurse should ensure that the client's blood is drawn 12 hr after the last dose of lithium to ensure accuracy. If these testing criteria are met and the client's lithium level is within the therapeutic range of 0.8 to 1.2 mEq/L, it is not necessary for the nurse to repeat the test. D - Lithium is a mood-stabilizing medication used in the treatment of bipolar disorder. It can lead to weight gain and is associated with adverse effects including fine tremors, thirst, and nausea. Therefore, the nurse should monitor the client for weight gain, not weight loss.

A charge nurse is evaluating the documentation of care for four client by a newly licensed nurse. Which of the following entries requires interventions by the charge nurse? A - Client medicated with morphine 30 mg PO for report of right shoulder pain rated 7 on a scale of 0 to 10. B - Administered 10.0 u of insulin SQ to client for elevated glucose level. C - Reinforced to client to turn, cough, and deep breathe every 2 hr while awake. Client verbalized understanding. D - Reported client's oral temperature 39.7° C (103.5° F) to provider.

B - Administered 10.0 u of insulin SQ to client for elevated glucose level.

A nurse is reinforcing teaching about formula feeding a newborn with a group of new parents. Which of the following instructions should the nurse include? A - Give approximately 240 mL (8 oz) per feeding. B - Allow 20 to 30 min for feedings. C - Ensure that the newborn empties the bottle. D - Wait to burp the newborn until the end of the feeding.

B - Allow 20 to 30 min for feedings.

A nurse is supervising a group of clients in the dining area. One of the clients throws a meal tray on the floor and begins to yell obscenities. Which of the following actions should the nurse take first? A - Administer haloperidol IM to the client. B - Allow the client to choose a quiet place away from others. C - Move the client to a seclusion room with continuous observation. D - Place the client in wrist restraints.

B - Allow the client to choose a quiet place away from others. Rational A - The nurse might need to administer haloperidol to calm the client. However, the nurse should initiate a less restrictive intervention first. Additionally, oral medications should be used if appropriate. Administering haloperidol or another psychotropic medication that is not part of the client's routine medication regime for the purpose of calming the client is considered a chemical restraint. B - When providing client care, the nurse should use the least restrictive intervention first. The nurse should first ask the client to choose a quiet area to go to allow the client to calm down and to prevent harm to others. The nurse should allow the client to verbalize any concerns and should give the client choices for a peaceful resolution to the situation. Providing choices can decrease any sense of powerlessness the client might be experiencing. C - The nurse might need to place the client in seclusion to prevent harm to others and allow the client to calm down. Clients require seclusion when exhibiting behavior that could result in self-harm or injury to other clients or staff. However, the nurse should initiate a less restrictive intervention first. D - The nurse might need to place the client in restraints to prevent harm to others and allow the client to calm down. Use of a therapeutic hold is also considered a form of restraint because it prevents or restricts physical movement. The nurse should initiate a less restrictive intervention first.

A nurse is reinforcing dietary teaching about a low-sodium diet with the parents of a child who is recovering from acute glomerulonephritis. Which of the following food choices by the parents indicates an understanding of the teaching? A - Pretzels B - Apples C - Canned corn D - Peanut butter

B - Apples

A nurse is preparing to administer ophthalmic drops to a child. Which of the following actions should the nurse take? A - Position the child with his head flexed while administering the medication. B - Apply pressure to the lacrimal punctum for 1 min following administration. C -Hold the dropper 5 cm (2 in) above the eye to administer the medication. D - Wipe the excess medication toward the inner canthus with a cotton swab.

B - Apply pressure to the lacrimal punctum for 1 min following administration.

A nurse is caring for a client who is experiencing alcohol withdrawal with manifestations of visual hallucinations. Which of the following actions should the nurse take? A - Apply wrist restraints to the client. B - Arrange for continuous observation of the client. C - Tell the client that the hallucinations are not real. D - Play loud music for the client.

B - Arrange for continuous observation of the client. Rational A - The nurse should not apply wrist restraints for a client who is experiencing visual hallucinations from alcohol withdrawal because this can increase the client's confusion. B - Clients who are experiencing visual hallucinations from alcohol are at risk for injury. Therefore, the nurse should arrange for continuous observation of the client to ensure client safety. C - The nurse should not argue or disagree with the client's hallucinations, but instead provide reassurance and reinforce reality. D - The nurse should keep the noise level low to minimize stimulation and prevent disorientation.

A nurse is preparing to administer a narcotic medication to a client who reports experiencing pain. Which of the following actions should the nurse plan to take? A - Place any wasted portion of the narcotic in the sharps container. B - Ask a second nurse to witness the discarding of any part of the narcotic. C - Remove the medication before counting the remaining narcotics. D - Report a narcotic discrepancy to the provider.

B - Ask a second nurse to witness the discarding of any part of the narcotic. Rational A - The nurse should plan to dispose of a narcotic, or any other medication, according to agency policy. The nurse should never dispose of or waste parts of narcotics or medications into the sharps container. B - The nurse should ask a second nurse to witness the discarding or wasting of parts of any narcotic. Both nurses should sign the control inventory form. This verifies that the narcotics were disposed of in a manner that adheres to the Controlled Substances Act (CSA). The narcotics disposal process will be directed by facility policy. C - The nurse should verify the number of narcotics with the inventory record before removing any of the narcotics. If there is a discrepancy, the nurse must investigate and correct it before removing the narcotic dose. D - The nurse should report a narcotic discrepancy immediately to the nurse manager, nurse supervisor, and pharmacy and should follow agency policy to account for the discrepancy.

A nurse is planning to administer digoxin to a client and auscultates an apical pulse of 52/min. Which of the following actions should the nurse take? A - Administer the medication as prescribed. B - Ask another nurse to check the apical pulse. C - Recheck the client's pulse in 2 hr. D - Report the finding to the facility pharmacist.

B - Ask another nurse to check the apical pulse. Rational A - The nurse should withhold the administration of digoxin for an apical pulse below 60/min. The nurse should have a second nurse confirm the apical pulse rate and notify the charge nurse or provider if bradycardia is confirmed. B - The nurse should ask a second nurse to verify the finding of an apical pulse less than 60/min. If bradycardia is confirmed, the nurse should notify the charge nurse or provider. C - The nurse should withhold the medication and have a second nurse confirm the pulse rate. If bradycardia is confirmed, the nurse should notify the charge nurse or provider and recheck the pulse rate within 1 hr. D - The nurse should have a second nurse confirm the finding, then report the pulse rate to the charge nurse or provider.

A nurse is collecting data from a client who was admitted with alcohol use disorder. Which of the following actions is the nurse's priority? A - Assist the client to identify negative effects of chemical dependency. B - Ask the client about thoughts of self-harm. C - Encourage the client to join a support group in the community. D - Help the client to focus on efforts to maintain sobriety.

B - Ask the client about thoughts of self-harm. Rational A - The nurse should assist the client in recovery by helping to identify the negative effects of alcohol use disorder by discussing the effect alcohol has had on the client's physical, social, and psychological well-being, as well as its effect on the client's relationships with others. Assisting the client to identify the effect of chemical dependency on their life and others can help the client in their recovery. Identifying that the use of alcohol is not an effective coping mechanism will also help with the client's recovery. However, another action is the priority. B - The greatest risk for injury for a client who has a history of alcohol use disorder and is experiencing withdrawal is self-harm. The priority action for the nurse to take is to determine whether the client has thoughts of self-harm or suicide. C - The nurse should encourage the client to attend a 12-step program, which facilitates a group setting made up of peers. Peer feedback can be more easily accepted by the client since the peers have similar experiences with substance use disorder. Peers can also provide motivation and encouragement to the client. However, another action is the priority. D - The nurse should provide positive reinforcement to the client for efforts made during the recovery process to assist the client in remaining focused on achieving and maintaining sobriety. However, another action is the priority.

A nurse is caring for a client who is scheduled for a cardiac catheterization with coronary angiography. The client has a history of coronary artery disease and takes cardiac medications. Which of the following actions should the nurse take? A - Instruct the client to take nothing by mouth 12 hr before the procedure. B - Ask the client if they have a history of a seafood allergy. C - Withhold the client's cardiac medications before the procedure. D - Inform the client that an urge to void will occur following the injection of the contrast dye.

B - Ask the client if they have a history of a seafood allergy. Rational A - The nurse should instruct the client to not drink or eat at least 8 hr before the diagnostic procedure to reduce the risk of nausea and vomiting during the procedure. B - The nurse should check the client for allergies and notify the provider. Identifying allergies to seafood, iodine, and other radiographic dyes can reduce the risk for an anaphylactic reaction to the contrast dye. C - The nurse should administer prescribed cardiac medication with a small sip of water before the procedure to decrease the chance of developing a cardiac dysrhythmia. The provider will determine which of the prescribed cardiac medications to administer in the pre-procedure prescriptions. D - An urge to void is not expected. The nurse should inform the client that following injection of the contrast dye, a warm flushed feeling occurs and a metallic taste in the mouth can develop. The nurse should also inform the client that it is common to have an irregular or rapid pulse during the procedure.

A nurse is collecting data from a client who is 4 hr postpartum and determines that the client's fundus is displaced to the right of the umbilicus. Which of the following actions should the nurse take? A - Place the client in the left-lateral position. B - Assist the client to the bathroom. C - Instruct the client to perform Kegel exercises. D - Encourage the client to breastfeed.

B - Assist the client to the bathroom. Rational A - The nurse should not place the client in the left-lateral position, as this will not assist in repositioning the uterus to midline. A displaced fundus is a sign of a full bladder, which prevents contraction of the uterus and can lead to hemorrhage. B - A full bladder causes the uterus to rise above the level of the umbilicus and deviate to one side. A full bladder also prevents contraction of the uterus, which increases the risk for hemorrhage. The nurse should assist the client to the bathroom to promote voiding. C - Kegel exercises strengthen perineal muscles and decrease the risk for incontinence. A displaced fundus is a sign of a full bladder, which can lead to hemorrhage. Kegel exercises will not assist with emptying the client's bladder. D - A displaced fundus is a sign of a full bladder, which prevents contraction of the uterus and can lead to hemorrhage. While breastfeeding can induce uterine contractions, the contractions will not be effective if the bladder is full. The nurse should assist the client to the bathroom to void.

A nurse in an long-term care facility is assisting with teaching a group of nurses about client rights. Which of the following information should the nurse include in the teaching? A - Clients should receive a 45-day notice of being transferred or discharged to another facility. B - Clients can request to review their own personal medical records. C - Clients have the legal right to refuse restraints. D - Clients in long-term care do not have the right to manage their own financial affairs or choose their own provider.

B - Clients can request to review their own personal medical records. Rational A - The client has the right for the long-term care facility's personnel to provide 30-day notice of being transferred or discharged to another facility due to financial reasons, when care is no longer needed, or for the client's welfare. B - Clients have the right to review their own medical records, participate in their own care, and expect adequate and appropriate care from the facility's personnel. C - Clients have the right to participate in their own care and to refuse treatment; however, if they are a harm to them self or others, they do not have the right to refuse seclusion or restraints. D - Clients in a long-term care facility have the right to make independent choices, which include managing their own financial affairs and choosing their own provider, as long as they are mentally competent.

A nurse in a long-term care facility is contributing to the plan of care for a client who has dementia and is increasingly anxious and confused. Which of the following actions should the nurse plan to implement? Select all A - Elevate the height of the bed. B - Assist the client with ADLs. C - Establish a toileting schedule for the client. D - Turn off the lights in the client's room at night. E - Reorient the client to the environment frequently.

B - Assist the client with ADLs. C - Establish a toileting schedule for the client. E - Reorient the client to the environment frequently. Rational A - Elevate the height of the bed is incorrect. The client should be able to get in and out of bed safely. Therefore, the nurse should not elevate the height of the bed. B - Assist the client with ADLs is correct. Clients who are experiencing increased anxiety and confusion can require assistance with bathing and dressing. Therefore, the nurse should assist this client with completion of ADLs. C - Establish a toileting schedule for the client is correct. The nurse should establish a toileting schedule for the client because this can reduce the risk of a client attempting to get out of bed, thereby decreasing the risk for falls. D - Turn off the lights in the client's room at night is incorrect. The client's room should be well lit to encourage effective client perception of the environment. This can help to decrease the client's anxiety and confusion. E - Reorient the client to the environment frequently is correct. The nurse should frequently reorient the client to their environment because this can help to decrease the client's anxiety and confusion.

A nurse is caring for a client who is 36 hr postoperative following a thyroidectomy and reports severe lower extremity muscle spasms. Which of the following actions should the nurse take? A - Check the pedal pulses. B - Attempt to elicit Chvostek's sign. C - Request a prescription for a muscle relaxant. D - Administer an oral potassium supplement.

B - Attempt to elicit Chvostek's sign. Rational A - A client who has had a thyroidectomy is at risk of hypocalcemia, or a low calcium level, due to possible disruption of the parathyroid gland during surgery. Low calcium levels can be manifested as numbness and tingling of the fingers and around the mouth, muscle spasms (particularly of the hands and feet), and hyperactive reflexes. The nurse should review the client's calcium level. The client's muscle spasms are likely caused by hypocalcemia. There is no indication of a problem with this client's circulation. Therefore, it is not necessary to check the pedal pulses until the routine time. B - A client who has had a thyroidectomy is at risk of hypocalcemia, or a low calcium level, due to disruption of the parathyroid gland during surgery. Manifestations of hypocalcemia can include numbness and tingling of the fingers and around the mouth, muscle spasms (particularly of the hands and feet), and hyperactive reflexes. Positive Chvostek's and Trousseau's signs are indications of hypocalcemia. The nurse should notify the provider and have calcium gluconate available for IV administration. C - A client who has had a thyroidectomy is at risk of hypocalcemia, or a low calcium level, due to disruption of the parathyroid gland during surgery. If the client has a low calcium level, replacement with oral or IV calcium is indicated. Once the calcium level is restored, the muscle spasms should stop. The nurse should not treat the manifestations without first correcting the hypocalcemia. D - A client who has had a thyroidectomy is at risk of hypocalcemia, or a low calcium level, due to disruption of the parathyroid gland during surgery. Manifestations of hypocalcemia can include numbness and tingling of the fingers and around the mouth, muscle spasms (particularly of the hands and feet), and hyperactive reflexes. The client requires treatment with calcium gluconate via IV administration. Clients who have low potassium levels often have accompanied leg cramps. Hypokalemia is not a risk associated with a thyroidectomy. Therefore, this is not the likely cause of the client's muscle cramps.

A nurse is reinforcing teaching with a client who has multiple sclerosis and a new prescription for baclofen. Which of the following instructions should the nurse include in the teaching? A - Consume a low-purine diet. B - Avoid stopping this medication suddenly. C - Use chamomile tea to alleviate insomnia. D - Take this medication on an empty stomach.

B - Avoid stopping this medication suddenly.

A nurse is assisting with scoliosis screenings for a group of school-age children. The nurse should place the students in which of the following positions during the screening? A - Clasping hands while arms are raised above the head B - Bending forward with back parallel to the floor C - Standing with feet shoulder-width apart D - Bending knees while placing hands on hips

B - Bending forward with back parallel to the floor

A nurse in a long-term care facility enters a client's room and finds the client lying on the floor. Which of the following actions should the nurse take first? A - Notify the charge nurse and the client's provider. B - Check the client for injuries. C - Request help to assist the client back to bed. D - Complete an incident report.

B - Check the client for injuries.

A nurse is caring for a client who experienced spontaneous rupture of membranes and is in active labor. Which of the following actions should the nurse take? A - Change the client's position every 2 hr. B - Check the client's temperature every 2 hr. C - Remind the client to bear down with each contraction. D - Encourage the client to empty the bladder every 4 hr.

B - Check the client's temperature every 2 hr. Rational A - The nurse should reposition the client every 30 to 60 min during the labor process to relieve fatigue, increase comfort, and improve circulation. The best position for the client to assume is the lateral position because it promotes optimal uteroplacental blood flow and increases fetal oxygen saturation. B - The nurse should check the client's temperature and the color and odor of the amniotic fluid every 2 hr to identify early manifestations of infection. After the membranes have ruptured, micro-organisms can ascend into the amniotic sac and place the client and fetus at risk for infection. C - The nurse should not encourage the client to push or bear down until the cervix is completely dilated to 10 cm. The client is in the active phase of the first stage of labor, and their cervix will be between 4 and 7 cm dilated. D - The nurse should encourage a client who is in labor to empty their bladder every 2 hr. Bladder distention can impede the descent of the fetus and slow the progression of labor.

A nurse is collecting data from a client who experienced a brief loss of consciousness after falling and hitting their head 12 hr ago. Which of the following is the priority findings to report to the charge nurse? A - Large bruise on forehead B - Clear fluid present in left ear C - Pupils are 4 mm and reactive to light D - Blood pressure 105/64 mm Hg

B - Clear fluid present in left ear Rational A - A bruised area on the head is non-urgent because it is an expected finding following a head injury from a fall. The nurse should monitor the client for manifestations of increased intracranial pressure including decreased level of consciousness, headache, nausea and vomiting, changes in speech and sensorimotor status, pupillary changes, severe hypertension, widening pulse pressure, bradycardia, and abnormal posturing (decerebrate or decorticate). B - When using the urgent vs non-urgent approach to client care, the nurse should determine that the priority finding to report to the charge nurse is the presence of clear fluid in left ear. Clear fluid in the ear canal or nose can indicate a tear in the meninges, which would allow cerebrospinal fluid (CSF) to escape. Leaking CSF increases the client's risk for meningitis. C - A pupil size of 4 mm is non-urgent because it is within in the expected reference range of 3 to 7 mm. The nurse should expect the client's pupils to react bilaterally to light stimulation and accommodation. Pupil changes are a late finding of increased intracranial pressure. The nurse should report if there is a change in pupil size or reactivity. D - A blood pressure of 105/64 mm Hg is non-urgent because it is normotensive. Increased intracranial pressure from a head injury can lead to increases in the systolic pressure while the diastolic remains the same or decreases. This difference between the systolic and diastolic readings is the pulse pressure. The nurse should report a widened pulse pressure to the provider immediately. Clients who have Cushing's triad (a late finding of increased intracranial pressure) will have bradycardia, severe hypertension, and a widened pulse pressure (systolic minus diastolic).

A home health nurse is reinforcing teaching about the effects of carbon monoxide poisoning. Which of the following manifestations should the nurse include? A - Diarrhea B - Ringing in the ears C - Headaches D - Irritability

C - Headaches

A nurse is assisting with the plan of care for a client who has herpes simplex. Which of the following isolation precautions should the nurse plan to implement? A - Droplet B - Contact C - Airborne D - Protective environment

B - Contact Rational A - The nurse should plan to implement droplet precautions for clients who have infections spread by droplet nuclei larger than 5 mm, including mumps, pertussis, and mycoplasma pneumonia. The nurse should place the client in a private room and should wear a mask when within 1 m (3.3 feet) of the client. B - Herpes simplex is a viral infection of the mouth, skin, saliva, and genitalia. It is spread by direct or indirect contact with lesions or bodily fluids. The nurse should plan to implement contact precautions using hand hygiene, gloves, and a gown. The nurse should place the client in a private room if possible. However, the nurse can place the client in a room with another client who has the same infection. C - The nurse should plan to implement airborne precautions for clients who have infections spread through droplet nuclei smaller than 5 mm, including measles, chickenpox (varicella), and pulmonary or laryngeal tuberculosis. The nurse should place the client in a private room with negative-pressure airflow of a minimum of 6 to 12 exchanges per hour. The nurse should wear a mask or N95 respirator mask if the client has tuberculosis. D - The nurse should plan to implement a protective environment for clients who are immunocompromised, such as clients who have cancer or have undergone a transplant. The nurse should place the client in a private room with positive-pressure airflow and a HEPA filter.

A nurse is collecting data from a newborn who has hypoglycemia. Which of the following manifestations should the nurse expect? A - Increased muscle tone B - Decreased temperature C - Transient nystagmus D - Abdominal distention

B - Decreased temperature Rational A - Increased muscle tone is a manifestation of a central nervous system disorder. B - The nurse should identify that decreased temperature is a manifestation of hypoglycemia. Other manifestations include decreased muscle tone, abnormal cry, and jitteriness. C - Transient nystagmus, or repetitive uncontrolled movement of the eyes, is an expected finding in a newborn and is not related to blood glucose level. D - Abdominal distention is a manifestation of a bowel obstruction.

A nurse is collecting data from a client who has bulimia nervosa. Which of the following manifestations should the nurse expect? A - Amenorrhea B - Dental caries C - Lanugo D - Yellow skin

B - Dental caries

A charge nurse is a member of the resource management team for a skilled care facility. Which of the following actions should the charge nurse implement to ensure the facility is providing cost-effective wound care for clients? A - Encourage nursing staff to read research articles about advancements in wound care. B - Develop a spreadsheet to prepare a budget for wound care supplies. C - Arrange for nursing staff to attend a wound care conference through teleconferencing. D - Prepare a slide presentation about recognizing wound infections.

B - Develop a spreadsheet to prepare a budget for wound care supplies.

A nurse is assisting with the planning of an in-service for a group of staff nurses about establishing a sterile field. Which of the following information should the nurse include? A - Pour sterile liquids while holding the bottle with the label facing the table. B - Discard any object that touches the outer 2.5 cm (1 in) border of the sterile field. C - Choose a work area 5 cm (2 in) below the level of the waist. D - Pull the outer flap of a sterile package toward the body.

B - Discard any object that touches the outer 2.5 cm (1 in) border of the sterile field. Rational A - The nurse should instruct staff nurses to pour liquids onto a sterile field by holding the label of the bottle in the palm of the hand. This prevents the liquid from running onto the label and making it unreadable. B - The nurse should instruct staff nurses that the outer 2.5 cm (1 in) border of a sterile field is considered unsterile. If an object comes into contact with this border, the nurse should discard it since it is no longer sterile. C - The nurse should instruct staff nurses to choose a work area that is at or slightly above waist level. This decreases the chance of contaminating the sterile field. The nurse should consider any object below waist level as contaminated. D - The nurse should instruct the staff nurses to open sterile packages by pulling the outer flap away from the body. This prevents the nurse from later having to reach across the package to open the last flap, which could contaminate the package.

A nurse is collecting data from a client who has hypokalemia. Which of the following findings should the nurse identify as the priority? A - Muscle weakness B - Dysrhythmia C - Abdominal pain D - Lethargy

B - Dysrhythmia

A nurse working in a provider's office is reinforcing teaching with a client who has a Grade I (mild) ankle sprain. Which of the following instructions should the nurse include? Select all A - Immobilize the ankle for 4 to 6 weeks. B - Elevate the ankle above the level of the heart. C - Take a muscle relaxant as needed. D - Wrap the ankle with an elasticized compression bandage. E - Apply an intermittent cold compress to the ankle for the first 24 to 48 hr.

B - Elevate the ankle above the level of the heart. D - Wrap the ankle with an elasticized compression bandage. E - Apply an intermittent cold compress to the ankle for the first 24 to 48 hr. Rational A - Immobilize the ankle for 4 to 6 weeks is incorrect. The client should rest the ankle, but immobilization even for a few days can lead to muscle wasting and cause permanent damage. B - Elevate the ankle above the level of the heart is correct. The client should elevate the ankle to promote venous return and decrease edema. C - Take a muscle relaxant as needed is incorrect. The nurse should reinforce teaching with the client that use of an NSAID medication such as ibuprofen or naproxen can help decrease swelling of the injured site and relieve discomfort. Clients who have a muscle strain should receive muscle relaxants to relieve muscle spasms. D - Wrap the ankle with an elasticized compression bandage is correct. Applying a compression bandage for a few days following the injury reduces swelling and provides joint support. If the client reports throbbing, discomfort, or a tight bandage, the nurse should remove and rewrap the bandage with less stretch. E - Apply an intermittent cold compress to the ankle for the first 24 to 48 hr is correct. The client should apply ice to the ankle for the first 24 to 48 hr to decrease edema and pain. The nurse should reinforce to the client not to apply ice directly to the skin or leave ice on the ankle for more than 20 min at a time. Longer exposure can damage the skin and even potentially result in frostbite.

A charge nurse is asked by two staff nurses to assist in resolving a conflict about holiday scheduling. Which of the following actions should the nurse take? A - Negotiate with the two staff nurses at the nurses' station. B - Encourage each staff nurse to give up something as part of the negotiation. C - Explain to the staff nurses that the holiday schedule is non-negotiable. D - Resolve the conflict with a win-yield outcome.

B - Encourage each staff nurse to give up something as part of the negotiation

A nurse is caring for an adult client who has visible injuries as a result of intimate partner violence. Which of the following actions should the nurse take? A - Insist that the client report the incident to the authorities before beginning treatment. B - Encourage the client to develop a safety plan. C - Recommend that the partner remain in the room during the interview with the client. D - Advise the client to obtain an order of protection from the court.

B - Encourage the client to develop a safety plan.

A nurse is preparing to witness a client who is scheduled for surgery sign an informed consent form. Which of the following tasks is the nurse's responsibility? A - Inform the client of any complications that might occur. B - Ensure that the client is competent to sign the consent. C - Explain the procedure to the client. D - Notify the provider when the client is ready to sign the consent.

B - Ensure that the client is competent to sign the consent. Rational A - It is the responsibility of the provider to inform the client of the benefits, risks, and possible complications of the procedure so the client can make an informed decision. This is not the responsibility of the nurse. B - The nurse should ensure that the client is competent, understands the procedure, and is not under the influence of any medications prior to the client giving informed consent. C - It is the responsibility of the provider to explain the procedure to the client, including the benefits, risks, and possible complications of the procedure so the client can make an informed decision. This is not the responsibility of the nurse. D - The nurse should notify the provider only when the client has questions about the procedure. The provider has the responsibility to inform the client of the benefits, risks, and possible complications of the procedure so the client can make an informed decision. There is no need to notify the provider if the nurse confirms that the client understands this information and is ready to sign the consent form.

A nurse is assisting with the care of a client who was admitted with deep-vein thrombosis. The client has decided to leave against medical advice. Which of the following actions should the nurse take? A - Administer a PRN sedative. B - Explain to the client the risk involved in leaving the hospital. C - Notify the client's next of kin. D - Assign an assistive personnel (AP) to provide one-to-one observation of the client.

B - Explain to the client the risk involved in leaving the hospital.

A nurse is assisting with updating the facility response plan regarding natural disasters. With which of the following agencies should the nurse plan to collaborate? A - World Health Organization (WHO) B - Federal Emergency Management Agency (FEMA) C - Centers for Disease Control and Prevention (CDC) D - Centers for Medicaid and Medicare Services (CMS)

B - Federal Emergency Management Agency (FEMA) Rational A - The WHO provides leadership and guidance for global health issues. However, the nurse should recognize that the WHO facilitates programs to reduce risks associated with disasters on community or national levels, not for individual medical facilities. B - FEMA is the federal agency that plans for management of response and coordinates response to emergency situations including natural disasters. Medical facilities plan response by organizing teams that follow FEMA's incident command system. This system provides a structure and chain of command for managing emergencies. C - The CDC focuses on occurrence and prevention of diseases in the U.S. The CDC has a strategic plan in place for mass casualty incidents that are the result of radioactive, biologic, or biochemical acts of terrorism. D - The CMS is a federal government agency that oversees the Medicaid and Medicare systems for payment of health care services. It does not provide assistance for natural disasters.

A nurse is caring for a client who. is at 20 weeks of gestation and is in the clinic for a routine prenatal visit. Which of the following findings in the data from the client's medical record should the nurse report to the provider? (Exhibbit) A - Weight B - Fundal Height C - Fetal heart rate D - blood pressure

B - Fundal Height Rational A - A client in the second trimester of pregnancy should gain approximately 0.4 kg (0.9 lb) per week. Therefore, a weight gain of 1.3 kg (2.9 lb) in 4 weeks is an expected finding. b - The height of the fundus in centimeters at 20 weeks of gestation is approximately the same as the number of weeks of gestation plus or minus 2 cm. Therefore, a fundal height of 25 cm is greater than the expected finding for 20 weeks of gestation. C - A fetal heart rate of 160/min at 20 weeks of gestation is within the expected reference range of 110 to 160/min. D - A maternal blood pressure of 130/80 mm Hg is below the reportable value of 140/90 mm Hg.

A nurse is collecting data form a child who has iron deficiency anemia. Which of the following data signifies that adherence to ferrous sulfate therapy has occurred? A - Occasional vomiting and nausea B - Green, tarry stools C - Tolerates milk D - Weight gain

B - Green, tarry stools

A nurse is collecting data from a client who is scheduled to receive digoxin. For which of the following findings should the nurse withhold the digoxin dose and notify the provider? A - BP 154/90 mm Hg B - Heart rate 52/min C - Digoxin level 0.9 ng/mL D - Potassium 4.2 mEq/L

B - Heart rate 52/min Rational A - Digoxin has hypotensive effects as a result of cardiac output improving and the contraction of arterioles and veins decreasing. A systolic pressure of 140 mm Hg or greater and a diastolic pressure of 90 mm Hg or greater is indicative of hypertension. The nurse should not withhold this medication for a blood pressure of 154/90 mm Hg. B - The expected reference range for the heart rate of an adult is 60 to 100/min. At rates lower than this, cardiac output is decreased and can result in syncope. Additionally, perfusion of the kidneys is decreased. The primary action of digoxin is to increase the force of ventricular contraction, thereby increasing cardiac output. This causes a decrease in sympathetic tone, which leads to a decrease in heart rate. If the client's heart rate is less than 60/min, or if the nurse notes any changes in rhythm, the nurse should withhold the medication and inform the provider. C - This digoxin level is within the expected reference range of 0.8 to 2.0 ng/mL. The nurse should not withhold this medication due to this laboratory result. Digoxin has a very narrow therapeutic range. Elevated levels produce digitalis toxicity and include symptoms such as abdominal pain, weakness, diplopia, halos seen around objects, and dysrhythmias. D - This potassium level is within the expected reference range of 3.5 to 5.0 mEq/L. The nurse should not withhold this medication due to this laboratory result. However, the nurse should monitor potassium levels because a client who takes digoxin is at risk for dysrhythmias, especially if the client is potassium-depleted due to taking diuretics.

A nurse is contributing to the plan of care for a client who is postoperative following a total knee arthroplasty. The client is using a continuous passive motion (CPM) machine. Which of the following interventions should the nurse recommend for the plan of care? A - Store the CPM machine on the floor when it is not in use. B - Keep a sheepskin pad between the client's extremity and the CPM. C - Check the cycle and range-of-motion settings at least every 12 hr. D - Align the frame joint of the CPM with the middle of the client's calf.

B - Keep a sheepskin pad between the client's extremity and the CPM.

A nurse is caring for a client who has a new diagnosis of liver failure. The client states, "The doctor told me about the typical treatment for this, but I'm not sure I want to go through all of that." Which of the following actions should the nurse take first? A - Discuss the role of a health care proxy in making treatment decisions. B - Help the client list their concerns about treatment. C - Provide the client with a brochure about liver disease. D - Give the client information about support groups for individuals who have liver disease.

B - Help the client list their concerns about treatment. Rational A - The nurse should discuss the role of a health care proxy in making treatment decisions to promote client advocacy when the client is no longer able to make decisions. However, there is another action the nurse should take first. B - The first action the nurse should take using the nursing process is to collect data from the client. By helping the client make a list of concerns about treatment, the nurse can assist with planning interventions to address the client's concerns and promote the client's ability to self-advocate. C - The nurse should provide the client with a brochure about liver disease to inform the client about what to expect following a new diagnosis. However, there is another action the nurse should take first. D - The nurse should give the client information about support groups for individuals who have liver disease to promote the client's ability to cope with the new diagnosis. However, there is another action the nurse should take first.

A nurse is participating in a health fair for older adult clients. Which of the following immunization should the nurse recommend for this age group? A - Meningococcal B - Herpes zoster C - Human papillomavirus (HPV) D - Measles, mumps, and rubella (MMR)

B - Herpes zoster

A nurse is reveiwing the laboratory results of a client who has chronic kidney failure and is receiving epoetin alfa. The nurse should identify that which of the following laboratory values indicates the treatment is effective? A- BUN 40 mg/dL B - Hgb 11 g/dL C - Urine specific gravity 1.035 D - Blood glucose 105 mg/dL

B - Hgb 11 g/dL

A nurse is collecting data form a 55-year-old female client who reports vagincal dryness and hot flashes. The client is interested in trying hormone replacement therapy (HRT). Which of the following should the nurse recognize as a contraindication to HRT? A - Five-year history of menopause manifestations B - History of treatment for blood clots C - Topiramate use for migraine headaches D - Increased serum cholesterol levels

B - History of treatment for blood clots Rational A - The nurse should identify that manifestations of menopause can last for 10 years or more and HRT is not contraindicated for a client whose menopause manifestations began 5 years ago. B - Estrogen increases the risk of blood clots. Therefore, a woman who has a history of blood clots should not receive HRT. C - The nurse should identify that the use of topiramate to treat migraine headaches can cause decreased absorption of estrogen when used as a contraceptive. However, topiramate is not a contraindication to HRT. D - The nurse should identify that one of the benefits of HRT is a decrease in LDL and an increase in HDL levels. Therefore, HRT is not contraindicated for a client who has increased serum cholesterol levels.

A nurse is caring for a client who has received methylergonovine. Which of the following should the nurse identify and document as an adverse effect of the medication? A - Hyperglycemia B - Hypertension C - Urinary retention D - Hyporeflexia

B - Hypertension

A nurse is monitoring an assistive personnel (AP) who is calculating I&O for a postoperative client. The client has a Jackson-Pratt drain and an indwelling urinary catheter. The nurse should recognize that the client's output is calculated and recorded correctly when the AP performs which of the following actions? A - Includes 0.9% sodium chloride used to irrigate the catheter in the calculated output. B - Includes emesis and wound drainage in the total recorded output. C - Measures the urine using the markings on the drainage bag. D - Documents drainage in cubic centimeters (cc) on the intake and output form.

B - Includes emesis and wound drainage in the total recorded output.

A nurse is assisting with the care of a client who is receiving carboprost tromethamine. Which of the following manifestations is an expected outcome of this medication? A - Decrease in maternal blood pressure B - Increase in uterine contractions C - Increase in fetal heart rate D - Decrease in fetal antibody response

B - Increase in uterine contractions Rational A - Carboprost tromethamine is a prostaglandin derivative and can lead to an increase in maternal blood pressure. B - A client can receive carboprost tromethamine, a prostaglandin derivative, to treat postpartum hemorrhage by increasing uterine contractions. C - A client should only receive carboprost tromethamine during the postpartum period or following a miscarriage. D - A client should only receive carboprost tromethamine during the postpartum period or following a miscarriage.

A nurse is preparing a client for a routine gynecological examination. Which of the following actions should the nurse take? A - Assist the client into a dorsal recumbent position prior to the examination. B - Instruct the client to empty their bladder prior to the examination. C - Advise the client to tighten their abdominal muscles during the internal examination. D - Provide sterile gloves for the provider to wear during the examination.

B - Instruct the client to empty their bladder prior to the examination. Rational A - The nurse should assist the client into a lithotomy position for a gynecological examination. In this position, the client's feet are in stirrups with the buttocks at the end of the examination table, allowing the provider to examine the client's genitalia and genital tract. The nurse should place clients in a dorsal recumbent position for examination of the thorax or abdomen. B - The nurse should instruct the client to empty their bladder prior to a gynecological examination. This action allows the provider to perform a more thorough palpation of the client's uterus and ovaries and promotes client comfort during the examination. During the gynecological examination, the provider can perform a vaginal examination and can assess the client's reproductive organs and external genitalia for abnormalities. Instructing the client to void also provides a urine specimen for urinalysis, which is often prescribed as part of the examination. C - The nurse should instruct the client to relax their abdominal muscles during the examination. This relaxation promotes comfort during the examination. The nurse should have the client place their arms at the side or across the chest and take deep breaths during the examination. These actions help to prevent the client's abdominal muscles from tightening. D - Prior to the exam, the nurse should prepare supplies needed by the provider. Supplies include clean gloves, a light source, vaginal speculum, and supplies for collection of cytological specimens. Clean gloves provide protection for the provider from contact with bodily fluids of the genitalia and genital tract. This examination is a clean procedure. Sterile gloves are not necessary.

A nurse is contributing to the plan of care for a client who has a new prescription for nystatin suspension for oral candidiasis. Which of the following interventions should the nurse include in the plan? A - Use a commercial mouthwash before taking the medication. B - Instruct the client to swish the medication in her mouth. C - Discontinue the medication as soon as the lesions are healed. D - Combine the medication with applesauce.

B - Instruct the client to swish the medication in her mouth.

A nurse is caring for a client who is 8 hr postoperative following a transurethral resection of the prostate (TURP) and has a continuous bladder irrigation system in place. Which of the following actions should the nurse take? select all A - Add the amount of bladder irrigation to the total output. B - Keep the tip of the tubing sterile when connecting it to the irrigation solution. C - Ensure the drainage tubing is patent. D - Contact the surgeon if the client reports a continual need to void. E - Notify the surgeon if the urine is bright red in appearance or has large clots.

B - Keep the tip of the tubing sterile when connecting it to the irrigation solution. C - Ensure the drainage tubing is patent. E - Notify the surgeon if the urine is bright red in appearance or has large clots. Rational A - Add the amount of bladder irrigation to the total output is incorrect. The nurse should subtract the amount of bladder irrigation solution from the total urine output amount. B - Keep the tip of the tubing sterile when connecting it to the irrigation solution is correct. Using sterile technique decreases the risk of contamination with micro-organisms and reduces the possibility of infection. The nurse should observe the client closely for manifestations of infection, such as fever and an elevated WBC count. C - Ensure the drainage tubing is patent is correct. The nurse should make sure the outflow drainage tubing of the three-way catheter system is not obstructed, kinked, or clamped. This will prevent excess accumulation of urine and irrigant in the bladder, which can cause distention and possible injury. D - Contact the surgeon if the client reports a continual need to void is incorrect. Following a TURP, the client will have a large catheter that is pulled taut and secured to the client's leg. This provides traction that holds the catheter balloon against the internal sphincter of the bladder. As a result, the client might experience the sensation of a continual need to void. The nurse should inform the client that the urge to void is expected. However, the client should not attempt to void around the catheter because this can cause bladder spasms, which can be painful and initiate bleeding. E - Notify the surgeon if the urine is bright red in appearance or has large clots is correct. It is important to document the characteristics of the drainage. It is expected to see blood with a few small blood clots. However, urine that is bright red, ketchup-like, or has large clots is an indication of excess bleeding and the nurse should report these findings to the surgeon. The nurse should also monitor the client's Hgb and Hct to help determine the degree of blood loss.

A nurse has just received change-of-shift report for a group of clients. Which of the following actions by the nurse demonstrates effective time management skills? A - Completes low-priority tasks first. B - Keeps a client to-do list for the day. C - Charts client tasks at the end of the shift. D - Focuses on several client tasks at a time.

B - Keeps a client to-do list for the day.

A nurse is reviewing the laboratory report of a preschooler. Which of the following laboratory results should the nurse report to the provider? A - Potassium 4.2 mEq/L B - Lead 14 mcg/dL C - Fasting glucose 75 mg/dL D - Hematocrit 40%

B - Lead 14 mcg/dL

A nurse is reinforcing teaching about infection transmission with an assistive personnel (AP) who is caring for a client who has a Clostridium difficile infection. Which of the following instructions should the nurse include? A - Use alcohol-based hand sanitizer after caring for the client. B - Leave the blood pressure cuff in the client's room. C - Keep the door to the client's room closed. D - Wear an N95 respirator when within 0.9 m (3 feet) of the client.

B - Leave the blood pressure cuff in the client's room. Rational A - Alcohol-based hand sanitizers do not kill C. difficile spores. The AP should wash both hands using soap and water after caring for the client. B - C. difficile is spread by indirect contact with equipment such as blood pressure cuffs, thermometers, and stethoscopes. Therefore, the AP should leave the blood pressure cuff and other equipment in the client's room. C - Contact precautions protect visitors and caregivers when they are within 0.9 m (3 feet) of the client against direct client and environmental contact infections. The AP should keep the door to a client's room closed when providing care for a client who has an infection that requires a negative pressure room and is transmitted by the airborne route, such as rubella and varicella. D - The AP should wear an N95 respirator when caring for a client who requires airborne precautions. Airborne precautions are implemented when the client has tuberculosis, varicella, rubella, or herpes zoster.

A nurse is caring for a client who appears distraught after losing all their possessions in an apartment fire. Which of the following interventions should the nurse implement? A - Encourage the client to block negative feelings. B - Maintain eye contact with the client during communication. C - Adopt an optimistic attitude by telling the client that the situation will end positively. D - Request a chaplain to help provide spiritual support for the client.

B - Maintain eye contact with the client during communication. Rational A - The nurse should encourage the client to express feelings regarding the loss of possessions and the effect of the fire on the client's life. Blocking of feelings, even those that are negative, is a maladaptive response to the crisis and can limit the client's ability to effectively cope with the situation. B - Maintaining eye contact with the client is a therapeutic communication technique. It is important for the nurse to provide an atmosphere of support and safety when caring for and communicating with the client. By showing concern for the client, the nurse helps to promote client trust and a sense of optimism. Maintaining eye contact demonstrates support, empathy, and advocacy. C - The nurse should focus on the client's feelings and unique responses to crisis. The nurse should not give reassurance regarding potential future positive outcomes, which minimizes the seriousness of the situation for the client. The nurse's goal should be the client's safety and reduction of anxiety through implementation of the client's coping skills. D - The facility chaplain is a support person who provides counsel and spiritual guidance for clients and their families. The nurse should assist the client to identify sources of support; however, the client should be the one to request a spiritual support person and the nurse should not make this decision on behalf of the client. The nurse should not presume to know what is best for the client. It might be appropriate in the future to connect the client with a person or group of people who have experienced a similar crisis, but it would not be appropriate to contact a chaplain without including the client in this decision.

A nurse is collecting data from a client who has antisocial personality disorder. Which of the following findings should the nurse expect? A- Anxiety B - Manipulative behavior C - Submissive behavior D - Perfectionism

B - Manipulative behavior Rational A - Anxiety is not a finding associated with clients who have antisocial personality disorder. This finding is common with clients who have schizotypal personality disorder. B -Manipulative behavior is a finding associated with clients who have antisocial personality disorder. Additional characteristics of clients who have this disorder include exploitative, seductive, and aggressive behaviors. C - Submissive behavior is not a finding associated with clients who have antisocial personality disorder. This finding is common with clients who have dependent personality disorder. D - Perfectionism is not a finding associated with clients who have antisocial personality disorder. This finding is common in clients who have obsessive compulsive disorder.

A nurse is caring for a client who has meningococcal pneumonia. Which of the following personal protective equipment should the nurse use? A- Gown B - Mask C - Sterile gloves D - Protective eyewear

B - Mask

A nurse is caring for a client who gave birth to a stillborn fetus one week ago. She stats to the nurse, "I am so angry that my doctor didn't take better care of me and my baby." Which of the following responses should the nurse make? A - "You will start feeling better in a few weeks." B - "You should begin to accept what happened so you can work through your loss." C - "It is important to share what you are feeling, even if it is anger." D - "Exhibiting anger toward others will only make you feel guilty later on."

C - "It is important to share what you are feeling, even if it is anger."

A nurse is reinforcing teaching about treatment for a food blockage in the intestine with a client who has an ileostomy. Which of the following information should the nurse include in the teaching? A - Lavage the ileostomy. B - Massage around the ileostomy stoma. C - Take a stimulating laxative. D - Begin a diet high in fiber.

B - Massage around the ileostomy stoma. Rational A - The nurse should tell the client not to lavage the ileostomy. The client should notify the provider or an enterostomal nurse if unable to relieve the blockage. B - The nurse should tell the client to massage the abdominal area around the stoma to create pressure behind the food blockage to move it forward through the stoma. Changing positions or taking a warm bath can help promote passage of the blockage. C - The nurse should not tell the client to take a stimulating laxative because it can increase cramping and will not move the food blockage forward. The client might need to go to the emergency department if the blockage does not resolve and the provider or enterostomal nurse are unavailable. D - The nurse should tell the client to begin a liquid diet until the food blockage is resolved and to increase fluid intake to 1.9 L (8 cups) daily. The client should slowly add foods high in fiber to the diet once the blockage resolves.

A nurse is reinforcing teaching with a client who has left-sided weakness and is beginning to use a cane for ambulation. Which of the following instructions should the nurse give to the client? A - Hold the cane with the left hand. B - Move the cane forward before advancing the left foot. C - Advance the cane forward 36 to 46 cm (14 to 18 in) with each step. D - Keep three points of support on the floor at all times.

B - Move the cane forward before advancing the left foot. Rational A - The client should hold the cane with the right hand so the cane is on the stronger side of the body. B - The client should move the cane forward while the body weight is on both legs. The client should then advance the weaker leg (left leg) toward the cane and then advance the stronger leg (right leg) forward past the cane. C - For maximum support when walking, the client should place the cane forward 15 to 30 cm (6 to 12 in), keeping body weight on both legs. D - The client should keep two points of support, not three, on the floor. For example, the client should have both feet, or one foot and the cane, on the floor at any given time. If the client becomes stronger and is able to maintain balance with minimal support, they can move the cane and weak leg at the same time.

A nurse in a mental health facility is caring for a client who has dementia. The client's agitation is increasing. Which of the following actions should the nurse take first? A - Move the client to a private room. B - Offer diversionary activities. C - Administer haloperidol. D - Apply wrist restraints.

B - Offer diversionary activities.

A nurse is monitoring an older adult client who has a history of an enlarged prostate and is experiencing suprapubbic discomfort. Which of the following actions should the nurse take first? A - Administer doxazosin. B - Palpate the abdomen. C - Insert an indwelling urinary catheter. D - Notify the primary care provider.

B - Palpate the abdomen.

A nurse is reveiwing the laboratory results of a 4-hr-old newborn. Which of the following findings should the nurse report to the provider? A - Hemoglobin 20 g/dL B - Platelet count 120,000/mm3 C - Glucose 50 mg/dL D - WBC count 20,000/mm3

B - Platelet count 120,000/mm3

A nurse is reinforcing teaching about car seat safety with the parents of a newborn who weighs 3.5 kg (7.7 lb). Which of the following information should the nurse include? A - Place the retainer clip over the newborn's abdomen. B - Position the child rear-facing in the back seat until at least 2 years of age. C - Position the child at a 60° angle in the car seat. D - Place the shoulder harnesses in the slot 5 cm (2 in) above the newborn's shoulders.

B - Position the child rear-facing in the back seat until at least 2 years of age. Rational A - The nurse should instruct the parents to place the retainer clip at the level of the newborn's axillae. The retainer clip should not be placed over the abdomen because this can cause abdominal injury in the event of a motor-vehicle crash. B - The nurse should instruct the parents to position the child rear-facing in the back seat of the car until at least 2 years of age or until the child exceeds the height and weight limits set by the car seat manufacturer and state requirements. C - The nurse should instruct the parents to position the child at a 45° angle in the car seat. Newborns do not have the ability to hold their heads upright. Therefore, placing the newborn at a 60° angle would cause slumping, which could lead to airway obstruction. D - The nurse should instruct the parents to place the shoulder harnesses in the slot at or below the level of the newborn's shoulders. The harnesses should be snug against the newborn's shoulders to prevent injury in the event of a motor-vehicle crash.

A nurse is reviewing the laboratory results of a client who has type 2 diabetes mellitus. The nurse should identify that which of the following laboratory values indicated the client is at risk for delayed wound healing? A - HbA1c 6% B - Prealbumin 12 mg/dL C - WBC 8,000/mm3 D - Creatinine 0.8 mg/dL

B - Prealbumin 12 mg/dL

A nurse is collecting data from a client who has bipolar disorder and a history of mania. Which of the following findings should the nurse identify as an indication that the client is relapsing? A - Weight gain B - Pressured speech C - Ritualistic behavior D - Anhedonia

B - Pressured speech

A nurse is caring for a client who is planning to become pregnant. The client asks the nurse why folic acid supplements are necessary. The nurse should inform the client that the purpose of the folic acid supplement is to do which of the following? A - Facilitate the storage of iron in the fetus' liver B - Prevent certain kinds of birth defects C - Inhibit premature labor D - Aid in the absorption of other important nutrients

B - Prevent certain kinds of birth defects

A nurse is contributing to the plan of care for a client who has acute hypothyroidism. Which of the following interventions should the nurse include in the plan? A - Provide the client with a cool environment. B - Provide the client with a reduced-calorie diet. C - Place the client on a fluid restriction. D - Place the client on strict bed rest.

B - Provide the client with a reduced-calorie diet.

A nurse is caring for a client who is 1 day postoperative following a hip arthroplasty. The client is exhibiting hypotension, tachycardia, and tachypnea. The nurse should recognize that these findings indicate which of the following complications? A - Wound infection B - Pulmonary embolism C - Thrombophlebitis D - Paralytic ileus

B - Pulmonary embolism

A nurse is preparing to transfer a client who has unilateral weakness from the bed to a bedside commode. Which of the following pieces of equipment should the nurse use to transfer the client? A - Mechanical lift B - Quad cane C - Slide board D - Walker

B - Quad cane Rational A - The nurse should use a mechanical lift to transfer a client who is unable to assist the nurse. The nurse should not lift more than 15.88 kg (35 lb) of the client's weight without the use of an assistive device in order to reduce the risk of injury to the nurse and maintain safety for the client. B - The nurse should provide a quad cane for a client who has unilateral weakness because it promotes mobility and independence while maintaining safety. The quad cane provides more support than a traditional cane. C - The nurse should use a slide board to transfer a client from the bed to a stretcher. The slide board helps to reduce friction while moving a client. The slide board is placed under a draw sheet for a client who is lying supine. D - The nurse should provide a walker for a client who has adequate strength in both arms to move the walker forward. A client who has unilateral weakness cannot perform this function and requires a different piece of equipment for assistance when moving from the bed to a bedside commode.

A nurse is reviewing the prenatal record of a client who is at 34 weeks of gestation. Which of the following results should the nurse identify as a desirable outcome? A - Negative rubella titer B - Reactive nonstress test C - 1-hr glucose tolerance screening test result of 150 mg/dL D - Hemoglobin 9.5 g/dL

B - Reactive nonstress test

A nurse is contributing to the plan of care for a client and recognizes that one of the established goals is unrealistic. Which of the following actions should the nurse take? A - Document the client's noncompliance with the plan of care. B - Recommend a revision to the plan of care. C - Discontinue nursing interventions related to the goal. D - Create a new plan of care.

B - Recommend a revision to the plan of care. Rational A - The nurse should provide objective documentation of the client's current state of health and abilities to comply with the plan. B - The nurse should recommend a revision to the plan of care to reflect the client's current state of health and abilities to adhere to the plan. C - The nurse should not discontinue nursing interventions related to the goal. The nurse should assist the RN to revise the interventions to meet the goal of the plan of care. D - It is beyond the scope of practice for the PN to create a new plan of care. The nurse should assist the RN in revising the client's plan of care.

A nurse is contributing to the plan of care for a client who has a new diagnosis of type 2 diabetes mellitus. Which of the following members of the interprofessional healthcare team should the nurse consult to assist the client with glucose management? A - Occupational therapist B - Registered dietitian C - Social worker D -Speech therapist

B - Registered dietitian

A nurse is assisting in the planning of in-home care for a client following a right-hip arthroplasty. Which of the following interventions is the nurse's priority? A - Provide the client with a list of follow-up appointments. B - Reinforce teaching about the client's use of a walker. C - Identify the client's support system. D -Discuss services that can assist the client with ADLs.

B - Reinforce teaching about the client's use of a walker.

A nurse is providing discharge teaching for the family of a client who has Parkinson's disease. Which of the following information should the nurse include in the teaching? A - Place the client on a low-calorie diet to prevent weight gain. B - Remind the client to avoid watching her feet when walking. C - Use small area rugs in the client's home for traction. D - Instruct the client to take tub baths instead of showers.

B - Remind the client to avoid watching her feet when walking. Rational A - The nurse should instruct the client's family to provide the client with extra calories and protein to prevent unintentional weight loss from expenditure of energy due to tremors, dyskinesia, and difficulty swallowing. B - The nurse should instruct the client's family to frequently remind the client to maintain correct posture and prevent falls by not watching her feet when walking. C - The nurse should instruct the client's family to avoid using area rugs in the client's home because her foot may drag or be stiff and catch on an area rug, which can cause a fall. D - The nurse should instruct the family to encourage the client to take walk-in, sit-down showers, because skeletal muscle rigidity can cause difficulty in moving, coordination, and balance, which increases the risk of a fall.

Following a blood draw procedure for a fasting blood sugar (FBBS) test, a client tells the nurse, "I'm glad they took my blood because I'm really hungry. I'm really hungry. All I've had since midnight is water and some juice." Which of the following actions should the nurse take? A - Offer the client breakfast then repeat the FBS request. B - Reschedule the FBS test for early the next morning. C - Request that the phlebotomist obtain another specimen. D - Ask the laboratory technician to repeat the test on the same specimen.

B - Reschedule the FBS test for early the next morning.

A newly licensed nurse is preparing to administer a subcutaneous medication to a client and is unsure of the correct technique. Which of the following actions should the nurse take? A - Ask the pharmacist to explain the procedure. B - Review the institutional policy and procedure manual. C - Refer to the American Nurses Association (ANA) Code of Ethics. D - Complete an incident report.

B - Review the institutional policy and procedure manual. Rational A- A pharmacist prepares and distributes prescribed medications and can provide information about medication adverse effects, toxicity, interactions, and incompatibilities. However, the pharmacist cannot provide information regarding medication administration procedures. B - The institutional policy and procedure manual will provide detailed information about how the nurse should perform client care procedures. It establishes the standard of practice for employees of an institution. Nurses should be familiar with their institution's policies and procedures and how to locate the manual if necessary. C - The ANA Code of Ethics is a set of principles nurses use for guidance in providing ethical client care. The code does not outline the steps of procedures, such as injections. D - The nurse should complete an incident report if an event occurs outside the standard care of a client. Examples of variances include falls, medication errors, and needlestick injuries.

A nurse is reinforcing teaching with the family of a client who has histrionic personality disorder. Which of the following high-risk behavior should the nurse instruct the family to observe for in the client? A - Self-mutilation through cutting B - Seductive behavior C - Repeated physical aggression D - Reckless driving

B - Seductive behavior

A nurse is reinforcing teaching with a client who has a new prescription for alendronate to treat osteoporosis. Which of the following instructions about taking this medication should the nurse include? A - Take an antacid with the medication. B - Sit upright for at least 30 min after taking the medication. C - Wait at least 15 min before eating or drinking anything except water. D - Take the medication at bedtime.

B - Sit upright for at least 30 min after taking the medication. Rational A - Antacids are available over the counter and are intended to help treat gastric upset. Antacids that contain calcium or magnesium can reduce the absorption of alendronate. The client should not take an antacid for at least 2 hr after taking this medication. B - Alendronate, a bisphosphonate, can cause esophagitis. Prolonged contact of the medication with the esophagus can cause ulceration. The client should sit upright or stand for at least 30 min after taking this medication to reduce the risk of esophageal injury. C - The client should have no food or beverages except for plain water for at least 30 min after taking this medication. This includes orange juice and coffee. Foods high in calcium, iron, and magnesium should not be taken within 2 hr of alendronate because this will reduce absorption of the medication. D - For optimal bioavailability, the client should take alendronate in the morning before breakfast. Taking the medication at bedtime is not recommended, because reflux can occur in the supine position, which will result in esophagitis or ulceration of the esophagus.

A nurse is reveiwing the laboratory report of a school-age child who is receiving prednisone. Which of the following laboratory results should the nurse report to the provider? A - Fasting blood glucose 74 mg/dL B - Sodium 150 mEq/L C - Potassium 4.2 mEq/L D - WBC count 9,400/mm3

B - Sodium 150 mEq/L

A nurse in a long-term care facility is caring for a client woh had a stroke 1 week ago. The client is experiencing left-side weakness, difficulty swallowing, drooping of the mouth, inarticulate speech, and memory loss. Which of the following referrals is the priority for the nurse to make? A - Physical therapy B - Speech therapy C - Cognitive therapy D - Occupational therapy

B - Speech therapy

A nurse is assisting the provider with a development assessment of a toddler. Which of the following behaviors should the nurse recognize as an expected findings? A - Walks backward with heel to toe B - Stands on one foot for several seconds C - Uses scissors to cut out shapes D - Prints letters with a pencil

B - Stands on one foot for several seconds

A nurse is caring for a client who is receiving intravenous therapy through a peripheral catheter. The nurse notes that the skin around the client's IV catheter site feels tight and cool to touch. Which of the following actions should the nurse take? A - Straighten the client's arm. B - Stop the infusion. C - Raise the height of the IV infusion bag. D - Apply a pressure dressing to the IV site.

B - Stop the infusion. Rational A - The nurse should straighten or reposition the client's arm to improve the rate of flow for a positional IV. However, this action will not address the infiltrated IV. B - The nurse should identify that the client's IV catheter has infiltrated. The nurse should stop the infusion, remove the catheter, elevate the client's arm to promote absorption of the infiltrated fluid, and apply either a warm or cold compress to promote comfort. Manifestations of infiltration include edema, skin blanching, cool, taut skin, and pain. If the IV solution is a vesicant, the catheter should remain in place in case an antidote is to be administered. C - The nurse should raise the height of the IV infusion bag to increase the rate of flow for an IV that requires additional gravity. However, this action will not address the infiltrated IV. D - The nurse can apply a pressure dressing for an IV that is bleeding at the puncture site. However, this action will not address the infiltrated IV and might result in tissue damage.

A nurse is caring for a client who received a skin tear during a routine dressing change. After completing an incident report, which of the following actions should the nurse take? A - Document the completion of the incident report in the client's medical record. B - Submit the incident report to the nurse manager for review. C - Mail a copy of the incident report to the facility's attorney. D - Obtain the client's signature on the incident report.

B - Submit the incident report to the nurse manager for review.

A nurse is caring for a client who is taking lisinopril. Which of the following findings should the nurse address first? A - Impaired sense of taste B - Swelling of the tongue C - Decreased blood pressure D - Rash

B - Swelling of the tongue Rational A - The nurse should monitor the client for potential anorexia and weight loss due to altered taste. However, there is another finding the nurse should address first. B - The greatest risk to the client from receiving lisinopril is angioedema, which is an indication of an anaphylactic allergic reaction. The nurse should implement actions to support respiratory function. Manifestations include giant wheals as well as edema of the tongue, glottis, and pharynx. Severe reactions should be treated with subcutaneous epinephrine. If angioedema develops, ACE inhibitors should be discontinued and never used again for this client. C - The nurse should monitor the client's blood pressure and reinforce that the client is to change positions slowly, because it is common for the client to experience orthostatic hypotension after receiving the first dose of an ACE inhibitor. However, there is another finding the nurse should address first. D - The nurse should continue to monitor the client and notify the provider that the client is experiencing a rash. However, there is another finding the nurse should address first.

A nurse is assisting with collecting data from a newborn who was born 2 hr ago and has respiratory distress. Which of the following findings should the nurse report to the provider? select all A - Acrocyanosis B - Tachypnea C - Nasal flaring D - Retractions E - Expiratory grunting

B - Tachypnea C - Nasal flaring D - Retractions E - Expiratory grunting Rational A - Acrocyanosis is incorrect. Acrocyanosis is a bluish discoloration of the hands and feet of the newborn and is an expected finding during the first 24 hr after birth. B - Tachypnea is correct. Tachypnea is a respiratory rate greater than 60/min and is a finding associated with respiratory distress in the newborn. C - Nasal flaring is correct. Nasal flaring is a finding associated with respiratory distress in the newborn. D - Retractions is correct. Retractions are a finding associated with respiratory distress in the newborn. E - Expiratory grunting is correct. Expiratory grunting is a finding associated with respiratory distress in the newborn.

A charge nurse is reinforcing teaching with a newly licensed nurse regarding vaccines to administer during pregnancy. Which of the following vaccines are safe for a newly licensed nurse to administer? Select all A - Measles, mumps, and rubella (MMR) B - Tetanus C - Varicella D - Inactivated influenza E - Recombinant hepatitis B

B - Tetanus D - Inactivated influenza E - Recombinant hepatitis B Rational A - Measles, mumps, and rubella (MMR) is incorrect. Vaccines that contain live or attenuated live viruses, such as the MMR vaccine, are contraindicated during pregnancy because of potential teratogenic effects on the fetus. B - Tetanus is correct. The tetanus vaccine can be administered during pregnancy because it does not contain a live virus. C - Varicella is incorrect. The varicella vaccine should not be administered during pregnancy because it is a live virus vaccine. D - Inactivated influenza is correct. The inactivated influenza vaccine can be administered during pregnancy because it does not contain a live virus. E - Recombinant hepatitis B is correct. The recombinant hepatitis B vaccine can be administered during pregnancy because it does not contain a live virus.

A nurse is preparing to give change-of-shift report about a client who had a transurethral resection of the prostate 6 hr ago and has continuous bladder irrigation. Which of the following information is the priority for the nurse to report? A - The client reports they have an urge to void. B - The client passed three large blood clots in their urine. C - The client was medicated 2 hr ago for gas pain. D - The client is taking only sips of their clear liquid diet.

B - The client passed three large blood clots in their urine. Rational A - A client who has an indwelling urinary catheter and reports an urge to void is at risk for impaired discomfort; however, another finding is the priority to report. The retention balloon of the catheter is an object foreign to the body and can produce bladder spasms or an urge to void. Securing the catheter to the client's thigh reduces irritation to the urethra and might decrease the urgency to void. B - The greatest risk to this client is injury from bladder obstruction; therefore, the priority information for the nurse to report is the passage of three large blood clots. Bleeding from surgery can cause clot formation, and large clots can obstruct the bladder. C - A client who is experiencing gas pain is at risk for impaired discomfort; however, another finding is the priority to report. Abdominal cramping due to gas is common following prostatectomy and should be managed with analgesia and ambulation when possible. The client can also experience constipation and should be warned not to strain to have a bowel movement. D - A client who is consuming a clear liquid diet is at risk for inadequate nutrition; however, another finding is the priority to report. Following surgery, the nurse should offer the client ice chips, then sips of clear liquids, progressing to full liquids and the prescribed solid diet as tolerated. The nurse should not recommend advancing the diet if the client has nausea or vomiting. The nurse can recommend a dietitian referral if altered nutrition is suspected.

A nurse is reinforcing teaching about home care with a client who had a knee arthroplasty. Which of the following factors should the nurse identify as an indication that a barrier to learning might be present? A - The client asks questions each time the nurse stops talking. B - The client stops the nurse and asks for pain medication. C - While the nurse is speaking, the client refers to the written materials. D - A family member who is present asks the client to repeat important points.

B - The client stops the nurse and asks for pain medication.

A nurse in a provider's office is collecting data from an older adult client who adult child reports that she "seems confused and can't seem to remember much." Which of the following findings should lead the nurse to suspect delirium? A - The client's confusion worsens during times of stress. B - The client's level of consciousness changes during the interview. C - The client's confusion improves in the evening. D - The client becomes irritable during the interview.

B - The client's level of consciousness changes during the interview. Rational A - A client who has depression would exhibit an exacerbation of confusion during times of stress. B -Delirium can rapidly alter the client's level of consciousness, which can manifest as agitation or stupor. Therefore, the nurse should suspect that this client is experiencing delirium. C - The client's confusion will worsen in the evening, and the sleep wake cycle might be reversed. D - A client who has depression would exhibit irritability, anxiety, and paranoid behavior during an interview.

A nurse is caring for a client who has schizophrenia. Which of the following actions by the nurse is a violation of the client's confidentiality? A - The nurse documents client statements word-for-word in the client's medical record. B - The nurse places the client's diagnosis on the white board in the client's room. C - The nurse faxes the client's known allergies to the pharmacy. D - The nurse reports threats the client made to harm her partner to the provider.

B - The nurse places the client's diagnosis on the white board in the client's room.

A nurse is reinforcing teaching with the parent of a preschooler about administering liquid oral medications. Which of the following actions by the parent indicates an understanding of the instructions? A - The parent administers the medication in the child's favorite food. B - The parent offers a frozen juice bar after administering the medication. C - The parent positions the child in a supine position to administer the medication. D - The parent measures the medication using a household teaspoon.

B - The parent offers a frozen juice bar after administering the medication. Rational A - The nurse should reinforce that the parent should avoid mixing medications in a child's favorite food because the child might no longer wish to eat the food once the child forms an association between that food and the medication. B - The parent can offer a frozen juice bar after administration of an oral medication to remove any unpleasant aftertaste from the medication. C - The nurse should reinforce that the parent should keep the child in a semi-upright position when administering oral medications to reduce the risk of aspiration. D - The nurse should reinforce with the parent that a household teaspoon, dropper, or measuring cup is not accurate enough to use when measuring liquid oral medications for a child. The parent should use a plastic disposable oral syringe or an approved medication measuring device to measure and administer the liquid oral medication.

A nurse is collecting data from a 6-month-old child who is eperinice a sickle cell crisis. Which of the following areas should the nurse observe when monitoring for manifestations of splenic sequestration? A - upper right quadrant B- upper left quadrant C - lower right quadrant

B- upper left quadrant Rational A is incorrect. The nurse should observe the location over the infant's liver when monitoring for manifestations of liver sequestration, or an enlarged liver. B is correct. The nurse should observe the location over the infant's spleen when monitoring for manifestations of splenic sequestration. Splenic sequestration is an enlargement of the spleen due to pooling of sickled cells in the blood. C is incorrect. The nurse should check the location over the kidney in an infant who is experiencing a sickle cell crisis for manifestations of a kidney infarction. Other manifestations of sickle cell anemia include dilute urine, hematuria, and enuresis.

A nurse is monitoring a client who has a prescription for wrist restraints to prevent removal of an indwelling urinary catheter. Which of the following findings requires interventions by the nurse? A - The restraint is padded at the client's wrist bone. B - The restraint is attached to the side rails of the bed. C - The restraints limit the range of movement of the client's hand. D - The nurse can insert two fingers between the client's wrist and the restraint.

B - The restraint is attached to the side rails of the bed. Rational A - The nurse should ensure that boney prominences, such as the wrist bones, are padded when applying restraints to prevent friction that can lead to skin breakdown. B - The use of restraints is sometimes necessary to provide for the safety of a client. The nurse can request a prescription for restraints if a client is at risk for falls, is confused, is at risk for removing support equipment or treatment, or is a threat to others. The provider should perform a face-to-face assessment and prescribe the restraints for a specific period of time and never PRN. The nurse should secure the restraint to a movable part of the bed frame and never to the side rails. This will allow for the restraints to move with the client when the nurse moves the head of the bed up or down. C - The intention of the restraints is to limit the client's movement and to prevent client removal of the indwelling urinary catheter. However, the nurse should avoid totally immobilizing the client to prevent injury. The nurse should review facility protocol regarding restraints and perform circulatory checks according to protocol. D - The nurse should be able to insert two fingers between the restraint and the client to ensure the restraint is not too tight, which can cause neurovascular injury. The nurse should check the client frequently, according to facility policy, for signs of injury. These checks should include skin integrity, pulses, color, temperature of skin, and sensation.

A community health nurse is reinforcing teaching with a group of parents about home safety for children of various age groups. Which of the following informations should the nurse plan to include? A - The risk of lead poisoning is greatest in 10-year-old children. B - The supine position is the safest for sleeping infants. C - Falls are the leading cause of injury in preschoolers. D - Air-popped popcorn is a recommended snack for toddlers.

B - The supine position is the safest for sleeping infants.

A nurse is reinforcing teaching with the guardian of a child who has a new diagnosis of enterobiasis. The nurse should advise the guardian to take which of the following actions to prevent infection? A - Dress the child in two-piece sleeping outfits. B - Trim the child's fingernails short. C - Have the child take a tub bath daily. D - Repeat treatment in 4 weeks.

B - Trim the child's fingernails short.

A nurse is assisting with the care of a child who is experiencing a tonic-clonic seizure and is vomiting. Which of the following actions is the nurse's priority? A - Administer oxygen via facemask. B - Turn the child to a side-lying position. C - Check the child's pulse rate. D - Suction the child's oral cavity.

B - Turn the child to a side-lying position. Rational A - Administering oxygen via facemask is important to prevent hypoxia, but it is not the nurse's priority action. B - The nurse's priority action when using the airway, breathing, circulation approach to client care is to turn the child to a side-lying position if she is vomiting to prevent obstruction of the airway and aspiration of the stomach contents. C - Checking the child's pulse rate is important to determine circulatory function, but it is not the nurse's priority action. D - Suctioning the child's oral cavity should be done after the seizure, rather than during the seizure. During the seizure, the child is likely to clench the jaw, often making it difficult to insert a suction catheter. Therefore, this is not the nurse's priority action.

A nurse is caring for a client who has diabetes insipidus and a new prescription for desmopressin. Which of the following laboratory values indicates a therapeutic responses to the medication? A - Fasting blood glucose 80 mg/dL B - Urine specific gravity 1.015 C - Serum sodium 134 mEq/L D - Serum creatinine 1.0 mg/dL

B - Urine specific gravity 1.015 Rational A - A fasting blood glucose of 80 mg/dL is within the expected reference range of 70 to 110 mg/dL. However, desmopressin is not expected to alter blood glucose levels. B - Diabetes insipidus occurs when the posterior pituitary gland does not secrete enough antidiuretic hormone to concentrate urine. The urine becomes diluted, and the serum becomes concentrated. A urine specific gravity within the expected reference range indicates a therapeutic response to the medication. This finding is within the expected reference range of 1.005 to 1.030. C - This finding is below the expected reference range of 136 to 145 mEq/L. Diabetes insipidus occurs when the posterior pituitary gland does not secrete enough antidiuretic hormone. Hyponatremia can be an adverse effect of desmopressin. D - A serum creatine of 1.0 mg/dL is within the expected reference range of 0.5 to 1.3 mg/dL. However, desmopressin is not expected to alter serum creatinine levels.

A nurse is performing a straight catheterization of a client in order to obtain a urine specimen for culture and sensitivity. Which of the following actions should the nurse take? A - Collect urine specimen from the catheter port. B - Use a sterile specimen container for collection. C - Ensure sterile water is used to inflate the balloon. D - Unwrap the catheter kit before washing the client's perineal area.

B - Use a sterile specimen container for collection. Rational A - The urine from a straight catheter (single lumen tube) flows directly into the specimen container. Collecting a urine specimen from a catheter port is necessary when the client has an indwelling urinary catheter in place. B - A catheter urine specimen for culture and sensitivity requires a sterile specimen from a straight or indwelling catheter using sterile technique. The nurse should use a sterile specimen container to prevent contamination of the specimen by micro-organisms outside of the bladder. C - A straight catheter has a single lumen for draining urine directly from the bladder into a sterile collection container. A straight catheter does not remain in the bladder and does not have a second lumen with a balloon. The nurse should include this step if inserting an indwelling urinary catheter. D - The nurse should clean the client's perineal area with soap and water before opening the sterile catheter kit to maintain aseptic technique.

A nurse is preparing to remove the sutures from a client's abdominal incision. Which of the following actions should the nurse plan to take? A - Clean the suture site with full-strength hydrogen peroxide prior to removal. B - Use suture scissors to cut the suture close to the skin before removal. C - Pull the visible portion of the suture through the skin using forceps. D - Remove each suture, one by one, moving from proximal to distal on the incision.

B - Use suture scissors to cut the suture close to the skin before removal. Rational A - The nurse should cleanse the incision site with 0.9% sodium chloride solution prior to removing sutures. This allows the skin to remain moist and promotes wound healing. The use of a disinfectant such as full-strength hydrogen peroxide can irritate the wound tissue and delay healing. B - The nurse should use sterile suture scissors to cut the suture as close to the skin as possible prior to removal. This action reduces the risk for infection by preventing exposed suture material, which is considered contaminated, from being pulled through the skin. C - The nurse should use forceps or hemostats to grasp the suture and pull it out without pulling the visible portion through the underlying skin. This technique prevents micro-organisms on the visible portion of the suture from being introduced into the sterile underlying skin. This helps to decrease the risk for infection. D - The nurse should remove the sutures in small groups or in an alternating pattern (e.g., the second, fourth, and sixth sutures) to decrease the client's risk for dehiscence. If dehiscence occurs, the remaining sutures serve to help maintain approximation of the incision edges while the charge nurse and provider are notified.

A nurse is determining a client's ability to perform self-care following a stroke. Which of the following findings should the nurse identify as the priority? A - Weak grip B - Vision impairment C - Aphasia D - Emotional lability

B - Vision impairment Rational A - A weak grip indicates that the client is at risk for impaired self-care. The nurse should collect further data to determine whether occupational therapy might be needed. However, another finding is the priority. B - The greatest risk to this client is injury from a fall. A visual impairment resulting from a stroke alters the client's visual field and increases the risk for falls while performing self-care activities. Therefore, the nurse should identify this as the priority finding. Visual impairments following a stroke can include partial to total blindness in one or both eyes. C - Aphasia indicates that the client is at risk for inability to communicate. The nurse should collect further data from the client and determine what type of device is needed to promote communication. However, another finding is the priority. D - Emotional lability indicates that the client is at risk for impaired coping following a stroke. The nurse should collect further data about the client's psychosocial status and check for other behavioral changes. However, another finding is the priority.

A nurse is caring for a client who has methicillin-resistant Staphylococcus aureus (MRSA) infection in a surgical wound. Which of the following information should the nurse plan to share with visitors? A - Visitors should call prior to visiting the client. B - Visitors must don a gown and gloves prior to entering the client's room. C - Visitors need to wear a mask when in close proximity to the client. D - Visitors may not bring fresh flowers into the client's room.

B - Visitors must don a gown and gloves prior to entering the client's room. Rational A - Visitors do not need to make arrangements prior to visiting a client who is on contact isolation precautions, but visitors should receive assistance before entering the client's room. B - The nurse should provide teaching to the visitors regarding the infection control measures for a client who is on contact isolation precautions. Contact precautions require visitors to put on a gown and gloves prior to entering the room of a client who has MRSA to prevent the spread of infection. C - The nurse should identify that visitors of clients who are on airborne or droplet precautions should wear a mask when within 3 feet of the client. However, MRSA is not spread through the respiratory tract and does not require airborne or droplet precautions. D - The nurse should identify that fresh flowers are contraindicated for a client who is on neutropenic precautions. However, they are not contraindicated for a client who has MRSA.

A nurse is checking the urinary output of an 8-month-old infant who is dehydrated and weights 10 kg (22 lb). The nurse collected 160 mL of urine during the past 8 hr. Which of the following actions should the nurse take? A - Obtain a urine specimen for culture and sensitivity. B - Weigh the infant's diapers following each diaper change. C - Provide 800 mL of fluid per day. D - Perform a bladder scan at the bedside.

B - Weigh the infant's diapers following each diaper change. Rational A - The nurse should obtain a urine specimen for culture and sensitivity if there is an indication of infection. Obtaining a urine specimen for culture and sensitivity is not required for an infant who is dehydrated. B - The nurse should weigh the infant's diapers to accurately monitor the infant's output and fluid balance. The infant should have a urinary output of 1 to 2 mL/kg/hr. One gram of wet diaper weight is equal to 1 mL of urine. C - The nurse should increase the infant's fluid intake to maintain daily fluid requirements. The infant needs a minimum of 1,000 mL of fluid per day. Daily Maintenance Fluid Requirements: 1. Calculate weight of child in kg. 2. Allow 100 mL/kg for first 10 kg. 3. Allow 50 mL/kg for second 10 kg. 4. Allow 20 mL/kg for remainder of weight in kg. 5. Divide total amount by 24 hr to obtain rate in mL/hr. D - The nurse should perform a bladder scan when urinary retention is suspected. The bladder scan is used to screen for postvoid residual volumes and determine the need for intermittent catheterization. However, there is no indication of urinary retention for this infant.

A charge nurse working in a long-term care facility is observing a newly licensed nurse administer a Mantoux tuberculin skin test. The charge nurse should intervene when the newly licensed nurse does which of the following? A - Creates a 6.35 mm (0.25 in) bleb in the intradermal space of the forearm B - Withdraws the needle and massages the site gently C - ​Stretches the skin tightly and injects the solution slowly D - Visualizes the tip of the needle under the skin as the solution is injected

B - Withdraws the needle and massages the site gently Rational A - The Mantoux tuberculin skin test is a method of determining whether a client is infected with Mycobacterium tuberculosis. During the test, the solution is injected intradermally into the forearm until a bleb forms. The bleb should measure approximately 6.35 mm (0.25 in) in diameter, resembling a mosquito bite. B - The Mantoux tuberculin skin test is a method of determining whether a client is infected with Mycobacterium tuberculosis. Following the injection, the needle is removed and gentle pressure is applied to the site. However, the nurse should not massage the site. Massaging can cause tissue damage and cause the solution to be dispersed into the surrounding tissue, altering the test's results. C - During administration of intradermal injections, the skin is stretched tightly with the nondominant hand. The needle is inserted slowly with the bevel up and at a 5° to 15° angle until the nurse feels resistance. The nurse should inject the solution slowly. D - The needle is inserted slowly with the bevel up and at a 5° to 15° angle until the nurse feels resistance. The needle is then advanced to approximately 3 mm (0.12 in) below the skin surface. The needle will be visible through the skin.

A nurse is collecting data from a client who has a heart rate of 86/min, a blood pressure of 80/40 mm Hg, respirations of 28/min with a productive cough, and a temperature of 38.6°C (101.5°F). After notifying the provider, which of the following actions should the nurse take first? A - Give acetaminophen. B - Withhold enalapril. C - Obtain a sputum culture. D - Administer erythromycin.

B - Withhold enalapril. Rational A - The nurse should give the client acetaminophen for the elevated temperature. However, this is not the priority action for the nurse to take. B - The greatest risk to the client is hypotension resulting from antihypertensive therapy. Therefore, the nurse should withhold this medication. Enalapril is an ACE inhibitor used to treat hypertension. The nurse should withhold the dose of medication to prevent a further decrease in blood pressure. C - The nurse should obtain a sputum culture to identify the causative infectious agent prior to administering an antibiotic for a client who has a productive cough. However, this is not the priority action for the nurse to take. D - The nurse should administer erythromycin, a broad-spectrum antibiotic, for a client who has an infection. However, this is not the priority action for the nurse to take.

A nurse is participating in disaster planning for an acute care facility. When using the color-coded triage tag system, which of the following tag colors should the nurse assign to a client who has an open fracture? A - Red B - Yellow C - Green D - Black

B - Yellow Rational A - The nurse should issue a red tag to clients whose injuries are emergent, including those who have life-threatening injuries, such as hemorrhagic shock or airway compromise. Clients who have a red tag typically have critical injuries that require immediate intervention to preserve life. B - The nurse should issue a yellow tag to clients whose injuries are urgent, such as open fractures or large wounds. Clients who have a yellow tag can wait a short time for treatment, but that treatment should occur within 30 min to 2 hr. C - The nurse should issue a green tag to clients whose injuries are nonurgent, such as those who have contusions, sprains, strains, and closed fractures. Clients who have a green tag have injuries that can wait more than 2 hr for treatment without serious consequence. D - The nurse should issue a black tag to clients who are not expected to live, even with extensive intervention. These clients are given the lowest priority and are allowed to die naturally. The nurse should also place a black tag on clients who have already died.

A nurse is reviewing the ABG findings of a client who has COPD and was admitted with pneumonia. Which of the following AGB results should the nurse anticipate? A - pH 7.48; PaCO2 38 mm Hg; HCO3- 32 mEq/L B - pH 7.33; PaCO2 58 mm Hg; HCO3- 25 mEq/L C - pH 7.52; PaCO2 22 mm Hg; HCO3- 22 mEq/L D - pH 7.32; PaCO2 38 mm Hg; HCO3- 20 mEq/L

B - pH 7.33; PaCO2 58 mm Hg; HCO3- 25 mEq/L Rational A - When the pH and HCO3- are both above the expected reference ranges of pH 7.35 to 7.45 and HCO3- 22 to 26 mEq/L, and the PaCO2 is within the expected reference range of 35 to 45 mm Hg, the nurse should identify the client's condition as metabolic alkalosis, which can be caused by frequent vomiting or excessive nasogastric suctioning. B - The nurse should identify that the client's pH is below the expected reference range of 7.35 to 7.45, indicating acidosis. The PaCO2 is above the expected reference range of 35 to 45 mm Hg, and the HCO3- is within the expected reference range of 22 to 26 mEq/L, indicating respiratory acidosis. With COPD, smooth muscle in the lungs constricts, causing the airways to narrow and the retention of CO2, thereby lowering the pH. The effects are worsened with respiratory infection. Other causes of respiratory acidosis include airway obstruction and respiratory depression. C - When the pH is above the expected reference range of 7.35 to 7.45 and the PaCO2 is below the expected reference range of 35 to 45 mm Hg, the nurse should identify the client's condition as respiratory alkalosis, which can be caused by hyperventilation from severe anxiety or pain. D - When the pH and HCO3- are both below the expected reference ranges of pH 7.35 to 7.45 and HCO3- 22 to 26 mEq/L, and the PaCO2 is within the expected reference range of 35 to 45 mm Hg, the nurse should identify the client's condition as metabolic acidosis, which can be caused by DKA or severe diarrhea.

A nurse on an inpatient unit is assisting with a group therapy session. During the session, a client begins to shout, using aggressive language. Which of the following statement should the nurse make to the client? A-"Why do you feel the need to speak this way to others in the group?" B- "When you raise your voice, it makes me feel uncomfortable and unsafe." C - "You are frightening others in the group when you show your anger." D. -"Why are you attending group therapy but not respecting the feelings of others?"

B- "When you raise your voice, it makes me feel uncomfortable and unsafe."

A nurse in a community center is reinforcing teaching about poison control with a group of parents. A parent asks what to do if a child ingests a large quantity of acetaminophen. Identify the sequence of actions the nurse should recommend to the parent. A - Identify the medication and dosage strength B- Determine if the child is breathing C - Empty the child's mouth of remaining pills and residue D - Call a poison control center

B- Determine if the child is breathing C - Empty the child's mouth of remaining pills and residue A - Identify the medication and dosage strength D - Call a poison control center

A nurse is reinforcing discharge instructions with a client who is postoperative following a right hip arthroplasty. Which of the following statements should the nurse make? A - "You may cross your legs in 60 days." B - "Avoid lying on your operative side." C - "Avoid bending your hips more than 90 degrees." D - "You may sleep on a soft mattress."

C - "Avoid bending your hips more than 90 degrees."

A nurse is reinforcing teaching with a client who has rheumatoid arthritis and a new prescription for methylprednisolone. Which of the following information should the nurse include? A - "Take the medication each evening." B - "Take the medication on an empty stomach." C - "Avoid individuals who are sick." D - "Weigh yourself each week to monitor for weight loss."

C - "Avoid individuals who are sick." Rational A - The nurse should instruct the client to take the medication each day between 0600 and 0800, when the body's natural release of steroids occurs, to decrease the risk for adrenal insufficiency. B - The nurse should instruct the client to take the medication with food to decrease gastric distress or black stools, which is an indication of bleeding in the gastrointestinal tract. C - The nurse should instruct the client to avoid individuals who are sick and report manifestations of infection, such as a fever or sore throat, to the provider because glucocorticoids can suppress the immune system. A suppressed immune system can increase the risk of contracting an illness from a microorganism. D - The nurse should instruct the client to weigh themselves daily to monitor for weight gain. The client can have significant weight gain with prolonged use of a corticosteroid, so weight gain should be reported to the provider. The nurse should also instruct the client to monitor for and report swelling or edema to extremities, bruising, abdominal pain, dark, tarry stools, and shortness of breath.

A nurse is preparing to administer a client's daily blood pressure medication. The client states, "That looks different from the pill I usually take." Which of the following responses should the nurse make? A - "You should skip this dose of your medication if you don't recognize the pill." B - "Your provider prescribed this medication." C - "Describe what your usual pill looks like." D - "The medication is the same, but this facility uses a different manufacturer."

C - "Describe what your usual pill looks like." Rational A - Rather than recommending that the client skip the dose of medication, the nurse should determine why the pill has a different appearance than the client expected by verifying that it is the correct medication. B - This response is dismissive of the client's concerns and does not allow the nurse to collect further information about the client's usual medication therapy. The nurse should verify that this is the correct medication and dose. C - The nurse should collect more data prior to administering the medication. It is possible that the pill the nurse is providing is not the correct dose or medication. If the medication is different from the client's home medication, the nurse should clarify the prescription with the provider in order to ensure safe and effective administration of therapy. D - This response is dismissive of the client's concerns. The nurse should not provide information until the situation is more completely explored to identify the cause of any confusion. The nurse should verify that the pill being provided is the correct medication and dose.

A nurse is reinforcing teaching with the guardian of a newborn who was recently circumcised. Which of the following statements should the nurse include? A - "Wash the penis with water and mild soap twice a day." B - "Apply the diaper tightly until the penis is healed." C - "Do not remove the yellow film that will form on the penis." D - "Use baby wipes to clean urine from the penis."

C - "Do not remove the yellow film that will form on the penis." Rational A - The nurse should instruct the guardian to cleanse the penis gently with plain warm water to remove urine and feces. Soap can cause pain at the circumcision site and disrupt healing. B - The nurse should instruct the guardian to apply the diaper loosely over the penis. This will prevent pressure on the circumcision site, which decreases pain and promotes healing. C - The nurse should instruct the guardian that a yellow exudate will form over the circumcision site within 24 hr of the circumcision. The exudate will last 2 to 3 days and should not be removed because this can cause bleeding and pain and disrupt healing. D - The nurse should instruct the guardian not to use baby wipes to clean the penis because they might contain alcohol, which can cause pain at the circumcision site and disrupt healing.

A nurse is collecting data from a client who is having difficulty coping with the death of his child. Which of the following questions by the nurse is the priority? A - "What events led you to seek help?" B - "Who can you turn to for support?" C - "Do you think about harming yourself?" D - "Which activities help you to have better days?"

C - "Do you think about harming yourself?"

A nurse is assisting with a mental status examination for who has schizophrenia. Which of the following statements should the nurse make to gather information about the client's ability to think abstractly? A - "Subtract 7 from 100 and then continue subtracting 7 from each answer." B - "What do you think about when you are angry?" C - "How is an orange similar to an apple?" D - "Tell me about a vacation you took when you were a child growing up."

C - "How is an orange similar to an apple?"

A charge nurse is reinforcing teaching about ethical principles with a group of nurses. Which of the following statements by one of the nurse indicates an understanding of the ethical principle of justice? A - "I should respect a client's decisions regarding their treatment." B - "I should avoid causing unintentional harm to all clients." C - "I should divide my time among my clients." D - "I should keep the promises I make to clients."

C - "I should divide my time among my clients." Rational A - Respecting a client's decisions regarding treatment upholds the ethical principle of autonomy. The nurse should include the client when making decisions regarding care. The nurse does not need to agree with a client's choices but should respect the client's decisions. B - Doing no harm to clients upholds the ethical principle of nonmaleficence. The nurse has a duty not to harm clients, either intentionally or unintentionally, while providing care. C -Dividing time among clients upholds the ethical principle of justice. The nurse should plan care to share time as equally as possible to best meet all clients' needs. D - Keeping promises and being faithful to agreements made to clients upholds the ethical principle of fidelity. The nurse should be responsible for following up on promises made to clients to maintain trust in the nurse-client relationship.

A nurse is reinforcing teaching about injury prevention with the guardian of an infant. Which of the following statements bby the guardian indicates an understanding of teaching? A - "I should lightly shake talcum powder on my baby's skin after each diaper change." B - "I should use a drop-side crib after my baby is 6-months-old." C - "I should make sure my baby's clothing does not have buttons on it." D - "I should ensure the crib slats are no more than 3 inches apart."

C - "I should make sure my baby's clothing does not have buttons on it."

A charge nurse in a long-term care facility is assisting in the orientation of a newly licensed nurse. When discussing equipment and supply use, which of the following statements by the newly licensed nurse indicates an understanding of the teaching? A - "I should sterilize blood pressure cuffs after each use." B - "I will store extra supplies in the client's room in case I need them." C - "I should tag any equipment not in working order and notify my supervisor." D - "I will keep the client's urinal on the overbed table for convenience."

C - "I should tag any equipment not in working order and notify my supervisor." Rational A - It is not necessary for noncritical items like blood pressure cuffs, stethoscopes, and bedside trays to be sterilized. The nurse should ensure that noncritical items are disinfected. B - To provide cost-effective care and maintain an organized care environment, the nurse should return unopened supplies to designated storage areas and should not store them in the client's room. C - The nurse should tag malfunctioning equipment to prevent the use of the equipment by other staff members. The nurse should then notify the supervisor or the biomedical department to have the equipment replaced or repaired. D - The nurse should not place the client's urinal or bedpan on the overbed table because the table is used for meal trays, toiletry items, and performing procedures. The overbed table should be cleaned with an antiseptic cleaner prior to use.

A nurse is reinforcing teaching with an adolescent client regarding testicular self-examination. Which of the following statements by the client demonstrates an understanding of the teaching? A - "I will perform the exam before I shower." B - "I will check my testicles every 6 months." C - "I understand that testicular cancer is painless." D - "I understand that pea-sized lumps are normal."

C - "I understand that testicular cancer is painless."

A nurse is caring for a client who is at 19 weeks of gestation following an amniocentesis. The client tells the nurse, "I am really worried something might be wrong with my baby." Which of the following responses should the nurse make? A - "If I were you, I would focus on having positive thoughts." B - "Worrying about the results of your amniocentesis is not good for you." C - "I understand that you are worried. Let's discuss your concerns." D - "You shouldn't worry so much. Everything will be all right."

C - "I understand that you are worried. Let's discuss your concerns." Rational A - This is an example of nontherapeutic communication because the nurse is giving advice and minimizing the client's feelings. This response assumes the nurse knows best and discourages further communication between the client and the nurse. B - This is an example of nontherapeutic communication. This response assumes the nurse knows best and discourages further communication between the client and the nurse. C - This is an example of therapeutic communication and validates the client's feelings and focuses on their concerns. This response encourages further communication between the client and the nurse. D - This is an example of nontherapeutic communication; it is an example of false reassurance, which discourages further communication between the client and the nurse.

A nurse is reinforcing teaching with a new parent about the prevention of newborn abduction. Which of the following statements by the parent indicates an understanding of the teaching? A -"Some assistive personnel may not have name badges." B - "A nurse will carry my baby back to the nursery in his arms for routine care when it is needed." C - "I will ask the nurse to take care of my baby in the nursery if I need to take a nap." D - "I can remove my baby's security band if she is in my room."

C - "I will ask the nurse to take care of my baby in the nursery if I need to take a nap."

A nurse is reinforcing teaching about glucose monitoring with a client who has type 1 diabetes mellitus. Which of the following client statements indicated an understanding of the teaching? A- "I will check my urine once per day for glucose." B - "I will notify my doctor if my blood glucose is more than 120 milligrams per deciliter before meals." C - "I will check my blood glucose every 4 hours when I am sick." D - "I will treat myself for hypoglycemia if I notice a fruity breath odor."

C - "I will check my blood glucose every 4 hours when I am sick." Rational A - Urine glucose testing is much less precise than blood glucose testing. Fluid intake, urine elimination patterns, and some medications can affect the results. The test is used for a quick screening, but the client should not use it for diabetic management. The client should perform urine ketone testing during acute illness, stress, or when manifestations of hyperglycemia are present. B - The client should have a preprandial (premeal) blood glucose of 70 to 140 mg/d. The nurse should instruct the client to notify the provider if blood glucose levels are higher than 200 mg/dL. C - The client should check blood glucose levels every 2 to 4 hr when sick. The client should also continue to take insulin or oral antidiabetic agents, consume 4 oz of sugar-free, noncaffeinated liquid every 30 min, and meet carbohydrate needs through soft food if possible. D - The nurse should reinforce with the client that fruity or acetone breath is caused by elevated blood glucose in ketoacidosis. Other manifestations of ketoacidosis include Kussmaul respirations, paresthesias, and hypotension. Intravenous rehydration, insulin therapy, and treatment of electrolyte imbalance are required for ketoacidosis.

A nurse is reinforcing teaching with a client who has cancer about reducing the risk of infection. Which of the following client statements indicates an understanding of the instructions? A -"I will eat a salad daily to increase my intake of vitamin K." B - "I can work in my flower garden as long as I wear gardening gloves." C - "I will clean my toothbrush using a hydrogen peroxide solution." D - "I should take a shower every other day using antibacterial soap."

C - "I will clean my toothbrush using a hydrogen peroxide solution." Rational A - The nurse should instruct the client to avoid eating salads, or any kind of raw fruits or vegetables, because these can contain micro-organisms and place the client at risk for developing an infection. B - The nurse should instruct the client to avoid working with soil in the garden, or with potted plants, because this type of exposure increases the client's risk for developing an infection. C - The nurse should instruct the client to clean toothbrushes using a hydrogen peroxide solution, bleach, or by running them through the dishwasher. This will decrease the client's risk for developing an infection. D - The nurse should instruct the client to take a bath or shower daily using an antimicrobial soap. Showering every other day using antibacterial soap increases the client's risk for developing an infection

A nurse is reinforcing teaching about glycosylated hemoglobin (HbA1c) testing with a client who has diabetes mellitus. Which of the following statements indicates that the client understands the teaching? A- "The HbA1c test should be performed 2 hr after I eat a meal that is high in carbohydrates." B - "The HbA1c test can help detect the presence of ketones in my body." C - "I will have my HbA1c checked twice per year." D - "I will plan to fast before I have my HbA1c tested."

C - "I will have my HbA1c checked twice per year."

A nurse is reinforcing teaching with a client who has asthma and a new prescription for salmeterol using a dry powder inhaler. Which of the following statements by the client demonstrates an understanding of the teaching? A - "I will blow into the mouthpiece to activate the inhaler." B - "I will shake the inhaler for 3 to 5 seconds prior to using it." C - "I will hold my breath for 10 seconds after I remove the inhaler from my mouth." D - "I will rinse my inhaler once per week to keep it clean."

C - "I will hold my breath for 10 seconds after I remove the inhaler from my mouth." Rational A - The client should seal their lips around the mouthpiece and inhale to activate the inhaler. The client should avoid exhaling into the inhaler because moisture can cause the medication to clump. B - A client who is using a metered-dose inhaler should shake it prior to use, but this step is not necessary when the client is using a dry-powder inhaler. C - The client should hold their breath for 5 to 10 seconds after using the inhaler to promote absorption of the medication into the lungs. The client should also rinse their mouth and then spit to minimize the amount of medication that is absorbed systemically. D - The client should avoid getting the inhaler wet because it can cause the medication to clump. A client who uses a metered-dose inhaler should clean the plastic case and the cap of the inhaler under warm running tap water daily.

A nurse is reinforcing teaching regarding bladder retraining with a client who has urinary incontinence. Which of the following statements by the client indicates an understanding of the teaching? A - "I should go to the bathroom whenever I feel the urge to void." B - "I will increase my intake of drinks that contain citrus juices." C - "I will keep a diary of my voiding patterns each day." D - "I will limit my fluid intake between the hours of 10:00 a.m. and 2:00 p.m."

C - "I will keep a diary of my voiding patterns each day." Rational A - During initial bladder training, the client should go to the bathroom to void at regularly set intervals. Initially, the client should attempt to restrict voiding to once every 2 to 3 hr during waking hours and every 4 to 6 hr during the night. B - The client should avoid excessive intake of beverages containing caffeine, citrus juices, and artificial sweeteners to prevent urinary frequency and urgency. C - In order to help track the effectiveness of bladder retraining, the client should keep a diary of their voiding patterns each day. This will not only assist in evaluating the effectiveness of the retraining program but will make the client more aware of usual voiding times to help avoid instances of incontinence. D - The client should consume liquids between the hours of 0600 and 1800 to promote urinary continence. Limiting fluid intake overnight and taking prescribed diuretics in the early morning will help the client to prevent incontinence during the night.

A nurse is reinforcing teaching about home safety with an older adult client. Which of the following statements by the client indicates an understanding of the teaching? A - "I will run extension cords under area rugs." B - "I will place broken glass in a plastic bag for disposal." C - "I will paint the edge of each of my entry steps a different color." D - "I will keep my water heater set at 130 degrees Fahrenheit."

C - "I will paint the edge of each of my entry steps a different color."

A nurse is discussing therapeutic milieu with a nursing supervisor. Which of the following statements by the nurse demonstrates an understanding of therapeutic milieu? A - "I will make the decisions for the benefit of the group." B - "I should structure activities to minimize interactions between clients." C - "I will use negative behaviors as learning opportunities for clients." D - "I should restrict privileges for clients who break the rules."

C - "I will use negative behaviors as learning opportunities for clients." Rational A - In a therapeutic milieu environment, clients are encouraged to develop autonomy and problem-solving skills. This is done during group meetings in which the clients make decisions to benefit each individual client and the group as a whole. B - The therapeutic milieu environment is designed to promote interaction between clients and to increase self-confidence and social skills. The nurse should encourage clients to interact appropriately with other clients on the unit. C - In a therapeutic milieu environment, even negative or unplanned situations or behaviors can become positive learning opportunities. Within the milieu, clients learn to interact with others and respond in real situations. The nurse can model appropriate behavior and communication for clients, even when events seem unconstructive. D - In a therapeutic milieu environment, the client's behavior is corrected through the use of peer feedback and expectations of behavior within the group. While disruptive behaviors are addressed by the group members at the time they occur, restriction of privileges is not part of a therapeutic milieu environment.

A nurse in a long-term care center is caring for an adult client who has Alzheimer's disease and whose partner died several years ago. The client appears upset and asks the nurse when his partner will visit again. The nurse states, "It seems like you are feeling lonely. Let's take a walk outside and talk." Which of the following communication strategies is the nurse using? A - Reminiscence therapy B - Feedback C - Validation therapy D - Reflecting

C - Validation therapy

A nurse is collecting data from the adult child of a client who is concerned about their parent's development of dementia. Which of the following statements by the client's child is the priority? A- "Sometimes it takes a few minutes for my mother to find the right word." B - "This month, my mother forgot to pay her water bill." C - "Last week, I found my mother's cell phone in the microwave." D - "Yesterday my mother called her friend by the wrong name."

C - "Last week, I found my mother's cell phone in the microwave." Rational A - The inability to find the right word is nonurgent because it is an expected finding of aging due to a decrease in number of neurons and blood flow to the brain. Therefore, there is another statement that is the nurse's priority. B - Occasionally forgetting to pay a monthly bill is nonurgent because it is an expected finding of forgetfulness associated with aging. Therefore, there is another statement that is the nurse's priority. C - When using the urgent vs nonurgent approach to client care, the nurse should determine that the priority statement by the client's child is about the misplaced cell phone. The inability to find misplaced items and putting them in odd places can be a manifestation of serious memory deficit. D - Calling someone by the wrong name is nonurgent because it is an expected finding of aging due to a decrease in number of neurons and blood flow to the brain. Therefore, there is another statement that is the nurse's priority.

A nurse is reinforcing teaching regarding the immunization schedule of a newborn. Which of the following statements made by the parent should the nurse recognize as an understanding of the newborn's immunization schedule? A - "My baby should not have a hepatitis B vaccine if I test negative." B - "My baby will receive his first varicella vaccine at 6 months." C - "My baby will receive his next immunization when he is 2 months old." D - "I will need to start my baby's immunizations when I put him in day care."

C - "My baby will receive his next immunization when he is 2 months old."

A nurse is reinforcing teaching with a newly licensed nurse about the role of the nurse in informed consent. Which of the following statements bby the newly licensed nurse indicates an understanding of the teaching? A - "My signature on the consent form indicates I informed the client he can't change his mind about the procedure." B - "By signing the consent form, I confirm that I was present when the provider explained the procedure to the client." C - "My signature on the consent form indicates the client gave consent for the procedure voluntarily." D - "By signing the consent form, I confirm that the client's family approves of the procedure."

C - "My signature on the consent form indicates the client gave consent for the procedure voluntarily."

A nurse is reinforcing teaching about glucose monitoring with the parent of a child who has type 1 diabetes mellitus. Which of the following instructions should the nurse include in the teaching? A - "Press the platform of the lancet firmly against your child's finger." B - "Obtain the blood sample from the center of your child's finger pad." C - "Put your child's finger under warm, running water prior to collecting blood." D - "Steady the finger against a hard surface while puncturing the skin."

C - "Put your child's finger under warm, running water prior to collecting blood."

A nurse is documenting information in a client's electronic health record. Which of the following information should the nurse include? A - "The client slept well." B - "The client's status is unchanged." C - "The client refused his bedtime snack." D - "The client's vital signs are normal."

C - "The client refused his bedtime snack." Rational A - The nurse should use information that is objective and specific when documenting information in the client's electronic health record. The nurse should avoid using a vague term such as "well" when documenting the client's condition because it is a subjective term. B - The nurse should use information that is objective and specific when documenting information in the client's electronic health record. The nurse should avoid using a vague term such as "unchanged" when documenting the client's condition because it is subjective and nonspecific. C - The nurse should use information that is objective and specific when documenting a client's response to an intervention in the client's electronic health record. The nurse should record the client's response to interventions, such as refusing a snack, meal, or medication when offered. D - The nurse should use information that is objective and specific when documenting information in the client's electronic health record. The nurse should avoid using vague terms such as "normal" when documenting the client's condition.

A nurse is caring for a client who has a new diagnosis of terminal lung cancer. Which of the following statements by the client indicates the denial phase of the grief process? A - "The doctor has been so good to me. I know he has tried everything he can. It is just my time." B - "I can't believe that doctor graduated from medical school. He doesn't know a thing about treating cancer." C - "The doctor says I only have a few months to live, but I know he is exaggerating to get me to take my medication." D - "Even though I am not hurting right now, I don't feel like I have the energy to get out of bed."

C - "The doctor says I only have a few months to live, but I know he is exaggerating to get me to take my medication." Rational A - This client statement is an example of acceptance. In this phase of the grief process, clients accept what is happening and find ways to plan for the future. B - This client statement is an example of anger. In this phase of the grief process, clients lash out at other people or things. C - This client statement is an example of denial. In this phase of the grief process, clients have difficulty believing a terminal diagnosis or loss. D - This client statement is an example of depression. In this phase of the grief process, clients are saddened about their inability to change the situation.

A nurse in a prenatal clinic is caring for a client who is at 16 weeks of gestation and has a positive hepatitis b test result. Which of the following actions should the nurse take? A - Instruct the client to avoid crowds until a repeat hepatitis B test is negative. B - Tell the client that she will need to start the hepatitis B vaccine series after delivery. C - Explain to the client that she will receive the hepatitis B immune globulin immediately. D - Inform the client that hepatitis B cannot be transmitted to the fetus.

C - Explain to the client that she will receive the hepatitis B immune globulin immediately.

A nurse is reinforcing teaching with the parents of a toddler who has strabismus. Which of the following treatments should the nurse plan to include in the teaching? A - Corrective biconcave lenses B - Laser surgery C - Eye patch D - Artificial tears

C - Eye patch

A nurse is collecting data from a client who has major depressive disorder. Which of the following findings is the priority for the nurse to report to the provider? A - Inability to make decisions B - Anhedonia C - Feelings of hopelessness D - Fatigue

C - Feelings of hopelessness

A nurse is collecting data from a client who states, "I just don't know what to do about my partner's drinking. Every time I see my partner drinking beer, I get very anxious." Which of the following responses by the nurse is therapeutic? A- "Try not to feel so anxious about something that is your partner's problem, not yours." B - "At one time, you told me you were drinking with your partner. Are you continuing to do that?" C - "The next time your partner starts drinking, what is something you might do to decrease your anxiety?" D - "I think you should attend an Al-Anon meeting. It is a support group for people who are troubled by another person's drinking."

C - "The next time your partner starts drinking, what is something you might do to decrease your anxiety?" Rational A - The nurse is using nontherapeutic communication by being dismissive and minimizing the client's concerns. B - The nurse is using nontherapeutic communication by making a value judgment toward the client. The nurse's response implies that the client is doing something wrong. This type of response can cause the client to stop talking and not share anything else with the nurse. C - The nurse is using therapeutic communication in the form of an open-ended question to encourage the client to focus on identifying an alternate course of action for the situation in question. D - The nurse is using nontherapeutic communication by giving advice. This response discourages further expression of the client's feelings and takes away from the client's own ability to demonstrate decision-making.

A nurse is reinforcing teaching about nutrition with a guardian of a toddler. Which of the following statements by the guardian indicates an understanding of the instructions? A- "I should expect my child to have an increased appetite." B - "My child's average daily intake should be about 2,000 calories." C - "The quality of my child's food is more important than the quantity." D - "My child should drink at least 4 cups of milk each day."

C - "The quality of my child's food is more important than the quantity." Rational A - The nurse should reinforce that at about 18 months of age, most toddlers experience a decrease in appetite, a phenomenon known as physiologic anorexia. They can become picky, fussy eaters with strong taste preferences. B - The nurse should reinforce that toddlers experience a decrease in appetite and calorie requirements. The average recommended caloric intake for children who are 2 to 3 years of age is 1,000 to 1,400 calories per day. C - The nurse should reinforce that toddlers can be picky eaters and usually eat only one or two adequate meals each day. It is important that meals be balanced with essential nutrients. The nutritional quality of the food is much more important than the quantity. D - The nurse should reinforce that a toddler's milk intake should not exceed 2 or 3 cups per day. Toddlers can develop milk anemia due to a decreased consumption of solid food.

A nurse is reinforcing teaching about universal newborn screening with the guardians of a newborn who is 12 hr old. Which of the following information should the nurse include in the teaching? A - "This test checks for the presence of elevated bilirubin levels." B - "We will obtain the blood sample from a vein in your baby's arm." C - "We will perform the test at least 24 hours after your baby's first feeding." D - "This screening will be repeated when your baby is 1 month old."

C - "We will perform the test at least 24 hours after your baby's first feeding." Rational A - The nurse should reinforce that the universal newborn screening tests for genetic and metabolic diseases that could cause significant health conditions. This screening does not check for bilirubin levels. B - The nurse should reinforce that a capillary blood sample will be obtained from the newborn via a heel stick. C - The nurse should reinforce that the blood sample should be obtained at least 24 hr after the newborn's first feeding to ensure that the results are accurate. D - The nurse should reinforce that this screening might need to be repeated when the newborn is 1 to 2 weeks old if the initial specimen was collected prior to the newborn being 24 hr old.

A nurse is reinforcing home safety instructions with the parents of toddler. Which of the following parent statements indicates an understanding of the teaching? A - "We will keep our child out of the sun between 3 p.m. and 5 p.m." B - "We will transition our child to a toddler bed when he is 2 feet tall." C - "We will turn the pot handles toward the back of the stove." D - "We will provide a healthy snack of peanuts."

C - "We will turn the pot handles toward the back of the stove."

A nurse is performing a mental status examination for a client who has had a change in sensorium. Which of the following questions should the nurse ask to determine the client's ability to think abstractly? A - "What did you have for breakfast this morning?" B - "Who is the current president of the United States?" C - "Would you tell me the meaning of the saying, 'A picture is worth a thousand words?'" D - "What number do you get when you subtract seven from 100, and then subtract seven again from your first answer?"

C - "Would you tell me the meaning of the saying, 'A picture is worth a thousand words?'" Rational A - The nurse should ask the client about events from earlier in the day or from the past few days to test the client's recent memory. B - The nurse should ask the client questions about generally known facts to test the client's knowledge base, or general level of intellect. If the client has a low level of education, the nurse should ask a different question to match the client's expected knowledge base. C - The nurse should ask the client to explain the meaning of a common proverb to test their ability to think abstractly. Having the client describe the meaning of a proverb tests a higher-level thought process. It also gives the nurse the opportunity to listen to the client's speech patterns. This allows the nurse to evaluate whether the client's thought process is organized and coherent. D - The nurse should ask the client to subtract seven from 100 and then continue subtracting seven from the answer if the nurse wants to test the client's attention to a problem and ability to calculate. This process is often called "serial sevens."

A nurse is reinforcing teaching with a client who has gonorrhea. Which of the following information should the nurse include? A - "Your partner will not require treatment for this infection." B - "You can resume sexual activity as soon as you begin treatment." C - "You are at risk for infertility with this infection, regardless of treatment." D - "You will not be at further risk for this infection following treatment."

C - "You are at risk for infertility with this infection, regardless of treatment."

A nurse is assisting with the discharge planning for a client. Which of the following actions should the nurse plan to take? A - Include the client's vital sign record in the discharge instructions. B - Begin discharge planning 24 hr prior to the client's scheduled discharge date. C -Include community resource phone numbers with the client's discharge instructions. D - Obtain a 3-month supply of the client's prescribed medications.

C -Include community resource phone numbers with the client's discharge instructions.

A nurse is reinforcing teaching with a client who has schizophrenia and a new prescription for olanzapine. Which of the following information should the nurse include in the teaching? A - "You should expect a decreased appetite." B - "You should anticipate diarrhea following meals." C - "You should regularly monitor your blood glucose levels." D - "You should take a hot shower or bath to relieve muscle aches."

C - "You should regularly monitor your blood glucose levels." Rational A - An adverse effect of olanzapine is increased appetite and weight gain. The nurse should reinforce teaching with the client about weight management techniques, such as increasing activity and decreasing calorie intake, to help counteract the adverse effects of the medication. B - The nurse should reinforce with the client that olanzapine has several adverse effects that can affect the gastrointestinal system such as dry mouth, increased appetite, nausea, and constipation. However, diarrhea is not a finding associated with olanzapine use. C - The nurse should reinforce the importance of carefully monitoring blood glucose levels for clients who have schizophrenia and take olanzapine. One adverse effect of olanzapine is hyperglycemia; therefore, it is important for the client to monitor glycemic control while taking this medication. D - The nurse should reinforce with the client to avoid extremes in temperature because olanzapine alters the body's ability to regulate temperature. Exposure to hot showers or baths can result in heat-related injuries.

A nurse is reinforcing teaching with a client who requests hydrotherapy for pain management during labor. Which of the following statements should the nurse include? A - "You will have an internal fetal monitor applied prior to hydrotherapy." B - "You will need to limit your time in the tub to no more than 20 minutes." C - "You will need to be in active labor before using hydrotherapy." D - "You will need to keep the water temperature above 98.6 degrees Fahrenheit during hydrotherapy."

C - "You will need to be in active labor before using hydrotherapy."

A nurse is reinforcing teaching with a client who is at 11 weeks of gestation and has phenylketonuria (PKU). Which of the following statements should the nurse make? A - "An ultrasound can determine if your baby has PKU." B - "Your doctor will need to schedule a cesarean birth to deliver your baby." C - "Your phenylalanine level should be less than 6 milligrams per deciliter during pregnancy." D - "Consume at least two 8-ounce glasses of milk each day while pregnant."

C - "Your phenylalanine level should be less than 6 milligrams per deciliter during pregnancy." Rational A - An ultrasound can show structural abnormalities such as neural tube or cardiac defects in the fetus. An amniocentesis can diagnose PKU in the fetus by chromosomal analysis. All newborns are screened for PKU following delivery by obtaining a blood sample at least 24 hr after the first feeding. The blood is screened for high levels of phenylalanine, which would indicate phenylketonuria. B - It is not necessary that the provider schedule a cesarean birth to deliver the baby. A client who has PKU can safely deliver vaginally. PKU is a metabolic disorder that can lead to neurologic complications in the fetus. However, it has no impact on the method of delivery. C - The client's phenylalanine level should be 2 to 6 mg/dL throughout pregnancy. Elevated levels of phenylalanine increase the risk for congenital anomalies in the fetus, including microcephaly, heart defects, and cognitive impairment. D - The client should follow a low-protein diet to limit intake of phenylalanine. They should avoid milk and dairy products, meats, and eggs. The client should follow a meat-free diet and increase her intake of fruits and vegetables.

A nurse is reinforcing teaching about food sources that are high in folate with a group of women who are pregnant. Which of the following foods should the nurse recommend to this group as the best source of folate? A - 1 cup dried prunes B - 1/2 cup boiled potatoes C - 1/2 cup dried peas D - 1 cup grapes

C - 1/2 cup dried peas

A nurse has just receiving change-of-shift report for four children in a pediatric unit. Which of the following children should the nurse collect data from first? A - A child who is 2 days postoperative following an appendectomy and reports incisional pain B - A child who has a new diagnosis of diabetes mellitus and an HbA1c level of 7.5% C - A child who has a fever and nuchal rigidity D - A child who experienced a seizure 1 hr ago and is resting

C - A child who has a fever and nuchal rigidity

A nurse is collecting data from four adolescent clients at the beginning of a shift. Which of the following findings is the nurse's priority? A - A client who is 1 day postoperative and has a temperature of 37.9° C (100.2° F) B - A client who has a superficial partial-thickness burn injury to the right leg and is crying C - A client who has asthma and is agitated D - A client who had an open reduction internal fixation of an ankle fracture and reports a pain level increase from 3 to 6

C - A client who has asthma and is agitated Rational A - An elevated temperature can indicate infection. However, this finding is not the priority. B - Crying can indicate a change in emotional status or pain. However, this finding is not the priority. C - When using the airway, breathing, circulation approach to client care, the nurse should determine that the priority finding is a client who has asthma and is agitated, which can indicate hypoxia. The nurse should report this finding to the provider and monitor this client closely for changes in condition such as airway compromise. D - An increase in pain level can indicate complications from the surgery. However, this finding is not the priority.

A nurse is assisting with screening a group of clients of major depressive disorder (MDD). The nurse should identify that which of the following clients is at an increased risk for the development of MDD? A - A client who is newly employed B - A client who abstains from alcohol C - A client who just gave birth D - A client who has been married for 15 years

C - A client who just gave birth

A nurse is assisting with the orientation of an assistive personnel (AP). For which of the following clients should the nurse instruct the AP to use a gait belt? A - A client who has a history of falls B - A bariatric client who requires assistance with repositioning in bed C - A client who requires minimal assistance ambulating D - A client who has a bilateral foot amputation

C - A client who requires minimal assistance ambulating Rational A - A gait belt should not be used for clients who are at high risk for falls because it provides minimum support to stabilize the client. The nurse should use a lift with an ambulation sling to assist a client who has a history of falls. B - A gait belt should not be used on bariatric clients as it can cause injury to the client. C - A gait belt should be used for an ambulatory client who is weight-bearing and requires minimal assistance. The gait belt is used to stabilize the client when ambulating to reduce the risk for client injury. D - A gait belt should be used for a client who is weight-bearing and requires minimal assistance.

A nurse is caring for a group of clients who are postoperative. Which of the following should the nurse report to the charge nurse? A - A client who has diabetes mellitus and a capillary blood glucose level of 98 mg/dL B - A client who had a closed reduction of a right femur fracture and has a +2 pedal pulse in the right foot C - A client whose blood pressure decreased from 138/86 to 106/60 mm Hg in 4 hr D - A client who is 12 hr postoperative and reports a pain level of 3 on a scale from 0 to 10

C - A client whose blood pressure decreased from 138/86 to 106/60 mm Hg in 4 hr Rational A - A capillary blood glucose level of 98 mg/dL is within the expected reference range of less than 200 mg/dL for a casual blood glucose. Therefore, the nurse does not need to report this finding to the charge nurse. B - A +2 pedal pulse on the affected side is an expected finding. For a client who has a skeletal fracture, the nurse should report a weak (+1) or absent pulse. The nurse should also check the client's distal circulation, movement, and sensation to monitor for circulatory complications. C - A decrease in blood pressure is an unexpected finding and can be an indication of hemorrhage, hypovolemia, or decreased cardiac output. Therefore, the nurse should report this finding to the charge nurse for further evaluation. D - A pain level of 3 out of 10 is an expected finding for a client who is 12 hr postoperative. A pain rating of 1 to 3 indicates mild pain, 4 to 6 indicates moderate pain, and 7 to 10 indicates severe pain. The nurse should continue to administer pain medication as prescribed and notify the provider if pain is not controlled by analgesia.

A nurse is preparing to document client care in the electronic health record for a client who is postoperative. Which of the following should the nurse include in the documentation? A - A verbal prescription from the provider taken by the charge nurse B - Entry of the completion of a procedure in advance C - A treatment that was refused by the client D - Subjective findings regarding the client's pain tolerance

C - A treatment that was refused by the client

A nurse is assisting with the care of a child who has sickle cell anemia. The nurse should identify which of the following as the potential cause of pain associated with sickle cell crisis? A - Crescent-shaped hemoglobin decreasing the viscosity of the circulating blood B - Deficiency of iron resulting in rapid metabolism of hemoglobin C - Abnormal red blood cells that clump together and occlude blood flow D - Defective hemoglobin formation causing an overabundance of erythrocytes

C - Abnormal red blood cells that clump together and occlude blood flow Rational A - Abnormal hemoglobin chains caused by sickle cell anemia form a sickle shape, which increases, rather than decreases, the viscosity of circulating blood. B - A deficiency of iron that results in rapid metabolism of hemoglobin is iron-deficiency anemia. Iron is usually stored in hemoglobin, bone marrow, the spleen, the liver, and muscle. When a client is iron deficient, the iron stores are depleted first, followed by the hemoglobin stores. As a result, RBCs become microcytic, and the client displays manifestations of anemia, including weakness, pallor, and fatigue. C - The nurse should identify that pain is the most common clinical finding of sickle cell crisis and is the result of tissue injury caused by poor oxygenation from obstructed blood flow. The RBCs become a sickle shape and clump together, occluding blood flow and leading to tissue hypoxia. This is also known as a vasoocclusive crisis. D - Defective hemoglobin formation causing an overabundance of erythrocytes is descriptive of beta thalassemia, or Cooley anemia. With this disorder, there is a partial or complete deficiency in the synthesis of hemoglobin molecules, which results in a defective formation of hemoglobin. This polypeptide unit is unbalanced and unstable, which damages RBCs and causes severe anemia.

A nurse is reviewing the laboratory results of a client who has schizophrenia and is taking clozapine. Which of the following findings should the nurse identify as an adverse effect of the medication? A - Glycosylated hemoglobin 4.3% B - Low-density lipoprotein (LDL) 100 mg/dL C - Absolute neutrophil count (ANC) 800/mm3 D - Platelets 175,000/mm3

C - Absolute neutrophil count (ANC) 800/mm3 Rational A - The nurse should identify that clozapine is a second-generation antipsychotic medication that can cause hyperglycemia and diabetes mellitus. Glycosylated hemoglobin provides an indication of the average blood glucose levels of a client over a period of 4 months. The client's glycosylated hemoglobin level is within the expected reference range of 4% to 5.9% and does not indicate the presence of long-term hyperglycemia. B - The nurse should identify that clozapine is a second-generation antipsychotic medication that can cause dyslipidemia. Low-density lipoproteins (LDL) carry cholesterol through the body and increased LDL levels place the client at risk for the development of arteriosclerosis and peripheral vascular disease. However, the client's level is within the expected reference range of less than 130 mg/dL and does not indicate dyslipidemia. C - The nurse should identify that an absolute neutrophil count (ANC) 800/mm3 is less than the expected level of greater than or equal to 1,500/mm3. Clozapine is a second-generation antipsychotic medication that can cause agranulocytosis, or a decreased level of neutrophils. The nurse should monitor the client for indications of infection, including an elevated temperature or a sore throat, and report these to the provider. D - The nurse should identify that the client's platelet level is within the expected reference range of 150,000 to 400,000/mm3. Medications such as fluoxetine, a selective serotonin reuptake inhibitor used in the treatment of depression, can result in low platelet levels and pose an increased risk for bleeding. However, this is not an adverse effect associated with clozapine.

A nurse on a mental health unit is prioritizing care for a group of clients. Which of the following actions should the nurse take first? A - Reinforce teaching about the importance of participating in group therapy for a client who has major depressive disorder. B - Administer lamotrigine to a client who has bipolar disorder and is pacing in the hallway. C - Administer haloperidol to a client who has schizophrenia and is yelling at other clients. D -Reinforce teaching on assertive behaviors for a client who has dependent personality disorder and is asking the nurse for help.

C - Administer haloperidol to a client who has schizophrenia and is yelling at other clients.

A nurse is providing palliative care to a client who is in the active phase of dying. Which of the following actions should the nurse take? A - Encourage the family to offer fluids to the client frequently. B - Provide nasotracheal suctioning to the client who has accumulated secretions. C - Administer morphine when the client displays manifestations of dyspnea. D - Whisper when communicating with the family.

C - Administer morphine when the client displays manifestations of dyspnea. Rational A - The nurse should not encourage the family to offer fluids to the client frequently because a client who is near the end of life has a depressed gag reflex and is at risk for aspiration. Clients who are near the end of life frequently experience anorexia and have a decreased desire for fluids. B - Nasotracheal suctioning can increase client discomfort. The nurse should administer a medication, such as a scopolamine patch, to reduce the client's secretions and promote quiet breathing. C - A client who is in the active phase of dying will often feel short of breath and possibly anxious. Administering morphine can relieve the feeling of dyspnea and anxiety, which are manifestations that can frighten the client and the family. D - A client who is actively dying often retains the ability to hear. The nurse should include the client in all conversations when at their bedside. The nurse should assume the client can hear and never engage in a conversation with the family by whispering.

A nurse is meeting a client on a mental health unit for the first time. Which of the following actions should the nurse take to establish a therapeutic relationship with the client? A - Provide sympathy during interaction. B - Focus on the client's verbal rather than nonverbal cues. C - Adopt an open posture when speaking to the client. D - Give personal opinions when the client asks.

C - Adopt an open posture when speaking to the client. Rational A - Sympathy involves a subjective look into a client's world. If a nurse over-identifies with a client, it can impede the nurse's ability to help the client work through the problem or situation being addressed. B - Therapeutic communication includes both verbal and nonverbal cues, the majority of which are nonverbal. Although the nurse should listen to the client's verbal communication, the nurse should also focus on the client's nonverbal communication to best understand the client's full meaning. C - A therapeutic relationship between the nurse and the client can assist the client in making positive changes, which then enable the client to better function in everyday life and in society. One of the ways to promote a therapeutic relationship with the client is to engage in active listening techniques such as facing the client during conversations, maintaining an open posture with arms and legs uncrossed, leaning in toward the client to convey interest in what the client is saying without invading the client's personal space, and being relaxed during interactions with the client. D - The nurse should refrain from giving personal opinions or discussing personal feelings and making judgments about the client's beliefs and feelings. Making personal value statements can prevent the client from expressing their thoughts and beliefs, which can lead to feelings of inadequacy, indecision, and dependence.

A nurse in a mental health facility is collecting data from a client who has schizophrenia. The nurse should identify that which of the following findings is referred to as a negative symptom of schizophrenia? A - Delusions B - Echolalia C - Apathy D- Paranoia

C - Apathy

A nurse is assisting with the care of a client who is in labor and is experiencing umbilical cord prolapse. Which of the following actions should the nurse take? A - Raise the head of the client's bed to 45°. B - Prepare to collect a fetal scalp blood sample. C - Apply warm saline compresses to the exposed cord. D - Apply oxygen at 3 L/min via nasal cannula.

C - Apply warm saline compresses to the exposed cord. Rational A - The nurse should not raise the head of the client's bed because this will increase the pressure on the umbilical cord. The nurse should place the client in a Trendelenburg or knee-chest position. B - The nurse should assist with fetal scalp blood sampling when there are concerns that the fetus may be acidotic. This would not be an appropriate intervention for a prolapsed umbilical cord. C - The nurse should apply warm saline compresses to prevent the cord from drying and the blood vessels from constricting which would decrease blood flow to the fetus. D - The nurse should apply oxygen at 8 to 10 L/min via nonrebreather face mask.

A nurse is collecting data from a client who is 1 day postpartum. Which of the following findings should the nurse report to the provider? A - A gush of dark blood when the client stands ​B - Increased urine output C - Area of warmth on the left calf ​D - Fundal height 1 cm below the umbilicus

C - Area of warmth on the left calf Rational A - The nurse should expect the client to experience a gush of vaginal bleeding upon standing. The lochia pools in the vagina while the client lies in bed. B - The nurse should expect the client to experience an increased urine output, due to postpartal diuresis. The client might have a urine output greater than 3 L per day for 2 to 3 days. C - The nurse should report an area of warmth, tenderness, or redness on the client's calf to the provider. This is a manifestation of thrombophlebitis, which is inflammation of the wall of a vein that often causes a clot to form. There is an increased incidence of thrombophlebitis in clients who are pregnant. D - The nurse should expect the fundus to be located 1 cm below the umbilicus 1 day after birth. The fundus descends 1 to 2 cm every 24 hr.

A hospice nurse is providing care for a client who is actively dying. Which of the following actions should the nurse take? A - Place the client in a supine position. B - Encourage the client to increase calorie intake. C - Ask if the client prefers to have the room lights on or off. D - Speak with an increased voice volume.

C - Ask if the client prefers to have the room lights on or off. Rational A - The nurse should place the client in a lateral position if the client is unconscious, or in Fowler's position if the client is conscious, to maintain an open airway. B - The nurse should encourage the client to take liquids as tolerated. The client might have altered nutrition, anorexia, or feelings of nausea. Therefore, the nurse should never force the client to eat. C - The nurse should ask if the client prefers to have the lights in the room remain on or off as a palliative measure. Some clients who are actively dying prefer a dark environment while others prefer to be able to easily see their surroundings. D - The nurse should speak to the client clearly and at a normal volume, without either whispering or speaking loudly. Hearing usually remains unchanged for the client while dying and can remain unchanged until death.

A nurse observes two assistive personnel (AP) at a client's bedside disagreeing about the way to bathe a client. Which of the following actions should the nurse take? A - Ask the client if they want a bath. B - Tell the AP to proceed with the client's bath. C - Ask the AP to speak to the nurse outside the client's room. D - Request assistance from a security officer.

C - Ask the AP to speak to the nurse outside the client's room. Rational A - It is not appropriate to ask the client if they want a bath while the staff is having a conflict. The conflict should be resolved without involving the client. B - Instructing the AP to proceed with the client's bath is not in the best interest of the client.The conflict must be addressed, and without resolution, the conflict might have a negative impact on the client. C - The nurse should remove the AP from the client's room and use active listening to resolve the conflict. D - There is no indication that a security officer needs to be involved in the conflict.

A nurse is reinforcing teaching with a newly licensed nurse about the administration of opioid pain medication. Which of the following instructions should the nurse include? A - Count the remaining narcotics after removing the client's medication from the locked dispenser. B - Place the unused portion of the client's medication in the sharps container at the nurses' station. C - Have a second nurse witness disposal of the unused portion of the client's medication. D - Report a discrepancy in the narcotic count immediately after administering the medication to the client.

C - Have a second nurse witness disposal of the unused portion of the client's medication.

A nurse is preparing to administer medications to an adolescent client. Which of the following actions should the nurse take to identify the client? A - Ask the client to state their room number. B - Ask the client to state their mother's maiden name. C - Ask the client to state their telephone number. D - Ask the client to state their home address.

C - Ask the client to state their telephone number. Rational A - The client's room number is not an acceptable identifier for the client because the room does not belong to the client. It is possible that a staff member might have transferred the client from their original room to a new room with a different room number, the client might be ambulatory and not in their own room, or another client might be visiting the client's room. B - The name of the client's parent is not an acceptable identifier because the parent can have multiple children. An identifier collected prior to medication administration must be person-specific. C - Appropriate client identifiers include the client's telephone number, name, photographic identification, facility identification number, date of birth, and other person-specific identifiers. The nurse should use at least two identifiers to verify the client's identity before administering medications. D - The client's home address does not belong solely to the client. The client's home address is not an accepted client identifier when administering medications.

A nurse is collecting data from a client who recently began taking gabapentin. Which of the following manifestations should the nurse identify is an adverse effect of gabapentin? A - Weight loss B - Hyperactivity C - Ataxia D - Dry mouth

C - Ataxia Rational A - Weight gain, not weight loss, is an adverse effect of gabapentin. B. - Somnolence, not hyperactivity, is an adverse effect of gabapentin. C - Ataxia, or impaired coordination, is a common adverse effect of gabapentin. D - Gingivitis, not dry mouth, is an adverse effect gabapentin.

A nurse is reinforcing teaching about smoking cessation with a client who wants to use 4-mg strength nicotine chewing gum. Which of the following instructions should the nurse include? A - Chew the nicotine gum quickly for maximum effect. B - Begin by chewing about 25 pieces of gum per day. C - Avoid eating or drinking for 15 min before chewing the gum. D - Continue using the nicotine gum for 1 year.

C - Avoid eating or drinking for 15 min before chewing the gum. Rational A - The client should chew the gum slowly for about 30 min to avoid releasing too much nicotine at once. B - The average dosage is 9 to 12 pieces of gum per day, with a maximum of 24 pieces per day. C - The client should not eat or drink anything for 15 min before chewing the gum to avoid interfering with the absorption of nicotine. D - After 3 months, the client should start tapering nicotine gum use. The client should not use the gum for more than 6 months.

A nurse is collecting data form a 12-month-old infant during a well-child visit. Which of the following findings should the nurse report to the provider? A - Heart rate 130/min B - Respiratory rate 30/min C - BP 115/70 mm Hg D - Temperature 37.5° C (99.5° F)

C - BP 115/70 mm Hg

A nurse is assisting with the care of a group of a children on a pediatric unit. Which of the following tasks should the nurse delegate to an assistive personnel (AP)? A- Educating an adolescent with diabetes how to perform a finger stick B - Giving telephone advice to the parent of an infant who has a high fever C - Bathing and bottle feeding an infant who is on the unit for antibiotic therapy D - Changing a sterile dressing for a child who had an appendectomy

C - Bathing and bottle feeding an infant who is on the unit for antibiotic therapy Rational A - Providing education to a client is a task that is outside the range of function of an AP. B - Giving phone support to a parent is a task that is outside the range of function of an AP. C - Bathing and feeding an infant is a task that is within the range of function of an AP. Other tasks the nurse can assign to an AP include measuring intake and output, toileting, changing bed linens, and obtaining vital signs on stable clients. D - Changing a sterile dressing is a task that is outside the range of function of an AP. This task requires use of the nursing process and must be performed by a nurse.

A nurse is collecting data from a 12-month-old infant during a well-child visit. The nurse should identify which of the following findings as a deviation from expected growth and development? A - Vocabulary of three words B - Negative Babinski reflex C - Birth weight doubled D - Unable to build a two-block tower

C - Birth weight doubled

A nurse is collecting data from a client who has hypothyroidism. Which of the following manifestations should the nurse anticipate? A - Blurred vision B - Insomnia C - Bradycardia D - Weight loss

C - Bradycardia

A nurse is caring for a client who is pregnant and has a prescription for nifedipine. Which of the following outcomes should the nurse expect from this medication? A - Fetal lung maturity B - Maternal blood glucose control C - Cessation of uterine contractions D - Resolution of maternal nausea

C - Cessation of uterine contractions

A nurse is reinforcing teaching with the family of a client who has a cervical injury and has a halo vest in place. Which of the following safety precaution should the nurse include in the teaching? A - Clean the pin sites every 72 hr. B - Use the halo ring to reposition the client when in bed. C - Change the sheepskin liner weekly. D - Tighten the traction bar as needed.

C - Change the sheepskin liner weekly.

A nurse is collecting data from a toddler who has gastroesophageal reflux disease (GERD). Which of the following findings should the nurse expect? A - Abdominal distention B - Constipation C - Chronic cough D - Decreased bowel sounds

C - Chronic cough

A nurse is assisting in the care of a client who has a serum potassium level of 5.3 mEq/L. Which of the following medications should the nurse plan to administer? A - Lisinopril B - Digoxin C - Furosemide D - Spironolactone

C - Furosemide Rational A - Lisinopril is an angiotensin converting enzyme (ACE) that inhibits the production of aldosterone, which can cause the kidneys to retain potassium. As a result, the client's serum potassium levels can be elevated. B - Potassium competes with digoxin in binding to other electrolytes and cells. When the client's potassium level is elevated, it decreases the effectiveness of digoxin. If potassium levels are low, digoxin toxicity can occur. C - This client's potassium level indicates hyperkalemia; therefore, the nurse should plan to administer furosemide. Furosemide is a loop diuretic that promotes excretion of potassium. D - Spironolactone is a potassium-sparing diuretic that causes retention of potassium. Because the client has an elevated serum potassium level, the nurse should not administer this medication.

A nurse is administering medications to a client through a nasogastric tube. Which of the following actions should the nurse take? A - Mix the medications with chilled fluid before administering. B - Administer the medications through the pigtail air vent on the nasogastric tube. C - Clamp the nasogastric tube before attaching the syringe to administer the medications. D - Mix the medications together in one syringe before administering.

C - Clamp the nasogastric tube before attaching the syringe to administer the medications. Rational A - The nurse should warm fluid to room temperature to mix the medications before administration. Cold fluids can cause abdominal cramping in the client. B - The nurse should not administer the medications through the pigtail air vent on the nasogastric tube to prevent clogging the air vent. The pigtail air vent prevents the nasogastric tube from adhering to the client's gastric wall and causing damage to gastric mucosa. C - The nurse should clamp or pinch the nasogastric tube before attaching the syringe to administer the medications and before removing the syringe following administration to prevent air from going into the client's stomach and causing abdominal discomfort. D - The nurse should administer each medication separately to prevent altering the medication's composition and interfering with the medication's efficacy.

A charge nurse is supervising a newly licensed nurse who is caring for a client who is in skeletal traction and requires pin site care. For which of the following actions by the newly licensed nurse should the charge nurse intervene? A - Checks the pin sites every 8 hr for infection B - Administers an analgesic prior to pin care C - Cleanses each pin site with a povidone-iodine solution D - Plans to cleanse the pin sites once daily.

C - Cleanses each pin site with a povidone-iodine solution Rational A - The nurse should check the pin sites every 8 to 12 hr for indications of pin site loosening or of infection, such as drainage, redness, and swelling. Pins placed in areas of soft tissue have increased incidence of infection. B - The nurse should administer an analgesic 30 min prior to performing pin care to decrease pain. The client should also receive analgesia before performing exercise, such as range of motion of unaffected joints. C - The nurse should clean pin sites daily with a chlorhexidine solution using aseptic technique to reduce the risk for infection. The nurse can use 0.9% sodium chloride solution to cleanse the pin sites if chlorhexidine is contraindicated for the client. D - The nurse should clean the pin sites daily using chlorhexidine solution. Drainage from the pins is particularly heavy during the first 48 to 72 hr postoperatively.

A nurse is caring for a client who has a history of breast cancer. The client asks the nurse about birth control. Which of the following methods of birth control is contraindicated for this client? A - Intrauterine device B - Latex condom C - Combination oral contraceptives D - Contraceptive sponge

C - Combination oral contraceptives

A nurse is caring for a client who has supine blood pressure of 86/58 mm Hg and reports feeling dizzy. Which of the following actions should the nurse take? A - Request a prescription for a diuretic from the provider. B - Ask an assistive personnel to recheck the blood pressure in 2 hr. C - Compare the result to the client's baseline. D - Assist the client to a sitting position.

C - Compare the result to the client's baseline. Rational A - A diuretic is a medication that reduces reabsorption of water by the kidneys, which lowers circulating fluid volume. This medication would further decrease client's blood pressure. B - The client's blood pressure requires further assessment and intervention by the nurse as it is below the expected reference range. Hypotension with dizziness increases the client's risk for injury due to a fall. C - Comparing baseline measurements of the client's blood pressure provides a reference for identification and accurate interpretation of any noted changes. D - The client's blood pressure is below the expected reference range in a supine position. It would be unsafe for a client to move into a sitting position as this could further decrease the blood pressure.

A nurse is preparing to administer an enteral feeding to a child who has cerebral palsy and a nasogastric tube. Which of the following actions should the nurse take? A - Administer 20 mL/min of formula by gravity. B - Refrigerate the formula for 30 min prior to administration. C - Confirm that the pH of the stomach contents is 5 or less. D - Flush the tube with 5 to 15 mL of 0.9% sodium chloride.

C - Confirm that the pH of the stomach contents is 5 or less.

A nurse is assisting with a presentation about nutrition for cancer prevention at a community center. Which of the following information should the nurse suggest including? A - Replace legumes with lean red meat. B - Eat three servings of fruits and vegetables daily. C - Consume fatty fish twice a week. D - Include different types of refined grains.

C - Consume fatty fish twice a week.

A nurse is reinforcing teaching with a client who is at 9 weeks of gestation and reports frequent episodes of nausea and vomiting. Which of the following instructions should the nurse include? A - Eat foods that are served hot. B - Drink 360 mL (12 oz) of fluids during mealtimes. C - Consume small meals frequently each day. D - Eat a high-protein snack before getting out of bed.

C - Consume small meals frequently each day.

A nurse is reviewing the laboratory results of a client who is scheduled for a CT scan with an IV contrast agent. Which of the following laboratory findings should the nurse report to the provider prior to the procedure? A - Sodium 136 mEq/L B - Potassium 4.8 mEq/L C - Creatinine 1.9 mg/dL D - Calcium 10 mg/dL

C - Creatinine 1.9 mg/dL

A nurse is reviewing the medical record of a client who is requesting a prescription for an oral contraceptive. For which of the following findings should the nurse notify the provider as a contraindication for oral contraceptive use? A - Fibrocystic breast change B - History of asthma C - Gallbladder disease D - Chlamydia infection

C - Gallbladder disease Rational A - Fibrocystic breast change is a condition where the glandular tissue of the breasts becomes lumpy and nodular. This is a benign condition and is not a contraindication for oral contraceptive use. B - A history of asthma is not a contraindication for the use of oral contraceptives. C - The nurse should identify gallbladder disease as a contraindication for the use of oral contraceptives. Increased levels of estrogen from oral contraceptives increase the risk for formation of gallstones. Therefore, the nurse should recommend another form of contraception for this client. Other contraindications include a history of thromboembolic disorders, migraine with neurologic manifestations, breast cancer, and lactation less than 6 weeks postpartum. D - A vaginal infection such as chlamydia is not a contraindication for the use of oral contraceptives.

A charge nurse is assisting in the teaching of a newly licensed nurse about using critical pathways. Which of the following information should the nurse include in the teaching? A - Critical pathways provide detailed nursing actions. B - Critical pathways prioritize daily nursing care tasks. C - Critical pathways are designed to standardize care. D - The nurse should complete each step on the critical pathway.

C - Critical pathways are designed to standardize care. Rational A - Critical pathways include client interventions but do not include detailed nursing actions. A critical or clinical pathway or care map can be used to reduce mistakes, improve outcomes and quality of care, and decrease duplication of effort. B - The nurse should continuously set and reset priorities to meet the needs of clients. A critical pathway does not prioritize daily nursing care tasks. A critical pathway can be used to support the implementation of clinical guidelines and protocols. C - Critical pathways are a guideline for standardized care within established timelines, resulting in improved client outcomes, decreased morbidity, and decreased mortality. Clinical practice guidelines reflect evidence-based practice and best practices. D - Critical pathways include interventions for various disciplines within the health care team. The nurse should carefully consult the pathway and protocol of the agency prior to implementing any pathway to become familiar with the process. The nurse should modify care to meet client needs. The nurse might not need to complete each step in the critical pathway.

A nurse is contributing to the plan of care for a client who has peripheral arterial disease (PAD) of the lower extremities. Which of the following interventions should the nurse include? A - Place moist heat pads on the extremities. B - Perform manual massage of the affected extremities. C - Dangle the extremities off the side of the bed. D - Apply support stockings before getting out of bed.

C - Dangle the extremities off the side of the bed.

A nurse is collecting data from a client who received ondansetron PO 30 min ago. Which of the following findings should the nurse identify as a therapeutic effect of the medication? A - Increased heart rate B - Decreased pain C - Decreased nausea D - Increased urine output

C - Decreased nausea Rational A - The nurse should identify that increased heart rate is not a therapeutic effect of ondansetron. However, changes in cardiac rhythm are a potential adverse effect of ondansetron. B - The nurse should identify that decreased pain is not a therapeutic effect of ondansetron. However, headache is a potential adverse effect of ondansetron. C - Ondansetron is an antiemetic medication that is administered to reduce nausea and vomiting; therefore, the nurse should identify that decreased nausea is a therapeutic effect of the medication. D - The nurse should identify that increased urine output is not a therapeutic effect of ondansetron. However, constipation and diarrhea are potential adverse effects of ondansetron.

A nurse is preparing to administer furosemide to a client who has heart failure. Which of the following findings should the nurse report before administering the medication? A - Elevated sodium B - Elevated blood pressure C - Decreased potassium D - Decreased urine output

C - Decreased potassium

A nurse is caring for a client who is unresponsive. The client's family members disagree about the provider's recommendation for placement of a feeding tube. Which process should the nurse follow when assisting with resolution of this ethical dilemma? (Place the steps in the order of performance) A - Collect data about the ethical dilemma B -Verbalize the dilemma in the simple statement C - Determine that an ethical dilemma is present D - Negotiate a plan with the client's family E - Identify possible solutions to the dilemma

C - Determine that an ethical dilemma is present A - Collect data about the ethical dilemma B -Verbalize the dilemma in the simple statement E - Identify possible solutions to the dilemma D - Negotiate a plan with the client's family

A nurse is caring for a client who has bipolar disorder. The client suddenly appears agitated and begins pacing at the end of the hallway with clenched fists. Which of the following actions should the nurse take first? A - Call for assistance to place the client in restraints. B - Administer a sedative to the client. C - Determine the client's intentions. D - Place the client into the assigned seclusion room.

C - Determine the client's intentions.

A nurse is collecting data from a client who recently began taking carvedilol. Which of the following findings should the nurse identify as an adverse effect of this medication? A - Hyperreflexia B - Tachycardia C - Dizziness when standing D - Dilated pupils

C - Dizziness when standing Rational A - The nurse should identify that paresthesia and muscle cramps, rather than hyperreflexia, are adverse effects of carvedilol. Other adverse effects of beta-blockers include bronchospasm, bradycardia, nasal congestion, and heart failure. B - ​The nurse should identify that bradycardia, rather than tachycardia, is an adverse effect of carvedilol. C - ​The nurse should identify that orthostatic hypotension is an adverse effect of beta-blockers like carvedilol. The nurse should check the client's blood pressure in a lying, sitting, and standing position and instruct the client to change positions slowly to reduce the impact of this adverse effect. D - The nurse should identify that blurred vision and dry eyes, rather than dilated pupils, are adverse effects of carvedilol.

A nurse is contributing to the plan of care for a client who has multiple sclerosis and is taking dantrolene to manage muscle spasms. Which of the following interventions should the nurse include? A - Apply hot packs to the client's muscles. B - Schedule physical therapy in the afternoon. C - Encourage the client to complete ADLs. D- Administer valerian to promote sleep.

C - Encourage the client to complete ADLs.

A nurse is caring for a client who is 24 hr postoperative following abdominal surgery and has an NG tube. Which of the following actions should the nurse plan to take to decrease the risk of postoperative complications? A - Offer sips of water to the client following oral care. B - Massage the client's lower extremities with lotion every 2 hr. C - Encourage the client to use an incentive spirometer every hour while awake. D - Place one or two pillows beneath the client's knees while he is in bed.

C - Encourage the client to use an incentive spirometer every hour while awake.

A nurse enters the room of a client who is sleeping and observes sparks coming for the frayed bed plug in the client's electrical outlet. Which of the following actions should the nurse take first? A - Unplug the client's bed. B - Pull the fire alarm closest to the area. C - Evacuate the client. D - Call maintenance for assistance.

C - Evacuate the client.

A nurse is reviewing the laboratory results of a client who is at 32 weeks of gestation. Which of the following laboratory findings should the nurse report to the provider? A - BUN 14 mg/dL B - Platelet count 200,000/mm3 C - Hematocrit 30% D - Creatinine 1.0 mg/dL

C - Hematocrit 30% Rational A - The nurse should identify that a BUN of 14 mg/dL is within the expected reference range of 10 to 20 mg/dL. B - The nurse should identify that a platelet count of 200,000/mm3 is within the expected reference range of 150,000 to 400,000/mm3. C - The nurse should identify that a hematocrit of 30% is below the expected reference range of greater than 33% for a client who is pregnant. A low Hct is an indication of anemia. Therefore, the nurse should report this finding to the provider. D - The nurse should identify that a creatinine of 1.0 mg/dL is within the expected reference range of 0.5 to 1.0 mg/dL for a female client aged 18 to 40 years.

A nurse in a pediatric clinic is observing for an anaphylactic reaction after administering an IM antibiotic to a child 5 min ago. Which of the following manifestations should the nurse expect to observe first? A - Wheezing B - Angioedema C - Hives D - Hypotension

C - Hives

A nurse at an outpatient mental health clinic is assisting with a group therapy session. One of the participants is having difficult staying seated and states loudly to the therapist, "I know more than you do about the people in this room!" The nurse should identity that which of the following findings is the likely explanation for the client's behavior? A - Somatization B - Opioid intoxication C - Hypomania D - Marijuana intoxication

C - Hypomania Rational A - Clients who have somatization behaviors exhibit increased anxiety about health concerns that cannot be explained medically. There is no evidence this client has personal health concerns at this time. B - Clients who have opioid intoxication are calm and demonstrate lethargy, euphoria, drowsiness, and slurred speech. The behavior this client is exhibiting is the opposite of those manifestations. C - The nurse should suspect hypomania as the likely cause of the client's current behavior and investigate these actions further after calmly escorting the client from the therapy session. Clients who have hypomania exhibit excessive energy and a decreased need for sleep. These clients are easily distracted in a group setting and have a pretentious, grandiose sense of self. D - Clients who are experiencing marijuana intoxication can be socially withdrawn. They might also exhibit impaired judgment, minimal coordination, and substandard reaction times. The behavior this client is exhibiting is the opposite of those manifestations.

A nurse is caring for a client who is receiving liothyronine for treatment of hypothyroidism. Which of the following findings should the nurse recognize as a therapeutic response to the medication? A - Decline in appetite B - Increase in daily weight C - Improvement in mood D - Decrease in heart rate

C - Improvement in mood Rational A - A decline in appetite is a manifestation of hypothyroidism. However, it is not a therapeutic response to this medication. B - An increase in weight is a manifestation of hypothyroidism. However, it is not a therapeutic response to the medication. C - Depression, lethargy, and fatigue are manifestations of hypothyroidism. A therapeutic response to this medication is an improvement in mood. Liothyronine is a synthetic preparation of triiodothyronine (T3), a naturally occurring thyroid hormone. Clients receive liothyronine to treat and improve the manifestations of hypothyroidism, which include anorexia, depression, lethargy, fatigue, cold and dry skin, pale and puffy face, brittle hair, decreased heart rate, decreased temperature, weight gain, and intolerance to cold. D - A decrease in heart rate is a manifestation of hypothyroidism. However, it is not a therapeutic response to this medication.

A nurse is reinforcing teaching with a client who has a new prescription for citalopram. The nurse should remind the client to monitor for which of the following as an adverse effect of the medication? A - Bleeding gums B - Decreased urination C - Increased drowsiness D - Ringing in ears

C - Increased drowsiness Rational A - Increased salivation and dry mouth, not bleeding gums, are potential adverse effects of citalopram. B - Increased urination, rather than decreased urination, is a potential adverse effect of citalopram. C - The nurse should instruct the client to monitor for drowsiness, which is a potential adverse effect of citalopram. If the client experiences drowsiness, they should avoid activities that require alertness, such as driving. D - Tinnitus is not a potential adverse effect of citalopram.

A nurse is caring for an adolescent client who requires a blood transfusion. The client's parents will not consent to the transfusion due to religious beliefs. Which of the following actions should the nurse take? A - Contact the facility chaplain to speak with the family. B - Reinforce teaching with the parents about why the blood transfusion is necessary. C - Inform the charge nurse and recommend that social services be contacted. D - Ask the client if she will accept the blood transfusion.

C - Inform the charge nurse and recommend that social services be contacted.

A nurse is assisting in the care of a client who has manifestation of sepsi. Which of the following provider prescription should the nurse implement first? A - Collect a sputum culture. B - Administer ceftriaxone by intermittent IV bolus. C - Initiate oxygen at 4 L/min via nasal cannula. D - Obtain blood cultures.

C - Initiate oxygen at 4 L/min via nasal cannula.

A nurse is assisting with the plan of care for a child who has aplastic anemia. Which of the following actions should the nurse include in the plan of care? A- Check the child's blood glucose level every 4 hr. B - Apply pressure to peripheral puncture sites for 3 min. C - Initiate protective environment isolation. D - Administer ferrous sulfate twice per day.

C - Initiate protective environment isolation. Rational A - The nurse should identify that aplastic anemia does not affect a child's blood glucose levels. Therefore, it is not necessary to check the child's blood glucose level every 4 hr. B - The nurse should identify that aplastic anemia decreases the bone marrow production of RBCs, WBCs and platelets. A child who has decreased platelets is at risk for bleeding. The nurse should apply pressure to peripheral puncture sites for at least 5 min to prevent bleeding following specimen collection. C - The nurse should initiate protective environment isolation for the child, which consists of a private, positive pressure room and nurses wearing respirator masks, gloves, and gowns. A child who has aplastic anemia has decreased RBCs, platelets, and WBCs, causing immune suppression and making the child susceptible to infection. D - The nurse should administer ferrous sulfate to a child who has iron-deficiency anemia. Aplastic anemia can require blood transfusions or stem cell placement.

A nurse is reinforcing teaching with a client about home safety precautions to reduce the risk of falls. Which of the following instructions should the nurse include in the teaching? A - Use a low toilet seat. B - Place throw rugs over noncarpeted surfaces. C - Install night lights in dark areas. D - Move the living areas to the second floor.

C - Install night lights in dark areas. Rational A - A raised toilet seat should be installed in the bathroom to reduce the risk of the client falling while getting on and off the toilet. B - Loose throw rugs present a hazard for tripping and falling. If a rug is used, it should be securely fastened to the floor. C - Night lights should be installed in the bedroom and other dark areas so that the client is able to see their path in the dark to reduce the risk of falling. D - Living areas should be moved to the first floor of the home so that the client does not have to frequently use the stairs.

A nurse is caring for a client who has acute pancreatitis. While providing care, the nurse observes ecchymosis around the umbilicus. The nurse should identify that this is a manifestation of which of the following? A - Cirrhosis of the liver B - Hypermotility of the bowel C - Intra-abdominal bleeding D - Acute cholecystitis

C - Intra-abdominal bleeding

A nurse is collecting data from a client and notices several skin lesions. Which of the following findings should the nurse report as possible melanoma? A - Scaly patches B - Silvery white plaques C - Irregular borders D - Raised edges

C - Irregular borders

A nurse is contributing to the plan of care for a client who was admitted to the neurological nuit following a stroke 3 hr ago. Which of the following interventions should the nurse identify as the priority? A - Encourage the client to participate in self-care. B - Assist the client with active range-of-motion exercises. C - Keep the client in a side-lying position. D - Maintain the client's body alignment.

C - Keep the client in a side-lying position. Rational A - The nurse should encourage the client to complete self-care to the extent that he is able. Self-care promotes mobility of the joints and increases the client's feelings of independence and self-esteem. However, there is another intervention that is the priority. B - The nurse should assist the client with active range-of-motion exercises and should provide passive range-of-motion exercises to the client's affected side to maintain joint mobility and improve muscle strength. However, there is another intervention that is the priority. C - The greatest risk to the client following a stroke is aspiration. The nurse should position the client in a lateral, or side-lying position, which will allow any secretions to drain out of the mouth, decreasing the risk for aspiration. Additionally, the nurse should have suction available in the event that any secretions are present in the oral cavity. D - The nurse should keep the client's body in alignment to maintain joint function and prevent skin breakdown caused by pressure on bony prominences. However, there is another intervention that is the priority.

A nurse is assisting a client who reports difficulty falling asleep. Which of the following activities should the nurse recommend to promote sleep? A - Get out of bed if unable to fall asleep within 60 min. B -Take a brisk walk before sleeping. C - Listen to soft music before sleeping. D - Drink adequate amounts of fluids before sleeping.

C - Listen to soft music before sleeping. Rational A - The client should get out of bed after 30 min if unable to fall asleep. B - The client should avoid stimulating activities, such as exercise, before bedtime. C - Listening to soft music can help the client to relax and reduces environmental stressors. D - The client should reduce fluids 2 to 4 hr before sleep. Drinking fluids before bedtime can cause the client to wake up during the night to use the bathroom.

A nurse is caring for a client who is 3 days postoperative following a total right hip arthroplasty. Which of the following actions should the nurse take? A - Use a traction boot to keep the client's right leg internally rotated. B - Have the client sit in a reclining chair when out of bed. C - Maintain abduction of the client's right leg while in bed. D - Encourage the client to perform passive range-of-motion exercises.

C - Maintain abduction of the client's right leg while in bed. Rational A - The nurse should not apply any type of traction boot or allow the client's leg to rotate internally or externally because it can cause a dislocation of the affected hip. B - The nurse should provide a chair that does not allow the client to recline because a reclining chair increases the risk of the client flexing at the hips beyond 90° when moving to a standing position. C - The nurse should maintain abduction of the client's right leg to prevent dislocation of the affected hip by placing an abductor pillow between the client's legs when resting in bed. D - The nurse should encourage the client to stand at the bedside on the day of surgery and, if prescribed by the provider, to walk using a walker. Passive range-of-motion exercises require flexion and extension of the joints and are not recommended 3 days following surgery.

A nurse is collecting data from an older adult client who has received a scheduled opioid analgesic and is now confused and appears drowsy. Which of the following actions should the nurse take? Select all A - Raise four side rails. B - Apply restraints PRN. C - Measure the client's SaO2. D - Instruct the client in the use of the call light. E - Apply an ambulation alarm to the client's leg.

C - Measure the client's SaO2. D - Instruct the client in the use of the call light. E - Apply an ambulation alarm to the client's leg. Rational A - Raise four side rails is incorrect. Raising all of the side rails is considered a restraint. The client should have a route to exit the bed safely and be able to maneuver freely within the bed. For a client who is disoriented, the risk for injury is greater with all side rails raised because the client could attempt to climb over the side rail or the foot of the bed. The nurse should place the bed in the lowest position. B - Apply restraints PRN is incorrect. Restraint requirements include a written prescription for a specific episode, time frame, and type. Clients who are at risk for harming themselves might require restraints; however, the nurse should use all other less restrictive alternatives first. The nurse should provide ongoing monitoring and documentation of the client's condition, including circulation and skin integrity around the restraint, as well as client behavior. C - Measure the client's SaO2 is correct. Confusion can be a manifestation of hypoxia caused by oversedation, respiratory depression, or worsening client condition. The nurse should collect relevant client data, such as pulse oximetry monitoring, when there is a change in the client's condition or behavior. D - Instruct the client in the use of the call light is correct. Having clients call for and wait for assistance before getting out of bed minimizes the risk of accidents. Nursing staff should ensure the call light is functional and within reach of the client. E - Apply an ambulation alarm to the client's leg is correct. The ambulation alarm signals when the leg is in a dependent position, such as over the side rail or on the floor. The signal alerts the staff to check on the client immediately, which can reduce the risk for injury to the client from a fall or provide prompt intervention.

A nurse is caring for a client who has a prescription for verapamil 120 mg per day in three divided doses. The nurse administered a 120-mg capsule of verapamil for the morning dose. Which of the following actions is the nurse's priority? A - Elevate the client's legs. B - Complete an incident report. C - Measure the client's blood pressure. D - Prepare the client for gastric lavage.

C - Measure the client's blood pressure. Rational A - The nurse should elevate the client's legs to facilitate blood flow to the brain and increase blood pressure. However, there is another action that is the nurse's priority. B - Medication errors require that the nurse who discovered the error complete an incident report. However, there is another action that is the nurse's priority. C - The first action the nurse should take using the nursing process is to collect data from the client. An overdose of verapamil can cause severe hypotension. Therefore, the priority action for the nurse to take is to monitor the client's blood pressure and intervene promptly for hypotension. D - The nurse might need to prepare the client for gastric lavage to remove any verapamil left in the client's stomach. However, there is another action that is the nurse's priority.

A nurse is caring for a client who has acute heroin use disorder and will soon experience manifestations of withdrawal. Which of the following medications should the nurse expect to administer to the client? A - Risperidone B - Disulfiram C - Methadone D - Chlordiazepoxide

C - Methadone Rational A - Risperidone is an antipsychotic medication prescribed for a client who has schizophrenia. This medication is not used to treat manifestations of heroin withdrawal. B - Disulfiram is prescribed for a client who has chronic alcohol use disorder to deter alcohol intake. Disulfiram causes unpleasant effects even if the client consumes small amounts of alcohol. This medication is not used to treat manifestations of heroin withdrawal. C - Methadone is a synthetic opiate that blocks the craving for and effects of opioids. The provider can prescribe this medication to assist with detoxification and to help the client maintain abstinence from heroin and other opioids. D - Chlordiazepoxide, a benzodiazepine, is prescribed to treat adverse effects of alcohol withdrawal. This medication is not used to treat manifestations of heroin withdrawal.

A nurse is collecting data from a client who is receiving long-term glucocorticoid therapy to treat lupus erythematosus (LE). The nurse should recognize that which of the following findings is an adverse effect of long-term glucocorticoid therapy? A - Paresthesia B - Weight loss C - Moon face D - Hypoglycemia

C - Moon face Rational A - Paresthesia is a manifestation of hyperkalemia. Hypokalemia is an adverse effect of glucocorticoid therapy. B - Weight gain is an adverse effect of glucocorticoid therapy. C - A client who is on long-term glucocorticoid therapy can develop cushingoid adverse effects due to adrenal suppression, such as moon face, weight gain, hyperglycemia, and hypokalemia. LE is a chronic systemic condition that can cause manifestations such as pleurisy, nephritis, and pericarditis. D - Hyperglycemia is an adverse effect of glucocorticoid therapy.

A nurse is collecting data from a client who is receiving hydrocodone for postoperative pain. When finding that the client's respiratory rate is 9/min, the nurse should anticipate a stat prescription for which of the following medications? A - Flumazenil B - Acetylcysteine C - Naloxone D - Atropine

C - Naloxone Rational A - Flumazenil, a benzodiazepine antagonist, is administered to reverse sedation caused by benzodiazepines, not opioids. This medication might not reverse the client's respiratory depression. It is approved for benzodiazepine overdose and for reversing the effects of benzodiazepines following general anesthesia. B - Acetylcysteine, a mucolytic, reduces the risk of hepatotoxicity after an acetaminophen overdose. It is also used to thin secretions for clients who have cystic fibrosis, atelectasis, and tracheostomies. It is administered prior to diagnostic procedures for clients who are at risk for renal dysfunction. C - Hydrocodone is an opioid analgesic. Naloxone, an opiate antagonist, reverses the effects of opioids, including respiratory depression and sedation. It should be administered with caution because this can cause complete withdrawal, leading to hypertension and the return of pain. It might need to be re-administered because it has a short half-life. For this reason, the nurse should continue to monitor the client's respiratory status. D - Atropine, an anticholinergic medication, treats bradycardia, not bradypnea. It is administered prior to surgical and diagnostic procedures because it decreases secretions from the salivary glands, sweat glands, bronchial glands, and gastric glands. This decreases the risk for aspiration.

A nurse is reviewing the current plan of care for an adolescent who has dysmenorrhea. Which of the following treatment prescriptions should the nurse expect? A - Folic acid 0.8 mg twice daily B - Vaginal culture C - Nonsteroidal anti-inflammatory drugs (NSAIDs) D - Amitriptyline hydrochloride 10 mg three times daily

C - Nonsteroidal anti-inflammatory drugs (NSAIDs) Rational A - The nurse should identify that folic acid is recommended for clients who are of childbearing age to prevent neural tube defects in a fetus should conception occur. Folic acid does not decrease dysmenorrhea. B - The nurse should expect the provider to prescribe a vaginal culture for a client who has manifestations of a vaginal infection. C - The nurse should expect the provider to prescribe a nonsteroidal anti-inflammatory drug (NSAID) for a client who has dysmenorrhea or painful menstruation. NSAIDs block the formation of prostaglandins, leading to a reduction of uterine activity and prevention of pain. Prophylactic use of NSAIDs has proven effective when begun a few days before the onset of menses. The relief appears to be a result of the prostaglandin inhibition rather than the analgesic effect. D - The nurse should expect the provider to prescribe amitriptyline, a tricyclic antidepressant medication, for a client who has a depressive disorder. Amitriptyline does not decrease dysmenorrhea.

A nurse is assisting with the discharge planning for a client who is postoperative following a total hip arthroplasty. Which of the following instructions should the nurse include in the discharge plan? A - Expect decreased sensation for the first postoperative week. B - Avoid lying on the operative side. C - Obtain a raised toilet seat. D - Cross legs at the ankles.

C - Obtain a raised toilet seat.

A nurse is assigned to care for a client who is in isolation. Which of the following actions should the nurse take to manage time effectively while caring for this client? A - Assign an assistive personnel (AP) to apply a medicated ointment during perineal care. B - Store several sets of extra bed linens in the client's room. C - Organize care into groups that can be performed at one time. D - Schedule time at the end of the shift to document all client care.

C - Organize care into groups that can be performed at one time.

A nurse is evaluating the laboratory results for a client who has malnutrition. Which of the following laboratory results should the nurse expect? A- Lymphocytes 30% B - Hemoglobin 14 g/dL C - Prealbumin 11 mg/dL D - Calcium 10 mg/dL

C - Prealbumin 11 mg/dL Rational A - A lymphocyte count of 30% is within the expected reference range of 20 to 40%. The nurse should expect a lymphocyte count below the expected reference range for a client who has malnutrition. B - A hemoglobin level of 14 g/dL is within the expected reference range of 14 to 18 g/dL for a male and 12 to 16 g/dL for a female. The nurse should expect a hemoglobin level below the expected reference range for a client who has malnutrition. C - A prealbumin level of 11 mg/dL is below the expected reference range of 15 to 36 mg/dL and is a manifestation of malnutrition. Prealbumin is a short-term indicator of insufficient nutritional intake. D - A calcium level of 10 mg/dL is within the expected reference range of 9.0 to 10.5 mg/dL. The nurse should expect a calcium level below the expected reference range for a client who has malnutrition.

A nurse is administering a liquid ophthalmic medication to a client. Which of the following actions should the nurse take? A - Clean the client's eyelid from the outer toward the inner canthus before administration of the medication. B - Instruct the client to squeeze their eyes shut after administration of the medication. C - Press firmly on the client's nasolacrimal duct following administration of the medication. D - Instill the medication directly onto the client's cornea.

C - Press firmly on the client's nasolacrimal duct following administration of the medication. Rational A - The nurse should don clean gloves and clean the client's eyelid and eyelashes with a moistened sterile cotton ball starting from the client's inner canthus to the outer canthus. This action reduces the risk of contamination of the client's other eye and the lacrimal duct. B - The nurse should instruct the client to close their eyelids gently after administration of the medication to distribute the medication over the eye. The client should avoid squeezing their eyes shut because this action can cause eye injury and might result in the loss of some of the medication out of the eye. C - The nurse should press the client's nasolacrimal duct firmly for 30 seconds to prevent the medication from running out of the eye and to prevent systemic absorption of the medication through the duct. D - The nurse should instill the medication into the client's conjunctival sac to prevent injury to the client's cornea.

A nurse is contributing to the plan of care for a client who is having difficulty eating following a stroke. Which of the following interventions should the nurse plan to implement first? A - Collaborate with a dietitian. B - Provide nutritional supplements. C - Recommend a referral for a speech language pathologist. D - Inform assistive personnel about proper positioning.

C - Recommend a referral for a speech language pathologist.

A nurse is caring for a client who has bacterial meningitis. Upon monitoring the client, which of the following findings should the nurse expect? A - Flaccid neck B - Stooped posture with shuffling gait C - Red macular rash D - Masklike facial expression

C - Red macular rash

A nurse is contributing to the plan of care for a client who has neutropenia following chemotherapy. Which of the following interventions should the nurse recommend for the client's plan of care? A - Encourage the client to use a disposable razor for shaving. B - Increase fresh uncooked produce in the client's diet. C - Reinforce teaching about deep breathing exercises. D - Insert an indwelling urinary catheter.

C - Reinforce teaching about deep breathing exercises. Rational A - The client should use an electric razor to prevent cutting the skin, which can increase the risk of bleeding and infection. B - The client should avoid consuming uncooked foods, such as raw fruits and vegetables, which can introduce bacteria into the gastrointestinal system. The client should also have a private room, designated equipment, and restricted exposure to live plants. C - A client who has neutropenia is at an increased risk for infection. Protection from and early recognition of infection is a high priority. Deep breathing exercises promote lung expansion and clear secretions that could cause infection. D - A client who has neutropenia should have minimal invasive procedures because they can increase the client's risk for injury and infection. The nurse should limit sources of exposure to infectious agents.

A nurse is collecting data from a client who has an acute infection and is shivering. The client's temperature is 40.2°C (104.4°F). Which of the following actions should the nurse take? A - Give the client a cold sponge bath. B - Bathe the client with an alcohol and water solution. C - Request a prescription for an antipyretic medication. D - Apply ice packs directly to the skin of client's groin area.

C - Request a prescription for an antipyretic medication. Rational A - The nurse should avoid giving the client a cold sponge bath because this action could increase the client's shivering. If a sponge bath is used, the bath should be performed with tepid water. However, this therapy can increase the risk of shivering, which is counterproductive to restoration of a normal body temperature. B - The nurse should avoid bathing the client with an alcohol and water solution. This action is not recommended because of the risk of increased shivering, which is counterproductive to restoration of a normal body temperature. The alcohol in the bath can also remove moisture from the client's skin. C - The nurse should implement actions to reduce the client's body temperature without causing the client to experience further shivering. Methods of restoring a normal body temperature include the administration of antipyretics like ibuprofen and acetaminophen. Antipyretic medications serve to increase the heat lost by the client's body, which helps to reduce fever. Antipyretics can be used along with a hypothermia blanket and a bath sheet as a skin barrier to achieve additional temperature reduction. D - The nurse should avoid applying ice packs to the client's groin and axillae. This action is not recommended because of the risk of increased shivering, which is counterproductive to restoration of a normal body temperature. There should always be a cloth barrier between ice packs and the client's skin to insulate the skin from damage that can occur due to the extreme cold.

A nurse is contributing to the plan of care for a client who has obsessive-compulsive disorder and continually washes her hands. Which of the following interventions should the nurse include? A - Turn off the water if the client washes her hands more than once. B - Do not allow the client to use a private restroom. C - Schedule times for the client to wash her hands during the day. D - Explain that unit privileges will be taken away if excessive handwashing continues.

C - Schedule times for the client to wash her hands during the day.

A nurse is discussing deliver models of care with a group of newly licensed nurses. Which of the following should the nurse include as an example of the functional nursing model of care? A - One nurse is assigned to complete care for one client. B - A group of nurses work together to care for a group of clients. C -Nurses are assigned specific tasks to perform for each of the clients. D - Assignments are made based on client location within the unit.

C -Nurses are assigned specific tasks to perform for each of the clients. Rational A - Nurses use a total client care model when assigning a nurse complete care for one client. B - Nurses use a team nursing model when assigning a team to care for a group of clients. C - Nurses use a functional nursing model when assigning specific tasks to staff. This approach places the focus on the task to be performed. D - Nurses use a team nursing model when assigning a team to care for a group of clients based on client location within the unit.

A nurse is contributing to the plan of care for a client who has a tracheostomy. Which of the following interventions should the nurse include? A - Use medical aseptic technique. B - Clean the inner cannula with mild soap and water. C - Secure new tracheostomy ties before removing old ones. D - Clean the outer cannula before the inner.

C - Secure new tracheostomy ties before removing old ones. Rational A - The nurse should maintain sterile technique (surgical asepsis) when suctioning and providing tracheostomy care to reduce the risk of infection. Surgical asepsis involves maintaining a sterile field, technique, and instruments. Medical asepsis is a clean technique using routine barrier protection and hand hygiene. B - The nurse should clean the inner cannula with sterile 0.9% sodium chloride solution or according to facility policy. If the inner cannula is disposable, the nurse should remove it and replace it with a new one. C - Tube dislodgement and accidental decannulation are potential complications of a tracheostomy. By keeping the old ties in place while applying new ties, the nurse can secure the tube and reduce the risk for dislodgement. Within the first 72 hr after surgery, accidental decannulation is an emergency because the stoma could close and it might be difficult to replace the tracheostomy tube. If accidental decannulation occurs within the first 72 hr, immediately hyperextend the client's neck, and with the obturator inserted into the tracheostomy tube, quickly and gently replace the tube and remove the obturator. The nurse should be sure to keep an obturator and extra tracheostomy tube at the bedside. D - Using principles of asepsis, the nurse should clean the least contaminated area first. The nurse should replace (disposable system) or cleanse the inner cannula first using 0.9% sodium chloride solution, in accordance with facility policy.

A nurse is reinforcing teaching with a client who has a prescription for fluoxetine to treat major depressive disorder. Which of the following manifestations should the nurse include as an adverse effect of fluoxetine? A - Pupillary constriction B - Hypersalivation C - Sexual dysfunction D - Hirsutism

C - Sexual dysfunction Rational A - The nurse should inform the client that mydriasis, or pupillary dilation, is an adverse effect of fluoxetine. Other adverse effects include blurred vision and nasal congestion. B - The nurse should inform the client that dry mouth is an adverse effect of taking fluoxetine. The client should be encouraged to use sugarless gum or candy and to rinse their mouth with water frequently to minimize the drying effects. C - Impaired sexual function is a very common adverse effect of taking fluoxetine. To minimize the effects, the provider might prescribe a medication holiday, having the client only take the medication on certain days of the week. Additional prescriptions might be required to help the client regain sexual function. The nurse should ask the client directly about sexual problems, because the client might be reluctant to report them. D - Taking fluoxetine can cause menstrual changes but it is not known to affect hair growth. The nurse should inform the client that manifestations such as hot flashes can also occur.

A nurse is reinforcing teaching with a client about the adverse effects on consuming alcohol while taking lorazepam. Which of the following adverse effects should the nurse include? A - Hypertension B - Tachypnea C - Sleep walking D - Motor tics

C - Sleep walking Rational A - The nurse should instruct the client to monitor for dizziness and headache and to change positions cautiously. Cardiovascular effects of taking lorazepam include hypotension, which can lead to cardiac arrest. B - The nurse should instruct the client that lorazepam can cause respiratory depression rather than increased respiratory rate. This effect is more prominent when combined with other CNS depressants such as alcohol. The nurse should instruct the client to avoid drinking alcohol while taking lorazepam. C - Some clients perform complex behaviors during sleep while taking a benzodiazepine. Clients have reported walking, driving, cooking, eating, and making phone calls. The risk for this adverse effect is increased with high dosage or alcohol consumption. Therefore, the nurse should instruct the client to avoid alcohol while taking this medication and advise the client to report occurrence of these behaviors to the provider immediately. D - CNS effects that can occur while taking lorazepam include dizziness and loss of balance, not motor tics. The nurse should provide guidance to the client about creating a safe home environment to reduce the risk for falls.

A nurse is reinforcing teaching with the guardians of a school-age child who has hearing loss. Which of following techniques should the nurse recommend to facilitate communication with the child? A - Exaggerate the pronunciation of each word. B - Keep hands still when speaking. C - Speak at the child's eye level. D - Avoid facial expressions when speaking.

C - Speak at the child's eye level.

A nurse is contributing to the plan of care to meet the nutritional needs of a client who has dysphagia. To which of the following interdisciplinary team members should the nurse refer the client? A - Physical therapist B - Social worker C - Speech pathologist D - Respiratory therapist

C - Speech pathologist Rational A - A physical therapist provides treatment to improve mobility and strength for clients who have musculoskeletal difficulties. There is no indication this client requires physical therapy. B - A social worker helps meet the client's psychosocial needs by coordinating financial and community resources, completing advance directives, and discharge planning. There is no indication this client requires a social worker. C - A speech pathologist assesses and makes recommendations for clients experiencing speech, language, and swallowing difficulties. The speech pathologist can teach swallowing techniques and exercises to facilitate swallowing. D - A respiratory therapist administers oxygen and performs respiratory treatments for clients who have respiratory difficulties. There is no indication this client requires respiratory assistance.

A nurse enters the room of a client who se transfusion of packed RBCs was initiated 15 min ago yb the RN. The client reports dyspnea and urticaria. Which of the following actions should the nurse perform first? A - Count the client's respiratory rate. B - Ask the client if chest pain is present. C - Stop the infusion. D - Administer an antihistamine.

C - Stop the infusion.

A nurse is planning to administer terbutaline to a client who is experiencing preterm labor. Which of the following routes of administration should the nurse plan to use? A - Intramuscular B - Intradermal C - Subcutaneous D - Topical

C - Subcutaneous

A nurse is assisting with planning care for a client who is breastfeeding and has mastitis. Which of the following recommendations should the nurse include? A- Instruct the client to discontinue feeding from the affected breast. B - Tell the client to wear an underwire bra. C- Instruct the client to apply warm compresses to the affected breast. D - Administer an antiviral medication.

C- Instruct the client to apply warm compresses to the affected breast.

A charge nurse is reinforcing teaching about advanced directives with a newly licensed nurse. Which of the following information should the charge nurse include? A - The provider must be present when the client signs the advance directives. B - A risk manager needs to review the client's advance directives. C - The client can change their decision about treatment at any time. D - The client must choose a relative as a health care surrogate.

C - The client can change their decision about treatment at any time. Rational A - The provider is not required to be present when the client signs the advance directives. However, the provider should also sign the advance directives in acknowledgement of the client's wishes. B - A client does not need to have a risk manager review the advance directives. The nurse should expect a risk manager to investigate unexpected injuries to clients and visitors, or situations that do not comply with typical operations. C - Advance directives consist of a living will and a health care proxy. A client can change their decision about treatment at any time after signing advance directives. The living will provides instructions to direct treatment in the event the client is unable to make their own treatment decisions. D - The durable power of attorney or health care proxy is a legal document that designates a person who will speak on the behalf of the client when the client is unable to do so. The client can choose anyone to be their health care surrogate.

A nurse is collecting data about the dietary habits of an adolescent client. The nurse should identify that which of the following findings puts the client at risk for nutritional deficits? A - The client chooses to eat more vegetables than fruits. B - The client chooses fast food for lunch once each week. C - The client skips eating dinner for track practice three times per week. D -The client eats breakfast on the way to school.

C - The client skips eating dinner for track practice three times per week.

A nurse is observing a client bathe her 1-day-old newborn. Which of the following actions should the nurse identify as an indication that the client understands how to bathe her newborn? A - The client shakes powder from the container onto the newborn's skin. B - The client uses a cotton-tipped swab to clean the newborn's ears. C - The client washes the newborn's hair before unwrapping her. D - The client rinses the newborn under warm, running water.

C - The client washes the newborn's hair before unwrapping her.

A nurse is collecting data from a client who has had an ileostomy for several years. Which of the following findings is the priority for the nurse to report to the charge nurse? A - The client wears drainage pouches for 8 days at a time. B - The ostomy output is watery. C - The stoma is pale in color. D - The skin around the stoma is red.

C - The stoma is pale in color. Rational A - Wearing an ostomy drainage pouch for 8 days could increase the client's risk for skin irritation and infection. However, there is another finding that is a greater risk to the client. The nurse should remind the client to only wear ostomy pouches for up to 7 days at a time. B - It is common for an ileostomy to drain frequently. The ileum is closer to the proximal portion of the digestive tract, so there has not been time for fluid to be reabsorbed in the intestines. The stool can be loose, dark green liquid. Watery output can indicate the client is at risk for dehydration. However, there is another finding that is a greater risk to the client. The nurse should continue to monitor for adequate fluid balance. C - A pale-colored stoma indicates the client is at greatest risk for tissue necrosis of the stoma. The stoma should be pinkish to cherry red, which indicates an adequate blood supply. If the stoma becomes pale, bluish, or dark, the nurse should report these findings to the charge nurse immediately. This is considered a medical emergency. D - The stool from the small intestine contains enzymes and bile salts, which can quickly irritate and injure the skin. However, there is another finding that is a greater risk to the client. The nurse should clean the skin and apply a skin barrier for further protection.

A nurse is collecting data from a toddler who has a fracture wrist and had a long arm cast applied 2 hr ago. Which of the following findings should the nurse report to the provider? A - The fingers of the affected hand have a capillary refill of 1 second. B - The fingertips of the affected hand are warm to the touch. C - The toddler is unable to move the fingers on the affected hand. D - The toddler requires assistance from the parent to keep the affected arm elevated.

C - The toddler is unable to move the fingers on the affected hand. Rational A - Capillary refill of less than 2 seconds is an indication of adequate perfusion. Therefore, this finding does not require notification of the provider. B - Fingertips of the affected hand that are cool to the touch can be an indication of compartment syndrome. Therefore, this finding does not require notification of the provider. C - The nurse should identify that the clinical findings of compartment syndrome include pallor, paresthesia, paralysis, pulselessness, and pain. This is an emergency finding that should be reported to the provider immediately because the toddler's hand is not receiving adequate blood supply, which can lead to tissue necrosis. If compartment syndrome is verified, the cast will need to be removed immediately. D - Keeping the affected arm elevated will assist with decreasing pain and swelling. Also, a long arm cast is heavy for a toddler to manage. Therefore, the toddler receiving assistance from the parent is an expected finding.

A nurse is monitoring a preschool following an abdominal CT scan with contrast dye. The nurse should identify which of the following as an indication that the preschooler experienced an allergic reaction to the contrast dye? A - Jaundice B - Hematuria C - Urticaria D - Petechiae

C - Urticaria

A nurse is planning to administer phytonadione to a newborn. Which of the following actions should the nurse take? A - Administer phytonadione 24 hr after birth. B - Use a 1-inch needle for administration. C - Use the vastus lateralis as the injection site. D - Reinforce to the guardian that the injection should be repeated in 2 weeks.

C - Use the vastus lateralis as the injection site.

A nurse is contributing to the plan of care for a newly admitted client who has Alzheimer's disease. Which of the following interventions should the nurse include? A - Use words instead of symbols to identify bathroom doors. B - Dim the lighting in the client's room to encourage resting. C - Use yellow tape to identify specific boundaries for the client. D - Approach the client from the side before touching.

C - Use yellow tape to identify specific boundaries for the client. Rational A - The nurse should use symbols rather than words on doors to identify the location of the client's bathroom or the dining room. B - The nurse should keep the client's room well-lit to decrease the risk for falls. C - The nurse should use yellow tape to identify specific boundaries, which will assist with orientation and decrease confusion for a client who has Alzheimer's disease. D - The nurse should ensure that they are fully visible to the client before touching to prevent a fright reaction.

A nurse is caring for a client who has a new prescription for raloxifene to treat postmenopausal osteoporosis. The nurse should monitor the client for which of the following adverse effects of this medication? A -Esophagitis B - Transient fever C - Venous thromboembolism D - Increase in LDL cholesterol

C - Venous thromboembolism Rational A - Raloxifene is not known to cause esophagitis. Esophagitis is a potential adverse effect of alendronate, which is another medication used to treat osteoporosis. B - Raloxifene is not known to cause transient fever. Transient fever is a potential adverse effect of zoledronate, which is another medication used to treat osteoporosis. C - Raloxifene increases the client's risk for deep-vein thrombosis (DVT), stroke, and pulmonary embolism. Because inactivity promotes DVT, clients should discontinue raloxifene at least 72 hr before prolonged immobilization. A history of venous thrombotic events is a contraindication for the use of this medication. D - Raloxifene can decrease, rather than increase, LDL cholesterol levels.

A nurse is preparing to suction a client who has a tracheostomy. Which of the following actions should the nurse take first? A - Insert the suction catheter into the tracheostomy. B - Rinse the catheter with sterile 0.9% sodium chloride. C - Ventilate with 100% oxygen. D - Occlude the vent on the catheter for 10 seconds.

C - Ventilate with 100% oxygen.

A nurse is preparing to administer clozapine for the first time to a client who has schizophrenia. The nurse explains the therapeutic and adverse effects of the medication to the client to administration. Which of the following ethical concepts is the nurse demonstrating? A - Autonomy B - Justice C - Veracity D - Confidentiality

C - Veracity

A nurse in a pediatric clinic is collecting data from an infant who recently started taking digoxin. Which of the following manifestations should the nurse identify as an indication of digoxin toxicity and report to the provider? A - Irritability B - Diaphoresis C - Vomiting D - Tachycardia

C - Vomiting

A nurse is reinforcing home safety instructions with the parents of a toddler. Which of the following patient statements indicates an understanding of the teaching? A - We will keep out child out of the sun between 3 pm and 5 pm B - We will transition out child to a toddler bed when he is 2 feet tall C - We will turn the pot handles toward the back of the stove D - we will provide a healthy snack of peanuts

C - We will turn the pot handles toward the back of the stove

A nurse is assisting with the admission of a client who has tuberculosis. Which of the following actions should the nurse take? A - Ensure the client's room has a positive pressure airflow exchange. B - Place the client in a semi-private room. C - Wear a HEPA mask when providing care for the client. D- Report the client's diagnosis to the facility's risk manager.

C - Wear a HEPA mask when providing care for the client. Rational A - The nurse should ensure the client's room has a negative pressure airflow exchange system to prevent the air from the client's room from entering the facility's air circulation. A positive pressure airflow exchange provides a protected environment for clients who are immunocompromised. B - The nurse should place the client in a private room to avoid spreading the infection to another client. Clients who have active tuberculosis require airborne precautions. C - The nurse should use personal protective equipment when caring for a client who is in transmission-based precautions. For a client who has tuberculosis, airborne precautions are initiated, which include the use of a HEPA mask to maintain the safety of the nurse and reduce the spread of infection. D - Tuberculosis is an infectious disease that poses a risk to public safety. Health care personnel should report instances of active tuberculosis to the local health department to decrease the risk of the public from contracting the disease. A facility risk manager investigates instances of medication or equipment errors that impact client safety.

A nurse is caring for a client who just received a dose of amoxicillin. Which of the following findings should the nurse address first? A - Urticaria B - Vomiting C - Wheezing D - Pruritus

C - Wheezing Rational A - Urticaria, or hives, is an indication of an adverse effect of the amoxicillin. If itching occurs, the client can use cool compresses. The nurse can also obtain a prescription for diphenhydramine. However, another finding is the nurse's priority. B - Vomiting is an indication of an adverse effect of the amoxicillin. Nausea, vomiting, and diarrhea can occur with administration of this medication. The nurse should closely monitor the client's fluid and electrolyte balance. However, another finding is the nurse's priority. C - The greatest risk to the client is anaphylaxis, resulting from an allergic reaction to the amoxicillin. The priority finding is wheezing, which is an indication of anaphylaxis. The nurse should notify the charge nurse and provider. Bronchospasms and edema in the airway will cause dyspnea and wheezing and can result in death. The nurse should advise the client to wear a form of identification that indicates an allergy to this medication. D - Pruritus is an indication of an adverse effect of amoxicillin. If itching occurs, the client can use cool compresses. The nurse can also obtain a prescription for diphenhydramine. However, another finding is the nurse's priority.

A nurse is reinforcing teaching about dietary changes with a client who has cardiovascular disease. Which of the following images indicates the type of cooking fat the nurse should recommend the client use when preparing meals? A - butter B - coconut oil C - olive oil D - shortening

C - olive oil

A nurse is performing an ECG on a client who is scheduled for surgery the following morning. In which of the following locations should the nurse place the V1 electrode A - upper right B - upper left C - right 4th intercostal 4 - left 4th intercostal

C - right 4th intercostal Rational A is incorrect. The nurse should identify that the Right Arm (RA) electrode should be positioned just below the right clavicle. B is incorrect. The nurse should identify that the Left Arm (LA) electrode should be positioned just below the left clavicle. C is correct. The nurse should identify that the V1 electrode should be placed in the 4th intercostal space just to the right of the sternum. Correct placement of the electrodes is vital in obtaining accurate information about the electrical activity of the heart. D is incorrect. The nurse should identify that the V2 electrode should be placed in the 4th intercostal space just to the left of the sternum.

A nurse is collecting data from a newborn whose mother had gestational diabetes mellitus. Which of the following findings should the nurse report to the provider? A - Calcium 9.2 mg/dL B - Heart rate 160/min C -Blood glucose 28 mg/dL D - Axillary temperature 36.5° C (97.7° F)

C -Blood glucose 28 mg/dL

A nurse is delegating tasks to assistive personnel (AP). Which of the following statements by the nurse includes the five rights of delegation? A - "Take the vital signs of the clients in rooms 226 through 232 and evaluate any changes." B - "Turn the client in room 621 to prevent pressure areas on his hip bones and assess for edema." C - "Perform the glucose checks on time and replace the glucometer back into the charger when you finish." D - "Ambulate the client in room 316 to the end of the hall before lunch and report any shortness of breath."

D - "Ambulate the client in room 316 to the end of the hall before lunch and report any shortness of breath." Rational A - This statement does not include the circumstance, supervision, or a task appropriate for an AP. B - This statement does not include direction, supervision, or a task appropriate for an AP. C - This statement does not include the correct client, circumstance, or supervision. D - The nurse should identify that the five rights of delegation include right task, circumstances, person, direction, communication, and level of supervision. This statement contains all five rights of delegation and is an appropriate task for the nurse to delegate to an AP.

A nurse is reinforcing teaching with the family of a preschooler whose parent has a terminal diagnosis. Which of the following statements should the nurse include when discussing age-appropriate responses to death? A - "Your child will likely exhibit fear of the impending death with verbal uncooperativeness." B - "At this age, your child will understand that death is irreversible." C - "Your child will likely be curious about what happens to the body after death." D - "At this age, your child likely believes his thoughts can cause another person's death."

D - "At this age, your child likely believes his thoughts can cause another person's death."

A nurse is reinforcing discharge teaching with a client who has cirrhosis. Which of the following instructions should the nurse include? A- "You can take acetaminophen for pain." B - "Consume a diet high in animal protein." C - "Sleep lying flat on your back." D - "Consume foods low in sodium."

D - "Consume foods low in sodium."

A charge nurse in a long-term care facility is reveiwing message boards in various client rooms. Which of the following information should the charge nurse request one of the nurses remove form the client's board? A - "Vital signs twice daily" B - "I&O q4h" C - "Laboratory test at 0600" D - "Hospice nurse visit at 1600"

D - "Hospice nurse visit at 1600"

A nurse is reinforcing discharge teaching with a client who is 2 days postpartum. The client expresses concern about a lack of family support and limited financial resources. Which of the following responses should the nurse make? A - "You will be fine. I've seen many new mothers cope with this situation." B - "Have you considered taking a community parenting class?" C - "Once you get past the postpartum period, you'll feel better about handling these challenges." D - "How do you feel about discussing your concerns with a social worker?"

D - "How do you feel about discussing your concerns with a social worker?"

A home health nurse is reinforcing teaching with a client about preventing complications of peripheral vascular disease. Which of the following statements indicates that the client is adhering to the nurse's instructions? A - "I apply rubbing alcohol to my feet every day to prevent infection." B - "I will wear clean, knee-high wool socks every day to help improve my circulation." C - "I use hot water bottles to keep my feet warm at night." D - "I don't cross my legs anymore."

D - "I don't cross my legs anymore." Rational A - Rubbing alcohol has a drying effect on skin and can increase cracking, allowing an entry point for infection. The client should apply lotions that do not contain alcohol. B - Wool socks can result in perspiration, which puts the client at risk for developing a fungal infection. The client should use light-weight socks to promote arterial blood flow. C - Clients who have peripheral vascular disease have decreased sensation of the affected extremities. Therefore, they are unable to detect the temperature of the water bottle, which increases the risk for burns. D - Clients who have peripheral vascular disease should not cross their legs because it can impede circulation.

A nurse is caring for a client who takes naltrexone for the treatment of alcohol use disorder. The nurse should identify that which of the following client statements indicates the medication is effective? A - "Naltrexone calms my nerves." B - "I get flushed when I drink alcohol while taking naltrexone." C -"Naltrexone decreases my fine hand tremors." D - "I drink less alcohol in a day while taking naltrexone."

D - "I drink less alcohol in a day while taking naltrexone."

A nurse is caring for a client who takes bupropion. Which of the following statements indicates that the client is experiencing a therapeutic effect of the medication? A - "My cravings for alcohol have decreased." B - "I am no longer hearing voices." C - "My muscle spasms have lessened." D - "I have been isolating myself less."

D - "I have been isolating myself less." Rational A - This is not a therapeutic effect of bupropion. This is an expected therapeutic effect of medications used for alcohol cessation such as naltrexone. B - This is not a therapeutic effect of bupropion. This is an expected therapeutic effect of an antipsychotic medication such as clozapine. C - This is not a therapeutic effect of bupropion. This is an expected therapeutic effect of benzodiazepines such as diazepam. D - Increased socialization is a therapeutic effect of the antidepressant bupropion, a norepinephrine dopamine reuptake inhibitor (NDRI).

A nurse is reinforcing teaching with a client who has mitral valve disease. Which of the following statements by the client indicates an understanding of the disease process? A - "I should call my doctor if I get a headache." B - "I may develop gastric reflux." C - "I may develop excessive bruising." D - "I should call my doctor if my ankles swell."

D - "I should call my doctor if my ankles swell." Rational A- Headaches are not a complication of mitral valve disease. B - Mitral valve disease does not cause gastric reflux. C - A provider may prescribe anticoagulants to prevent thrombus formation on the valve, which can cause excessive bruising for a client who has mitral valve disease. However, excessive bruising is not a direct result of the disease. D - Swelling of the ankles can indicate heart failure. The client should report this finding to the provider.

A nurse is reinforcing teaching with a client who has generalized anxiety disorder and is to start therapy with buspirone. Which of the following statements should the nurse identify as an indication that the client understands the information? A -"This medication can cause dependence." B - "I'll take an extra dose of my medication when I start to feel anxious." C - "It's important for me to take my medication 30 minutes before bedtime." D - "I should expect some improvement of my symptoms in about 10 days."

D - "I should expect some improvement of my symptoms in about 10 days."

A nurse is reinforcing discharge teaching about methods to prevent engorgement during lactation suppression with a client who is bottle-feeding her newborn. Which of the following statements should the nurse identify as an indication that the client understands the instructions? A - "I will massage my breasts while I take a shower." B - "I should wear an underwire bra during the day." C - "I should use a breast pump several times a day to relieve discomfort." D - "I will apply cold cabbage leaves to my breasts throughout the day."

D - "I will apply cold cabbage leaves to my breasts throughout the day."

A nurse is preparing to administer the measles, mumps, and rubella (MMR) vaccine to a preschooler. The nurse should recognize which of the following statements by the parent as a contraindication to receiving the immunization? A - "My child is allergic to latex." B - "My child is allergic to baker's yeast." C - "My child received amoxicillin recently." D - "My child received an immunoglobulin last month."

D - "My child received an immunoglobulin last month."

A nurse is reinforcing teaching with a client about advance directives. Which of the following statements by the client indicates an understanding of the teaching? A - "Advance directives limit my ability to make health care decisions." B - "A living will determines who will make treatment decisions on my behalf." C - "I am required to choose a family member to be my durable power of attorney." D - "I will be asked by health care staff if I have advance directives each time I am admitted."

D - "I will be asked by health care staff if I have advance directives each time I am admitted." Rational A - Advance directives are legal documents that specify clients' wishes regarding treatment if they become unable to make their own decisions. A client retains the right to refuse treatment following implementation of advance directives. B - A living will is a document that allows clients to specify what treatment they want to receive or refuse in the event they become terminally ill or are unable to make their own decisions. The living will does not determine who makes treatment decisions for the client. C - A durable power of attorney is a document specifying whom a client has chosen to make health care decisions on their behalf should they become unable to do so. Clients can choose anyone they trust to function as their health care surrogate, including a friend or clergy member. D - The Patient Self-Determination Act requires health care personnel to ask clients with every admission if they have advance directives in place that outline their health care wishes if they are unable to make or communicate their own decisions.

A nurse is reinforcing teaching with the guardian of a school-ae child who has type 1 diabetes mellitus and an upper respiratory infection. Which of the following statements by the guardian indicates an understanding of the teaching? A - "I will omit my child's insulin dose during her illness." B - "I will monitor my child's urine for ketones once a day." C - "I will notify the doctor if my child's random glucose is greater than 140." D - "I will check my child's blood glucose level every 3 hours."

D - "I will check my child's blood glucose level every 3 hours." Rational A - A child who has type 1 diabetes mellitus is at increased risk for hyperglycemia during illness and should continue to receive insulin to regulate blood glucose levels. B - The guardian should check the child's urine for ketones every 3 hr when the child is sick to monitor for diabetic ketoacidosis. C - The guardian should notify the provider if the child's blood glucose level is greater than 240 mg/dL, the child is vomiting, or there are ketones in the child's urine. D - The nurse should instruct the guardian to check the child's blood glucose level every 3 hr during illness to monitor for diabetic ketoacidosis.

A nurse is reinforcing discharge teaching with the parent of a newborn. Which of the following statements by the parent indicates an understanding of the teaching? A - "I should place my baby on their stomach when they are sleeping." B - "I will allow my baby to sleep with a stuffed animal." C - "I should avoid offering a pacifier to my baby at bedtime." D - "I will dress my baby in a sleep sack before putting them to bed."

D - "I will dress my baby in a sleep sack before putting them to bed." Rational A - The parent should not place the newborn on their abdomen when they are sleeping because this increases the risk for sudden infant death syndrome (SIDS). The nurse should instruct the parent to place the newborn in the supine position when they are sleeping. B - The parent should not place objects, such as stuffed animals, in the newborn's crib because this can increase the risk for suffocation. C - The parent should offer a pacifier to the newborn at bedtime because this can help soothe the newborn and decreases the risk for sudden infant death syndrome (SIDS). D - The parent should dress the newborn in a sleep sack, which is a one-piece sleeper that keeps the newborn's body covered. This decreases the need to use blankets and quilts to cover the newborn for warmth, thereby decreasing the risk for suffocation.

A nurse is reinforcing discharge teaching with the guardian of a school-age child who has a new prescription for home oxygen therapy. Which of the following statements bby the guardian indicates an understanding of the teaching? A - "I will restrict the length of the oxygen tubing to no longer than 3 feet." B - "I will place the extra oxygen tanks in a horizontal position for storage." C - "I will check the oxygen delivery equipment once every week." D - "I will make sure that electrical devices in the house are grounded."

D - "I will make sure that electrical devices in the house are grounded."

A nurse is reinforcing teaching with a client whose prescription of metoprolol was changed to metoprolol/hydrochlorothiazide. Which of the following statements bbuy the client indicates understanding of the teaching? A - "I will take this medication at bedtime." B - "With the new medication, I should experience fewer adverse effects." C - "I will not have to do anything different because it is the same medication." D - "I will need to maintain an adequate intake of potassium in my diet."

D - "I will need to maintain an adequate intake of potassium in my diet." Rational A - The medication includes a diuretic that will increase urination. To avoid nocturia, the nurse should instruct the client to take the medication early in the day. B - The addition of hydrochlorothiazide will add the adverse effects of a diuretic to the adverse effects of a beta blocker, like metoprolol. There is no indication that the dosage of metoprolol has been decreased. The addition of a diuretic to a beta blocker, metoprolol, is possible if hypertension is inadequately controlled by a beta blocker alone or if the client requires a lower dose of metoprolol to avoid adverse effects. C - The nurse should reinforce with the client that this new medication prescription includes a diuretic that requires an increase in dietary potassium intake. D - Hydrochlorothiazide is a potassium-wasting diuretic. The client will need to have an adequate intake of potassium to avoid hypokalemia. The addition of a diuretic to a beta blocker, like metoprolol, is possible if hypertension is inadequately controlled by a beta blocker alone or if the client requires a lower dose of metoprolol to avoid adverse effects. However, this combination of medications predisposes the client to the additional adverse effects of the diuretic.

A nurse is assisting with the evacuation of clients who have been triaged following a mass casualty event. Which of the following clients should the nurse recommend for first transport to the health care facility? A - A client who has a penetrating head wound and has been assigned a black tag B - A client who has a compound fracture to the left arm and has been assigned a yellow tag C- A client who has multiple abrasions and bruising to the trunk and has been assigned a white tag D - A client who has paradoxical respirations and has been assigned a red tag

D - A client who has paradoxical respirations and has been assigned a red tag

A nurse is caring for a client who has major depressive disorder and has signed a consent form for electronconculsiev therapy (ECT). On the first treatment day, the client tells the nurse that they refuse to undergo ECT. Which of the following responses should the nurse make? A- "Many other clients have had success with this treatment." B - "Your provider would not have recommended this procedure if they did not think it would help." C - "You need this treatment in order to get better." D - "I will notify the provider of your decision."

D - "I will notify the provider of your decision." Rational A - This is a nontherapeutic response because the nurse is expressing disapproval of the client's decision. The nurse should respect the client's right to refuse treatment at any time. B - This is a nontherapeutic response because the nurse is making a defensive response to the client's decision. The nurse should respect the client's right to refuse treatment at any time. C - This is a nontherapeutic response because the nurse is giving the client false reassurance. The nurse should respect the client's right to refuse treatment at any time. D - The nurse should identify that the client has the right to refuse or delay treatment, even if they have signed the consent form. Informed consent is a communication between the provider and client regarding the risks and benefits of treatment.

A nurse is reinforcing teaching about expected withdrawal manifestations with a client who has enrolled in a smoking cessation course. Which of the following client statements indicates an understanding of the information? A - "I can expect my cigarette cravings to go away within 7 to 10 days." B - "I will likely feel like my heart is racing even when I'm resting." C - "I should increase my intake of calories because of the expected weight loss." D - "I will probably feel irritable within 24 hours of my last cigarette."

D - "I will probably feel irritable within 24 hours of my last cigarette."

A nurse is reinforcing teaching during a prenatal class with a group of clients about expected changes during pregnancy. Which of the following statements by one of the clients indicate an understanding of the teaching? A - ​"My nipples and areolae will become pale as my breasts enlarge." B - "I should recognize fetal movement by 12 weeks of gestation." C - "My baby will drop into my pelvis by 35 weeks of gestation." D - "It is common to have a white vaginal discharge."

D - "It is common to have a white vaginal discharge." Rational A - ​As levels of estrogen and progesterone increase, nipples and areolae become more pigmented. B - A multiparous client usually first recognizes fetal movements, or quickening, as early as 16 weeks of gestation. A nulliparous client might not notice these sensations until 18 weeks of gestation or later. C - Lightening occurs between 38 and 40 weeks of gestation, where fundal height drops as the fetus begins to descend and engage in the pelvis. D - Leukorrhea is a grayish-white mucoid discharge that occurs in response to cervical stimulation by estrogen and progesterone.

A nurse is reinforcing discharge teaching with the guardian of a school-age child who has acute lymphocytic leukemia and an absolute neutrophil count of 450/mm3. Which of the following instructions should the nurse include? A - "Allow your child to receive the varicella immunization." B - "Take your child's rectal temperature twice per day." C - "Increase your child's intake of fresh fruits and vegetables." D - "Keep your child away from crowded areas."

D - "Keep your child away from crowded areas."

A nurse is caring for an adolescent client who is a practicing Jehovah's Witness and is scheduled for surgery for a ruptured appendix. The adolescent tells the nurse that based on her religious beliefs, she cannot receive a blood transfusion. Which of the following responses should the nurse make? A - "Why do members of your faith believe this?" B - "You'll only receive blood if you need it." C - "Your parents would want you to have a blood transfusion." D - "Let's discuss the possibility of you needing a blood transfusion with your parents."

D - "Let's discuss the possibility of you needing a blood transfusion with your parents."

A nurse is reinforcing teaching with a client who is scheduled for electroconvulsive therapy (ECT). Which of the following information should the nurse include? A - "You will not require anesthesia when undergoing this procedure." B - "The grand mal seizure will only last for a few minutes." C - "Most clients only require one ECT treatment to relieve symptoms." D - "Memory loss is a common adverse effect of ECT."

D - "Memory loss is a common adverse effect of ECT." Rational A - The nurse should inform the client to expect to receive a general anesthetic and a muscle-paralyzing agent prior to undergoing the ECT procedure to prevent muscle distress and bone fractures. B - The nurse should inform the client that, during ECT, an electrical current will pass through the brain by way of electrodes attached to the head, which will induce a brief seizure of 30 to 60 seconds. The electrical current is thought to stimulate the release of helpful neurotransmitters, which can help to relieve the manifestations of the client's condition. C - The nurse should inform the client that most clients require more than one ECT treatment to relieve recurrent depressive or psychotic manifestations. Clients usually undergo six to 12 treatments. D - The nurse should inform the client that memory loss is a common adverse effect of the therapy and many clients experience memory deficits following treatment. The nurse should inform the client that, although the loss of memory is usually temporary, some clients experience permanent memory loss.

A nurse is reinforcing teaching with a client who is starting to take metformin to treat diabetes mellitus. Which of the following information should the nurse include in the teaching? A - "Take this medication on an empty stomach." B - "This medication can cause constipation." C - "Store this medication in your refrigerator." D - "Monitor for muscle pain while taking this medication."

D - "Monitor for muscle pain while taking this medication." Rational A - The nurse should instruct the client to take this medication with meals to reduce gastrointestinal distress. The client should take immediate release tablets with the morning and evening meals. Extended-release tablets are typically prescribed once daily with the evening meal. B - The nurse should instruct the client that metformin can cause diarrhea, anorexia, and nausea. The provider might prescribe a low dosage at the start of therapy and gradually increase the amount until the target dosage is reached to reduce the occurrence of gastrointestinal distress. C - The nurse should instruct the client that this medication does not need to be refrigerated. A client who takes insulin should store unopened vials of insulin in the refrigerator to maintain potency. D - The nurse should instruct the client to monitor and report early manifestations of lactic acidosis, such as hyperventilation, myalgia, and drowsiness. If manifestations occur, the provider should discontinue the metformin until lactic acidosis is confirmed. Lactic acidosis is a medical emergency and is treated with hemodialysis.

A nurse is reinforcing teaching with the guardian of a child who has a new prescription for ferrous sulfate to treat severe iron-deficiency anemia. Which of the following statements by the guardian indicates an understanding of the instructions? A - "I should administer the medication in one large dose per day." B - "Giving my child citrus juice with the medication will reduce absorption of the iron." C - "My child's medication will be more effective if it is administered with meals." D - "My child's blood count will need to be monitored routinely while taking the medication."

D - "My child's blood count will need to be monitored routinely while taking the medication." Rational A - The nurse should instruct the guardian to administer the ferrous sulfate in two or three doses throughout the day to promote adequate absorption. B - Vitamin C will increase the absorption of iron. The nurse should instruct the guardian to mix the liquid iron preparations with a small amount of juice. C - The nurse should inform the guardian that ferrous sulfate is more effective if given 1 hr before or 2 hr after meals. D - The nurse should instruct the guardian that the child's hemoglobin and hematocrit will need to be monitored routinely through blood tests for the next several weeks to determine the effectiveness of treatment. The nurse should also inform the parent that treatment can take up to 3 months to be effective.

A nurse is caring for a client who has terminal pancreatic cancer. The client states, "I don't think I can go on any longer." Which of the following responses should the nurse make? A - "Can I get you something for the pain?" B - "You should talk about this with your family." C - "Tomorrow will be a better day." D - "Tell me more about the way you are feeling."

D - "Tell me more about the way you are feeling."

A nurse is reinforcing teaching with the guardian of a school-age child who has ADHD and a new prescription for methylphenidate. Which of the following statements should the nurse make? A - "This medication can cause your child to experience ringing in the ears." B - "Administer the medication 1 hour before bedtime." C - "Hot flashes are an expected adverse effect of this medication." D - "Weigh your child twice each week while they are taking this medication."

D - "Weigh your child twice each week while they are taking this medication." Rational A - Tinnitus is not an adverse effect of methylphenidate. The nurse should instruct the guardian about expected adverse effects of methylphenidate. These include insomnia, anorexia, dry mouth, and growth suppression. B - Methylphenidate is a central nervous system stimulant and can cause insomnia, necessitating administration no later than 1800. The nurse should instruct the guardian to administer methylphenidate in the morning so the medication is most effective during school hours. C - Hot flashes are a rare adverse effect that should be reported to the provider immediately. The nurse should instruct the guardian about expected adverse effects of methylphenidate. These include insomnia, anorexia, dry mouth, and growth suppression. D - The nurse should instruct the guardian to weigh the child twice each week and notify the provider if weight loss occurs. Weight loss is a potential adverse effect of methylphenidate. Methylphenidate is a central nervous system stimulant, which decreases the child's appetite and can also cause growth suppression.

A nurse is discussing health screening guidelines with an older adult client. Which of the following statements should the nurse include? A - "You should have a screening for glaucoma every 5 years." B - "You should have a physical examination every other year." C - "You should have your hearing checked every 2 years." D - "You should have a pneumococcal immunization every 10 years."

D - "You should have a pneumococcal immunization every 10 years."

A nurse is reviewing the medical records of a group of clients in an outpatient clinic. For which of the following clients should the nurse anticipate scheduling a colonoscopy to detect colorectal cancer (CRC)? A - A 53-year-old client who had a sigmoidoscopy 3 years ago B - A 34-year-old client who reports a new onset of constipation C - A 32-year-old client who had a double-contrast barium enema 2 years ago D - A 51-year-old client who is being seen for an annual physical examination

D - A 51-year-old client who is being seen for an annual physical examination Rational A - Screening recommendations for adults 45 years of age and older who are at average risk for CRC include the option of a flexible sigmoidoscopy every 5 years. B - Unless the client has risk factors for CRC or the constipation becomes chronic, the nurse should not anticipate scheduling this client for a colonoscopy. C - Screening recommendations for adults 50 years of age and older who are at average risk for CRC include the option of a double-contrast barium enema every 5 years. D - Starting at 45 years of age, clients considered to be at average risk and who have no family history of CRC should have a screening colonoscopy every 10 years. Clients who are at increased risk for CRC might have the screening earlier and more often.

A nurse is collecting data from four clients. The nurse should recognize that which of the following clients is demonstrating a maladaptive defense mechanism. A - A client throws a purse to the ground after misplacing a wallet. B - A client who is frustrated with clinic staff sits in the lobby writing angry comments in a journal. C - A client ignores the thought of pain when scheduling oral surgery. D - A client who has a fear of needles regularly forgets to schedule necessary laboratory appointments.

D - A client who has a fear of needles regularly forgets to schedule necessary laboratory appointments. Rational A - This client is demonstrating an adaptive use of displacement to shift negative feelings to a less threatening object or person. An example of maladaptive use of displacement would be injuring a stranger because of angry feelings toward a friend. B - This client is demonstrating an adaptive use of sublimation to substitute an immature impulse to lash out at staff with the more mature activity of journaling about their feelings. The use of sublimation is adaptive and positive. C - This client is demonstrating an adaptive use of suppression, which is the voluntary denial of anxiety-producing thoughts. This can be used as a method of minimizing anxiety. An example of maladaptive use of suppression would be for a client to avoid thinking about a medical problem that requires attention rather than seeking help. D - Repression occurs when a client deals with anxiety by unconsciously putting the unacceptable or stress-producing thought out of their consciousness. In this case, the client's use of repression is maladaptive because the client is not receiving the needed health care.

A nurse on an inpatient unit is collecting data from a group of clients. Which of the following findings should the nurse report to the provider? A - A client who has schizophrenia and is using neologisms B - A client who has bipolar disorder and is experiencing flight of ideas C - A client who has depression and avoids making eye contact D - A client who has borderline personality disorder and is pacing restlessly

D - A client who has borderline personality disorder and is pacing restlessly

A nurse is receiving change-of-shift report on a group of clients. Which of the following clients should the nurse plan to see first? A - A client who had a urine output of 280 mL on the previous 8 hr shift B - A client who reports a pain level of 4 on a 0 to 10 pain scale 1 hr following analgesia C - A client who had a laparoscopic appendectomy 2 hr ago and is vomiting D - A client who has tachycardia and a respiratory rate of 32/min

D - A client who has tachycardia and a respiratory rate of 32/min Rational A - A urine output of 280 mL is within the expected reference range of greater than or equal to 30 mL/hr. The nurse should continue to monitor the client's output for any changes. However, the nurse should plan to see another client first. B - A pain level of 4 on a 0 to 10 pain scale is moderate pain and is a common occurrence 1 hr after a procedure requiring analgesia. The nurse should address the client's pain. However, the nurse should plan to see another client first. C - Nausea and vomiting following surgery is common. The nurse should monitor the client and likely administer an antiemetic. However, the nurse should plan to see another client first. D - When using the airway, breathing, circulation approach to client care, the nurse should identify that a client who has tachycardia and a respiratory rate of 32/min is at increased risk for respiratory compromise and cardiac arrest. Therefore, the nurse should plan to see this client first.

A nurse in a prenatal clinic is collecting data from four clients. The nurse should report which of the following client findings to the provider? A - A client who is at 28 weeks of gestation has nonpitting edema in the ankles. B - A client who is at 38 weeks of gestation reports difficulty sleeping. C - A client who is at 20 weeks of gestation has varicose veins. D - A client who is at 33 weeks of gestation reports a severe headache.

D - A client who is at 33 weeks of gestation reports a severe headache. Rational A - Clients who are at 28 weeks of gestation often have nonpitting edema in the lower extremities or ankles due to the enlarging uterus decreasing venous return. B - Clients who are at 38 weeks of gestation often have insomnia due to shortness of breath, urinary frequency, fetal movement, and other discomforts. C - Clients who are at 20 weeks of gestation often have varicose veins due to the relaxation of the smooth muscles of the veins and vasocongestion. D - The nurse should identify that a client who is at 33 weeks of gestation with a severe headache has an indication of gestational hypertension or preeclampsia. Therefore, the nurse should report this finding to the provider.

A nurse is reviewing policies with a newly hired staff member about client refusal of treatment. Which of the following information should the nurse include? A - A client must have advance directives on file to refuse treatment. B - A family can overrule a competent adult's choice to refuse life-saving treatment. C - A client attempting to cause harm can refuse chemical restraint. D - A court can overturn a parent's decision to refuse treatment for a child.

D - A court can overturn a parent's decision to refuse treatment for a child. Rational A - Advance directives are not required for a client to refuse treatment. Advance directives are helpful in showing decisions made by a once-competent individual. A client can determine in advance directives which treatments should not be administered in the future. B - Competent clients have the right to refuse treatment. The nurse should encourage communication between the client and family to promote understanding, but the decision is up to the client. When competency is questioned, the nurse should consider the client's advance directives. C - A client who is actively attempting to harm themself or another person can be given sedation against their will. A court hearing may be required to determine whether a client is competent to refuse medication for mental health disorders in non-emergent situations. D - A parent can choose to refuse treatment for a child for many reasons, including religious preference. However, the nurse has a responsibility to ensure the safety of the child. Legal intervention can be required to overturn the parent's decision if it threatens the child's life.

A nurse in a substance use disorder treatment center is discussing community resources with a newly licensed nurse. The nurse should identify that a referral to Al-Anon is appropriate for which of the following individuals? A- An older adult client who has alcohol use disorder B - A coworker who is considering an alcohol treatment program C - A member of Alcoholics Anonymous who recently resumed alcohol intake D - A family member of a client who has alcohol use disorder

D - A family member of a client who has alcohol use disorder Rational A - The nurse should identify that an older adult client who has alcohol use disorder will benefit from a referral to Alcoholics Anonymous rather than Al-Anon. Alcoholics Anonymous uses a 12-step program to assist clients in overcoming alcohol use disorder. B - The nurse should identify that a coworker who has alcohol use disorder will benefit from a referral to Alcoholics Anonymous rather than Al-Anon. Alcoholics Anonymous uses a 12-step program to assist clients in overcoming alcohol use disorder. C - The nurse should work with the health care team to identify the cause of the client's resumption of alcohol intake. The nurse should then recommend community resource referrals, such as a new referral to Alcoholics Anonymous, rather than Al-Anon. Alcoholics Anonymous uses a 12-step program to assist clients in overcoming alcohol use disorder. D - The nurse should identify that a referral to the community-based self-help group Al-Anon is appropriate for family members or friends of a client who has alcohol use disorder. For adolescent family members of a client who has alcohol use disorder, the nurse can recommend a referral to Alateen. Al-Anon and Alateen are based on the 12-step model originated by Alcoholics Anonymous, a self-help group for clients who have alcohol use disorder.

A charge nurse is observing the actions of other nurses on the unit. Which of the following actions should the charge nurse identify as adhering to the code of ethics for nurses? A - A nurse informs a client's family member that the client refused morning medications. B - A nurse spends less time with a client who frequently tells long stories. C - A nurse delegates the task of inserting a urinary catheter for a client to an assistive personnel (AP). D - A nurse declines a new client's request to connect and communicate on an online social network.

D - A nurse declines a new client's request to connect and communicate on an online social network. Rational A - The nurse should only discuss a client's personal information, diagnosis, treatment, or assessment with members of the health care team who are involved in that client's care. This upholds the ethical principle of confidentiality. B - The nurse should try to divide their time evenly between clients according to each client's individual needs, regardless of the client's personal interactions with the nurse. The nurse can schedule tasks in the client's room to allow time to listen to the stories while performing care because this upholds the ethical principle of justice. C -The nurse should only delegate interventions and tasks that are within the delegate's training and ability according to each state's Nurse Practice Act. The task of inserting a urinary catheter should not be delegated to an AP because this action is not within the AP's range of function. Ensuring that delegated tasks are within the AP's range of function upholds the ethical principle of nonmaleficence. D - The nurse should not connect with a new client on social media platforms because this can interfere with the nurse's ability to maintain a therapeutic relationship. Friendship with a client compromises the ability of the nurse to remain objective. This action upholds the nursing code of ethics.

A nurse is reviewing the plan of care for a client who is malnourished and has several referrals. Which of the following services should the nurse expect a social worker to provide? A - Showing a client how to use special utensils for cooking B - Prescribing nutritional requirements for the client C - Recommending food choices for the client D - Accessing food resources for the client

D - Accessing food resources for the client Rational A - The nurse should expect an occupational therapist to provide functional assistance to perform ADLs. An occupational therapist can provide a client with special utensils for cooking and eating if the client's grip is impaired. B - The nurse should expect the provider to prescribe nutritional requirements for the client. The provider should select to increase or restrict certain nutrients in the diet based on the client's diagnosis and individualized needs. The provider's prescription guides how other interprofessional team members will assist the client to meet nutritional needs. C - The nurse should expect a dietitian to educate the client about following a prescribed therapeutic diet. The dietician may assess cultural preferences affecting the client's diet, review the client's eating habits, calculate the client's calorie needs, and provide information about food selection and meal preparation. D - Following an illness, clients might require special supplies or face financial hardship. The social worker helps connect the client with financial support and resources for self-care and may be asked to determine the safety of the client's living conditions. Social workers also provide counseling for a client facing role or life transitions.

A nurse is caring for a client who received an injection of penicillin G. The client experiences dyspnea and states, "My tongue feels swollen." Which of the following should be the nurse's priority action? A - Obtain IV fluids for administration. ​B - Record the observed data in the client's medical record. C - Deliver a dose of methylprednisolone IV. D - Administer epinephrine subcutaneously.

D - Administer epinephrine subcutaneously. Rational A - A goal of treatment in anaphylaxis is to raise the blood pressure. Fluid administration, along with vasopressors, can correct hypotension during an anaphylactic response. Vasopressor medications, such as vasopressin, act to raise blood pressure quickly in emergencies, such as anaphylaxis by vasoconstriction of the coronary arteries, and can be given IM or subcutaneously. Fluid therapy is one component of treatment, but it is not the priority action for the nurse. Normal saline is isotonic and would be the expected intravenous fluid for the nurse to give to increase fluid volume. The client's airway is the priority in accordance with Maslow's hierarchy rather than circulation, which is provided by blood volume and pressure. B - ​It is critical to the safety of the client that any anaphylactic event be recorded in the medical record. The record should clearly indicate which medication the provider identified as the cause of the event. This will be completed when the client has been treated for immediate life-threatening manifestations. This is not the nurse's priority action. C - The nurse should administer methylprednisolone, a glucocorticoid, to inhibit phagocytosis and decrease the inflammatory response. This action will improve respirations and assist in minimizing the effects of shock. However, this is not the nurse's priority action. D - Epinephrine is the medication of choice in response to anaphylaxis. It is administered subcutaneously into the outer thigh. Epinephrine can be given through clothing to prevent delay of administration. The effect of the epinephrine is to act on adrenergic receptors, causing bronchodilation of the lungs and an elevation of blood pressure. The client's airway is the priority in accordance with Maslow's hierarchy of needs. By stimulating both alpha- and beta-adrenergic receptors to cause these effects, it accomplishes more of the goals of treatment of anaphylaxis than any other single therapy. This is the nurse's priority action.

A nurse is preparing to administer a parenteral dose of ketorolac to a child. Which of the following action should the nurse take? A - Discard the solution if it is clear with a yellow tinge. B - Give ketorolac with an oral NSAID for severe pain. C - Determine if the client has an allergy to eggs prior to administration. D - Administer the medication deep into the muscle.

D - Administer the medication deep into the muscle. Rational A - Ketorolac is a clear solution that has a yellow tinge. The nurse should not discard the solution. B - Ketorolac is a first-generation NSAID. The nurse should not administer an oral NSAID with this medication because of the increased risk for adverse GI effects and hypersensitivity. C - Ketorolac can be administered to children who have an allergy to eggs. There is no known contraindication for administering ketorolac to someone who has an allergy to eggs. D - The nurse should administer ketorolac slowly and deeply into the muscle to optimize absorption.

A nurse is caring for a client who has diabetes mellitus and takes insulin detemir twice each day. Due to an increased glucose level, the provider orders a one-time dose of regular insulin concurrently. Which of the following actions should the nurse take? A - Draw up the regular insulin into the syringe before drawing up the insulin detemir. B - Draw up the insulin detemir into the syringe before drawing up the regular insulin. C - Administer the insulin detemir by an alternative route of administration. D - Administer the two insulins using separate syringes.

D - Administer the two insulins using separate syringes. Rational A - When two insulins are compatible, the nurse should draw up the regular insulin first. However, this action is not appropriate in this situation because the nurse should not mix insulin detemir or insulin glargine with any other insulin. B - When two insulins are compatible, the nurse should draw up the regular insulin first. However, this action is not appropriate in this situation because the nurse should not mix insulin detemir or insulin glargine with any other insulin. C - Insulin detemir is available as a subcutaneous solution for injection. It cannot be given IM or through an insulin infusion pump. Regular insulin can be administered subcutaneously or by IV routes. D - Mixing two insulins in the same syringe is appropriate for administering regular insulin with neutral protamine hagedorn (NPH) insulin. However, the nurse should not mix insulin detemir or insulin glargine with any other insulin.

A school nurse is caring for a child who is experiencing an acute asthma attack. Which of the following medications should the nurse expect to administer? A - Zafirlukast B - Budesonide C - Montelukast D - Albuterol

D - Albuterol Rational A - Zafirlukast is a leukotriene modifier that is used as prophylaxis and maintenance therapy for asthma and to prevent exercise-induced bronchospasm. It is not considered a rescue medication for an acute asthma attack. B - Budesonide is a glucocorticoid that is used for long-term control and prophylaxis of chronic asthma. It is not considered a rescue medication for an acute asthma attack. C - Montelukast is the most commonly prescribed leukotriene modifier. It is used as prophylaxis and maintenance therapy for asthma and to prevent exercise-induced bronchospasm. It is not considered a rescue medication for an acute asthma attack. D - The nurse should expect to administer albuterol to a child who is experiencing an acute exacerbation of asthma because it is considered a rescue medication. Rescue medications are classified according to their rapid onset of action. Asthma is a chronic inflammatory disorder of the airways. Asthmatic episodes are associated with airflow limitation or reversible obstruction. Albuterol is a beta-adrenergic agonist used for the treatment of acute exacerbations of asthma by promoting bronchodilation and suppressing histamine release in the lungs. This medication can be given by inhalation, orally, or as a parenteral preparation. The inhaled medication has a more rapid onset of action than the oral form and reduces the adverse effects of irritability, tremors, nervousness, and insomnia.

A nurse is preparing to administer phytonadione IM to a newborn. Which of the following actions should the nurse plan to take? A - Inject the medication into the newborn's deltoid muscle. B - Use a 1-inch needle to ensure the medication is given deep IM. C - Administer the medication when the newborn is at least 1 day old. D - Apply gentle pressure to the site after injecting the medication.

D - Apply gentle pressure to the site after injecting the medication. Rational A - The nurse should administer the medication in the middle third of the vastus lateralis muscle. A newborn's deltoid muscle does not have an adequate amount of tissue for an IM injection. B - The nurse should use a 5/8 inch needle to inject the medication at a 90° angle. Using a 1-inch needle to administer any IM medication for a newborn is unsafe and therefore contraindicated. C - The nurse should administer the medication soon after birth or after the first breastfeeding. D - The nurse should apply gentle pressure to the area following medication administration with dry gauze to minimize pain and bleeding at the site.

A nurse is contributing to the plan of care for a newborn who was circumcised with a plastic bell device. Which of the following actions should the nurse include in the plan? A - Apply petrolatum gauze to the site with each diaper change. B - Use a mild soap and warm water to wash the site twice each day after the procedure. C - Apply the diaper snugly over the site. D - Apply pressure with sterile gauze if bleeding occurs at the site.

D - Apply pressure with sterile gauze if bleeding occurs at the site.

A nurse is collecting data on a client who is scheduled for a cardiac catheterization. Which of the following laboratory levels should the nurse review prior to the procedure? A - Albumin B - Phosphorus C - TSH D - BUN

D - BUN Rational BUN levels indicate kidney function. Contrast media used during cardiac catheterization can cause renal failure. The nurse should review this laboratory level to determine if the client can tolerate the IV contrast dye during the procedure.

A nurse is assisting with the care of a client who has an estimated 15% of total body surface area burn injury, is taking oral nutrition, and reports following a strict vegan diet. Which of the following food choices should the nurse recommend? A - Tuna salad B - Scrambled eggs C - Cream of broccoli soup D - Black beans and rice

D - Black beans and rice Rational A - Tuna is high in protein, which is necessary for tissue repair following a burn injury; however, a client who follows a strict vegan diet only consumes protein from plant-based foods. B - Scrambled eggs are high in protein, which is necessary for tissue repair following a burn injury; however, a client who follows a strict vegan diet excludes any type of animal products, including eggs, cheese, and yogurt. C - Cream-based soups can provide protein to promote tissue repair following burn injury; however, a client who follows a strict vegan diet does not consume any foods derived from animal products, such as milk or other dairy products. D - The client needs an increased protein intake to aid in the promotion of tissue healing following burn-injury or other disorders that cause metabolic stress. A client who follows a strict vegan diet should obtain adequate protein from nuts and legumes to promote balanced nutrition and tissue repair. Black beans and rice contain complementary proteins that form a complete protein when consumed together.

A nurse is reveiwing the medical record for a client who has sepsis. Which of the following findings is the priority to report to the provider? A - Temperature B - WBC count C - Creatinine D - Blood pressure

D - Blood pressure Rational A - A temperature of 38.3° C (101° F) is above the expected reference range of 36° to 38° C (96.8° to 100.4° F) and indicates the client is at risk for hyperthermia. However, another finding is the priority. Temperature can be within or above the expected reference range in sepsis, and older adult clients can experience hypothermia. Decreased body temperature can also be a manifestation of late septic shock. B - A WBC count of 12,500/mm3 is above the expected reference range of 5,000 to 10,000 mm3 and indicates infection. However, another finding is the priority. Gram-negative or gram-positive bacteria are the most likely cause of infection and subsequent sepsis. C - A creatinine of 1.4 mg/dL is above the expected reference range of 0.5 to 1.3 mg/dL and indicates kidney impairment due to hypovolemia. However, another finding is the priority. The nurse should monitor a client who has sepsis for a urine output of 20 to 30 mL/hr. D - When using the airway, breathing, circulation approach to client care, the nurse should determine that the priority finding to report to the provider is a blood pressure of 84/62 mm Hg. A systolic blood pressure below 90 mm Hg indicates hypotension. Complications of sepsis include hypovolemia due to fluid leak from capillaries or blood loss through impaired coagulation. This client requires emergency treatment such as fluid replacement and vasopressors to increase perfusion to vital organs.

A nurse is collecting data from a client who has been taking lithium for 3 months. Which of the following findings is the priority to report to the provider? A - Impaired memory B - Abdominal bloating C - Headache D - Blurred vision

D - Blurred vision Rational A - The nurse should report the client's impaired memory to the provider as this can cause frustration for the client; however, there is another finding that is the priority. The nurse should reinforce with the client to report the development of slurred speech or hand tremors as these are manifestations of lithium toxicity. Impaired memory can occur during early treatment with therapeutic serum lithium levels, and often subsides. B - The nurse should report abdominal bloating to the provider as this can cause discomfort for the client; however, there is another finding that is the priority. The nurse should reinforce with the client to report increased urinary output or diarrhea as these are manifestations of lithium toxicity. Abdominal bloating can occur during early treatment with therapeutic serum lithium levels, and often subsides. C - The nurse should report a headache to the provider as this can cause discomfort for the client; however, there is another finding that is the priority. The nurse should reinforce with the client to report seizures or sedation, which are findings of lithium toxicity. Headaches can occur during early treatment with therapeutic serum lithium levels, and often subsides. D - Blurred vision indicates the client is at greatest risk for injury from lithium toxicity; therefore, this is the priority finding for the nurse to report to the provider. Other adverse manifestations of toxicity at lithium levels greater than 2 mEq/L include ataxia, giddiness, tinnitus, muscle twitching, and coarse tremors. If not treated, lithium toxicity can lead to seizures, coma, and death.

A nurse is collecting data from a client who has chronic kidney disease with hyperkalemia. Which of the following findings should the nurse expect related to hyperkalemia? A - Polyuria B - Constipation C - Anorexia D - Bradycardia

D - Bradycardia Rational A - Polyuria is a manifestation of hypokalemia. B - Constipation is a manifestation of hypokalemia. C - Anorexia is a manifestation of hypokalemia. D - The client who has hyperkalemia can have an irregular, slow heart rate, known as bradycardia.

A nurse is reinforcing teaching about interventions to treat breast engorgement with a client who is breastfeeding. Which of the following instructions should the nurse include in the teaching? A - Avoid using a breast pump during times of engorgement. B - Apply warm compresses to the breasts after feedings. C -Decrease daily fluid intake. D - Breastfeed the newborn at least every 2 hr.

D - Breastfeed the newborn at least every 2 hr.

A nurse is caring for a client who has returned to the clinic after receiving abnormal cytology findings from a Papanicolaou (Pap) test. Which of the following procedures should the nurse anticipate the client will undergo? A - Dilation and evacuation B - Transvaginal ultrasound C - Human chorionic gonadotropin (hCG) test D - Colposcopy with directed biopsy

D - Colposcopy with directed biopsy Rational A - Dilation and evacuation is performed as a diagnostic procedure when a client has excessive vaginal bleeding. It can also be performed as a therapeutic procedure to remove retained placenta, or tissues that have hypertrophied or are retained following an abortion. B - A client undergoes a transvaginal ultrasound to visualize the thickness of the uterus and structure of organs as well as to determine pregnancy at the early stages. C - A client undergoes a hormone hCG test to determine pregnancy. It is also used to monitor clients who are pregnant and have elevated risk factors for miscarriage or chromosomal abnormalities of the fetus. For clients who are not pregnant, it can identify the presence of hCG-producing cancers. D - Follow-up for a client who has an abnormal Pap test includes colposcopy and directed biopsy. The provider will obtain a cervical tissue specimen and send it for testing to identify precancerous or cancerous cells.

A nurse is assisting with planning care for a client who has had a stroke. The nurse should initiate a referral to an occupational therapist for which of the following tasks? A - Assisting with ambulatory devices B - Introducing a bladder training program C - Incorporating RDAs D - Completing ADLs

D - Completing ADLs Rational A - A physical therapist assists a client with mobility skills, including the use of ambulatory devices such as a walker or a cane. B - The nurse should assist the client with bowel and bladder training. This training can include a regular toileting schedule and applying light pressure to the bladder to facilitate urination, which is known as Crede's maneuver. C - A dietitian should help the client meet recommended dietary allowances (RDAs). The dietitian also can assess the client for dysphagia and develop meal plans based on the client's needs. D - An occupational therapist assists the client to develop fine motor skills and coordination, such as improving hand strength and hand movements. The occupational therapist focuses on self-management of ADLs, such as skills needed for eating, hygiene, and dressing.

A nurse is contributing to the plan of care for a 6-month-old infant who is to have continuous pulse oximetry monitoring. Which of the following interventions should the nurse recommend including? A - Reposition the sensor to a new site once every 24 hr. B - Secure the oximetry sensor to the infant's wrist. C - Apply conduction gel to the skin before attaching the sensor. D - Cover the oximetry sensor with clothing.

D - Cover the oximetry sensor with clothing. Rational A - The pulse oximetry sensor should not remain in one location for an extended period of time. Therefore, the nurse should move the sensor to a new site every 4 to 8 hr to prevent pressure necrosis of the skin. B - The pulse oximeter should be placed around the infant's hand or foot to obtain an accurate pulse oximetry reading. C - The pulse oximeter uses a sensor and a photodetector to measure oxygen in hemoglobin. Conduction gel would interfere with the reading because it would not allow the sensor to attach properly to the skin. D - The nurse should cover the sensor with clothing to prevent outside light from causing an altered or false reading. The nurse should also monitor the site frequently for color, temperature, and a pulse to prevent tissue necrosis.

A nurse in a long-term care facility is preparing for an interprofessional team conference. Which of the following actions should the nurse take? A - Assist the provider in determining which medications are appropriate for the client. B - Calculate a client's energy requirements based on a specific disease process. C - Review insurance requirements for the client's length of stay. D - Create a list of needs based on questions or concerns from the client.

D - Create a list of needs based on questions or concerns from the client. Rational A - The nurse should identify this as a task for a pharmacist. The pharmacist has in-depth knowledge of medications, interactions, and effects. The pharmacist can recommend appropriate prescriptions and dosages to the provider. B - The nurse should identify this as a task for a dietitian. The dietitian has advanced knowledge of disease and how to determine client calorie needs. The dietitian can design special diets and serve as a resource person to staff. The nurse should reinforce information provided by the dietitian. C - The nurse should identify this as a task for a case manager. The case manager oversees client care to ensure that needs are covered and prevent financial waste. A case manager or social worker is knowledgeable of client insurance requirements and other financial needs. D - The nurse is the client advocate. As the direct caregiver, the nurse is often aware of needs and issues affecting the client that other members of the interprofessional team are not. The nurse should ensure the client's concerns are addressed during the interprofessional conference.

A nurse is collecting data on a client who is taking lithium for the treatment of bipolar disorder. Which of the following findings should the nurse identify as an adverse effect of lithium therapy? A - Hypoglycemia B - Hepatic toxicity C - Decreased urination D - Fine hand tremors

D - Fine hand tremors Rational A - Hyperglycemia, not hypoglycemia, is an adverse effect of lithium therapy. B - Renal toxicity, not hepatic toxicity, is an adverse effect of lithium therapy. C - Lithium therapy can cause an increase in urination, not a decrease. D - Fine hand tremors, especially in the fingers, are an adverse effect of lithium therapy.

A nurse is collecting data from a client who is in her second trimester of pregnancy. Which of the following findings should the nurse report to the provider? A - Increased leukorrhea B - Hyperpigmentation of the face C - Varicose veins D - Frequent uterine contractions

D - Frequent uterine contractions

A nurse is assisting with the transfer of a client from the intensive care unit to a cardiac step-down unit. To facilitate the transfer, which of the following actions should the nurse initiate? A - Write prescriptions for the client to resume home medications not prescribed by the provider. B - Make a list of the medications the client received in the intensive care unit. C - Prepare a discharge medication list for the client. D - Create a list of the client's current medication prescriptions.

D - Create a list of the client's current medication prescriptions. Rational A - This action is outside the scope of practice for a practical nurse. Prescriptions to continue home medications should come from the client's provider. Home medications might no longer be necessary or might be incompatible with current medications. B - This information is available in the medication administration record of the client's medical record. Medications received while in the intensive care unit might be different than the client's current medications. C - The nurse should prepare a discharge medication list for the client at the time of discharge. The client's current medications might be different than discharge prescriptions. D - The nurse should create a list of all of the client's current medications. This allows the nurse to check for any duplications or incompatibilities. The nurse should compare the list of current medications with the list of newly prescribed medications. Once the data is collected, the provider will evaluate the client's needs.

A nurse is collecting data from a client who has malnutrition. Which of the following findings should the nurse identify as an indication of malnutrition? A - Increased tendon reflexes B - Increased muscle tone C - Decreased risk of infection D - Decreased mental status

D - Decreased mental status Rational A - A client who has malnutrition can have decreased or absent knee and ankle reflexes due to electrolyte changes that affect neuromuscular activity. Other nervous system findings of malnutrition include poor attention span and paresthesias. B - A client who has malnutrition can have decreased muscle tone. Electrolytes, such as potassium, magnesium, and calcium, are important for neuromuscular activity and are frequently inadequate with nutritional deficits. C - A client who has malnutrition can have an increased rate of infection. To reduce the risk for infections, the client needs an adequate intake of all nutrients, especially vitamins and calories. Adequate protein, calories, and vitamins, such as vitamins B and C, improve tissue healing. D - A client who has malnutrition can have a decreased mental status because the brain requires glucose to function. When the body lacks adequate glucose, the body will metabolize tissue such as muscle and fat. The resulting metabolic acidosis can decrease the client's mental status. Nutritional deficits of fluids will result in dehydration, which can also cause a decrease in mental status.

A nurse is collecting data form a client who is taking levodopa/carbidopa. The nurse should identify which of the following findings as an expected response to this medication? A - Reduced drowsiness B - Increased memory C - Improved mood D - Decreased tremors

D - Decreased tremors Rational A - Levodopa/carbidopa is a dopamine agonist used to treat Parkinson's disease. Drowsiness is an adverse effect of this medication B - Levodopa/carbidopa is a dopamine agonist used to treat Parkinson's disease. Memory loss is an adverse effect of this medication. C - Levodopa/carbidopa is a dopamine agonist used to treat Parkinson's disease. Anxiety and confusion are adverse effects of this medication. D - Levodopa/carbidopa is a dopamine agonist used to treat Parkinson's disease. Levodopa increases dopamine synthesis and carbidopa increases the effects of levodopa. Therapeutic effects include reduced tremors, decreased rigidity, increased balance, and improved performance of activities of daily living.

A nurse is caring for a client who has a prescription for phenazopyridine. Which of the following should the nurse identify as a therapeutic effect of the medication? A - Reduces bacteria in the urinary tract B - Suppresses urge to void C - Prevents nerve stimulation to the bladder muscle D - Decreases pain during urination

D - Decreases pain during urination Rational A - Bacteria in the urinary tract is reduced with the use of an antimicrobial medication, such as fosfomycin. B - The urge to void is suppressed with the use of an antispasmodic for urinary incontinence, such as oxybutynin. C - Nerve stimulation to the bladder muscle is prevented with the use of an antispasmodic, such as hyoscyamine. D - Phenazopyridine reduces pain and burning during urination by exerting an anesthetic effect on the mucosa of the urinary tract.

A nurse is caring for a client who has obsessive-compulsive disorder (OCD) and is experiencing anxiety. Which of the following actions should the nurse take? A - Ignore distorted thinking if it occurs. B - Encourage the client to participate in group activities. C - Offer detailed explanations when providing information. D - Determine the client's level of anxiety.

D - Determine the client's level of anxiety. Rational A - The nurse should identify that clients who are experiencing anxiety can have distorted thinking. Rather than ignore the distorted thinking, the nurse should reinforce reality when this occurs. B - The nurse should decrease environmental stimuli for a client who has OCD and is experiencing anxiety. Therefore, the nurse should discourage this client from participating in group activities. C - The nurse should provide brief, simple instructions to help the client retain information and to reduce overall anxiety. Providing detailed information to a client who is experiencing anxiety can cause increased stress. D - The nurse should determine whether the client is experiencing a mild, moderate, or severe level of anxiety to identify the necessary actions to take when providing care for the client.

A nurse is collecting data from a client who is experiencing alcohol withdrawal. Which of the following manifestations should the nurse expect? A - Increased yawning B - Bradycardia C - Hypersomnia D - Diaphoresis

D - Diaphoresis

A nurse is assisting with the planning of an interdisciplinary care conference for a newly admitted client who is in the acute stage of anorexia nervosa. Which of the following members of the interdisciplinary treatment team should the nurse include? A - Occupational therapist B - Pharmacist C - Nurse researcher D - Dietitian

D - Dietitian

A nurse is assisting the charge nurse with developing an in-service about caring for clients who have internal sealed radiation implants. Which of the following information should the nurse include? A - Restrict the time pregnant women are allowed in the client's room to 15 min. B - Pick up a radiation implant with a double-gloved hand if it becomes dislodged. C - Limit time spent in the client's room to 2 hr during an 8 hr shift. D - Dispose of radiation implants in a lead container.

D - Dispose of radiation implants in a lead container.

A nurse is reinforcing teaching with a client who has a new prescription for alendronate 35 mg PO to prevent osteoporosis. Which of the following instructions should the nurse include? A - Take the medication once per month. B - Lie down for 30 min after taking the medication. C- If the medication causes heartburn, take it with an antacid. D - Drink a full glass of water when taking the medication.

D - Drink a full glass of water when taking the medication. Rational A - The client should take a 35-mg dose once per week to reduce the risk for osteoporosis. Some clients take smaller doses of alendronate daily. B - Clients who take alendronate are at risk for esophagitis and other GI disturbances. The client should remain upright for at least 30 min after taking this medication. C - After the client takes the medication, the client should avoid taking antacids or other medications for at least 30 min. The client should also avoid taking medications or foods that contain calcium for 2 hr. D - The client should drink at least 240 mL (8 oz) of water when taking alendronate to reduce the risk of esophagitis. Other beverages decrease medication absorption. The client should avoid eating or drinking immediately after taking the medication. Alendronate is a bisphosphonate that inhibits bone resorption.

A nurse is caring for a client who is postoperative and has an epidural infusion. Which of the following findings should the nurse recognize as the priority? A - Pruritus B - Nausea C - Urinary retention D - Dyspnea

D - Dyspnea

A nurse is caring for a client who is receiving chemotherapy. The client mentions that she has a loss of appetite because she has sores in her mouth and that food no longer tastes good. Which of the following suggestions to the client should the nurse make? A - Drink water before and after each bite. B - Consume foods that are served hot rather than cold. C - Rinse with a glycerin-based mouthwash before meals. D - Eat several, small-portioned meals daily.

D - Eat several, small-portioned meals daily.

A nurse is contributing to the plan of care for a client who has bulimia nervosa and is slightly overweight. Which of the following interventions should the nurse recommend added to the plan of care? A - Reinforce teaching with the client about manifestations of hyperkalemia. B - Use aversion therapy with the client. C - Avoid discussing triggers that cause the client to binge. D - Encourage the client to participate in meal planning.

D - Encourage the client to participate in meal planning. Rational A - Hypokalemia is a possible complication of bulimia nervosa that can occur due to self-induced vomiting. The nurse should reinforce teaching about the manifestations of hypokalemia, such as fatigue, leg cramps, constipation, and nausea. B - Aversion therapy is intended to suppress an undesirable behavior and is used to treat some clients who have alcohol use disorder or engage in other unwanted behaviors. Aversion therapy is not used to treat eating disorders. Positive reinforcement is the preferred treatment for bulimia nervosa. C - The client should be encouraged to discuss triggers for binge eating in order to examine the thinking that leads to activities associated with the client's bulimia disorder. This will help the client understand how unresolved emotions can cause maladaptive eating behaviors. D - The client should be encouraged to participate in meal planning. The nurse should stress the importance of nutritionally balanced, low-fat meals containing foods that the client likes. Creating and following a meal plan can assist the client to achieve desired treatment outcomes.

A nurse is providing end-of-life care to a client who has metastatic lung cancer. Which of the following interventions should the nurse take to support the client's family? A - Discourage family members from long visits so the client can rest. B - Avoid discussing the manifestations of impending death with the client's family. C - Encourage family members to feed the client. D - Encourage the family to offer the client a back massage.

D - Encourage the family to offer the client a back massage. Rational A - The nurse should encourage family members to remain with a client who is at the end of life. This can involve making exceptions to visitation policies and providing privacy for the family. Supporting the grieving family is important during end-of-life care. B - The nurse should discuss manifestations of impending death with the client's family to reduce anxiety, stress, and fear. C - The nurse should reinforce with the client's family that, in the last days of life, clients often develop anorexia or feel nauseated by food, and eating can cause pain and discomfort. In addition, the nutrients in food are not able to be absorbed as the client's body is shutting down. D - The nurse should encourage the client's family to continue to touch the client through the use of massage, holding the client's hand, or brushing the client's hair because this provides reassurance and comfort for both client and family.

A nurse is contributing to the plan of care for an older adult client who is at risk for osteoporosis. Which of the following interventions should the nurse include to prevent bone loss. A - Increase fluid intake. B - Encourage range-of-motion exercises. C - Massage bony prominences. D - Encourage weight-bearing exercises.

D - Encourage weight-bearing exercises.

A nurse is collecting data from a client who is at 33 weeks of gestation. Which of the following findings should the nurse identify as an indication of a potential complication of pregnancy? A - Leg cramps B - Tingling of fingers C - Varicose veins D - Epigastric pain

D - Epigastric pain

A nurse is collecting data from a client who is at 35 weeks of gestation. Which of the following findings should the nurse identify as a possible indication of preeclampsia? A - Vaginal bleeding B - Tingling of fingers C - Leg cramps D - Epigastric pain

D - Epigastric pain Rational A - The nurse should identify vaginal bleeding as a manifestation of placenta previa or abruptio placentae. The nurse should report vaginal bleeding to the provider. B - The nurse should identify tingling of the fingers as an expected manifestation during the second and third trimesters of pregnancy. This manifestation is caused by traction on the brachial plexus due to the drooping of the client's shoulders. C - The nurse should identify leg cramps as an expected manifestation during the third trimester of pregnancy. This manifestation is caused by compression of the pelvic nerves from the enlarged uterus. D - The nurse should identify epigastric pain as a manifestation of preeclampsia, and report this finding to the provider. Epigastric pain is caused by decreased perfusion to the liver, which leads to edema. Other manifestations of preeclampsia include hyperreflexia, headache, visual disturbances, and decreased urinary output.

A nurse is assisting with the care of a client who is at 40 weeks of gestation and is in active labor. Which of the following findings should the nurse report to the charge nurse? A - Maternal temperature of 37.5° C (99.5° F) B - Contractions every 3 min C - Presence of bloody show D - Prolonged deceleration of FHR

D - Prolonged deceleration of FHR

A nurse is collecting data from a newborn who is 8 hr old. WHich of the following findings should the nurse report to the provider? A - Caput succedaneum B - Mongolian spots C - Transient strabismus D - Generalized petechiae

D - Generalized petechiae Rational A - The nurse should expect a newborn to exhibit variations in head shape, including caput succedaneum, which is an edematous area over the scalp. It typically resolves independently in 3 to 4 days. B - The nurse should expect a newborn of Native American, African American, or Asian decent to have bluish-purple variations and discolorations, also known as Mongolian spots, over the buttocks or lower back. These areas fade gradually over months to years. However, careful and concise education and documentation of these areas by the nurse is extremely important because they are often mistaken for bruises once the newborn is discharged, which can lead to suspicion of child maltreatment. C - The nurse should expect a newborn to have crossing of the eyes, also known as transient strabismus. This should resolve independently within 3 to 4 months. D - The nurse should identify that generalized petechiae in a newborn indicates a severe infection or a low platelet count, indicating a clotting factor deficiency. The nurse should report this finding to the provider immediately.

A nurse is caring for a client who has difficulty swallowing. Which of the following actions should the nurse implement to prevent aspiration? A - Provide small, frequent meals. B - Tell the client to extend his neck when swallowing. C - Provide mouth care before meals. D - Give the client liquids with increased viscosity.

D - Give the client liquids with increased viscosity.

A nurse is caring for a client who has a prescription for glimepiride. Which of the following findings indicates a therapeutic response to the medication regimen? A - Fasting blood glucose 120 mg/dL B - Urine specific gravity 1.02 C - Hematocrit 44% D - Glycosylated hemoglobin (HbA1c) 4%

D - Glycosylated hemoglobin (HbA1c) 4% Rational A - This blood glucose level is outside the expected reference range of 74 to 106 mg/dL. Therefore, this finding is not an indication of a therapeutic response to glimepiride. B - Urine specific gravity measures the concentration of the particles found in the urine. A urine specific gravity of 1.02 is within the expected reference range of 1.005 to 1.030. However, this findings is not an indication of a therapeutic response to glimepiride. The nurse can test the urine for glucose to determine if the medication is effective. C - A hematocrit of 44% is within the expected reference range of 37% to 52%. However, this finding is not an indication of a therapeutic response to glimepiride. D - Glimepiride is an oral antidiabetic medication that stimulates the release of insulin from pancreatic islets. HbA1c measures the average blood glucose levels over the previous 120 days and is a good indicator of average client glucose levels. The expected reference range for HbA1c is a value of less than 7%. Therefore, an HbA1c of 4% is an indication of a therapeutic response to glimepiride.

A nurse is contributing to the plan of care for a client who has methicillin-resistant Staphylococcus aureus (MRSA) infection and is on contact isolation precautions. Which of the following actions should the nurse take? A - Keep the door of the client's room closed at all times. B - Remove gloves after leaving the client's room. C - Wear a mask when working within 1 m (3 feet) of the client. D - Have a designated stethoscope in the client's room.

D - Have a designated stethoscope in the client's room.

A nurse is assisting in the care of a client during the active phase of labor. Which of the following actions should the nurse take to promote the client's comfort? A - Prepare the client for a pudendal nerve block. B - Administer a sedative to the client. C - Encourage the client to push. D - Have the client perform relaxing breathing techniques.

D - Have the client perform relaxing breathing techniques.

A nurse is contributing to the plan of care for a client who has bipolar disorder and is exhibiting mania. Which of the following interventions should the nurse include to improve the nutrition status of the client? A - Weigh the client at the same time every other day. B - Encourage the client to attend communal meals. C - Request that the client keep a detailed food and beverage diary each day. D - Have the client's favorite snacks available at all times.

D - Have the client's favorite snacks available at all times.

A nurse is caring for a client 6 hr after a vaginal birth who is going to breastfeed her newborn. The client reports perineal pain of 6 on a scale for 0 to 10. The nurse also notes mild perineal edema and ecchymosis, with a fundus that is 2 cm above the umbilicus with deviation to the right. Which of the following actions is the nurse's priority? A- Administer analgesics. B - Apply an ice pack to the perineum. C - Assist the client with breastfeeding. D - Help the client ambulate to the toilet.

D - Help the client ambulate to the toilet. Rational A - The nurse should administer analgesics to relieve the client's pain; however, another action is the nurse's priority. B - The nurse should apply an ice pack to the client's perineum to reduce swelling and relieve the client's pain; however, another action is the nurse's priority. C - The nurse should assist the client with breastfeeding to promote uterine involution; however, another action is the nurse's priority. D - The greatest risk for this client is postpartum hemorrhage from uterine atony. Therefore, the priority intervention by the nurse is to assist the client to empty her bladder, which will allow the uterus to contract.

A nurse is reviewing the medical record of a client who takes warfarin and has a prescription for trimethoprim sulfamethoxazole. The nurse should report which of the following findings as an adverse effect of this combination of medication? A - Hypertension B - Hypernatremia C - Hyperglycemia D - Hemorrhage

D - Hemorrhage Rational A - Hypertension is not an adverse effect of the combination of warfarin with trimethoprim-sulfamethoxazole. The nurse should recognize hypotension as an adverse effect of trimethoprim-sulfamethoxazole. B - Hypernatremia is not an adverse effect of the combination of warfarin with trimethoprim-sulfamethoxazole. The nurse should recognize hyponatremia as an adverse effect of trimethoprim-sulfamethoxazole. C - Hyperglycemia is not an adverse effect of the combination of warfarin with trimethoprim-sulfamethoxazole. The nurse should recognize hypoglycemia as an adverse effect of trimethoprim-sulfamethoxazole. D - The nurse should recognize that trimethoprim-sulfamethoxazole, a sulfonamide combination, can increase the effects of warfarin, increasing the client's risk for bleeding. If the two medications must be given concurrently, the nurse should monitor the client's coagulation laboratory tests and INR carefully and report the results to the provider.

A nurse is reinforcing teaching with a client who is postoperative about the use of an incentive spirometer. Which of the following information should the nurse include? A - Take shallow breaths when using the spirometer. B - Cough prior to using the spirometer. C - Use the spirometer twice every 3 hr. D - Hold breath for 2 seconds, then exhale slowly.

D - Hold breath for 2 seconds, then exhale slowly. Rational A - The nurse should instruct the client to take slow, deep breaths when using an incentive spirometer. The client should hold their breath after inhalation to maintain lung expansion. B - The nurse should instruct the client to cough after taking deep breaths with an incentive spirometer. Deep breaths can loosen secretions in the lungs, and coughing helps to remove them. C - The nurse should instruct the client to use an incentive spirometer 4 to 10 times per hr. This increases lung expansion and prevents atelectasis. D - The nurse should instruct the client to inhale slowly and deeply, hold their breath for 2 to 3 seconds, and then exhale slowly. This allows for the alveoli of the lungs to expand, which reduces the risk of progressive collapse of the alveoli. The client should work up to holding their breath for 6 seconds following inhalation.

A nurse is developing countertransference toward a client during the working phase of the nurse-client relationship. To correct the situation, which of the following actions should the nurse take? A - Tell the client how to change her behaviors. B - Talk to the client about the developing feelings. C - Ask to be reassigned to a different client. D - Identify personal response to the client.

D - Identify personal response to the client.

A nurse is contributing to the plan of care for an adolescent client who has human immunodeficiency virus (HIV). Bases on the adolescent's diagnosis, which of the following actions should be included in the plan of care? A - Instruct visitors to wear gowns and masks when entering the client's room. B - Contact the dietary department to request that foods be delivered on disposable dishes. C -Prepare a negative-pressure airflow room for the client. D - Inform the client regarding routes of transmission.

D - Inform the client regarding routes of transmission.

A nurse is reinforcing teaching with the parent of a child who has ADHD and is exhibiting disruptive behaviors at home. Which of the following actions should the nurse instruct the patient to take? A - Avoid enforcing a strict schedule for the child at home. B - Use biofeedback with the child. C - Give the child a PRN dose of methylphenidate. D - Initiate a point system for the child.

D - Initiate a point system for the child.

A nurse on a postpartum unit is assisting in the care of a client who is experiencing hypovolemic shock. Which of the following actions should the nurse take? A - Place the client in high-Fowler's position. B- Administer terbutaline subcutaneously. C - Apply oxygen at 2 L/min via nasal cannula. D - Insert an indwelling urinary catheter.

D - Insert an indwelling urinary catheter.

A nurse is preparing to instill a vaginal medication in suppository form to a client. Which of the following actions should the nurse take? A - Wear sterile gloves. B - Use the dominant hand to retract the labia. C - Guide the suppository along the anterior vaginal wall. D - Insert the suppository using the entire length of the finger.

D - Insert the suppository using the entire length of the finger. Rational A - The nurse should wear clean gloves for this procedure. If the client has a latex allergy, the nurse should use latex-free gloves B - The nurse should use the nondominant hand to expose the vaginal orifice. The dominant hand can then be used to insert the suppository. C - The nurse should guide the suppository along the posterior vaginal wall. This action helps to ensure distribution of the medication along the vaginal wall. D - To ensure adequate distribution of the vaginal medication, the nurse should insert the suppository until the entire length of the index finger is inside the vagina, or as far in as it will go, to an approximate length of 7.6 to 10.2 cm (3 to 4 in).

A nurse is contributing to the plan of care for an older adult client who is experiencing weight loss. Which of the following interventions should the nurse include? A - Select foods with less complex carbohydrates. B - Discourage visitors at mealtime. C - Provide the largest meal in the evening. D - Instruct the client to eat nutrient-dense foods first.

D - Instruct the client to eat nutrient-dense foods first. Rational A - The nurse should assist the client to select foods with more complex carbohydrates, such as legumes, potatoes, cereals, rice, and breads. The nurse should also assist the client to select less sugar-rich foods that are more appealing but can curb the client's appetite. B - The nurse should plan to encourage the client to have visitors while eating, establishing mealtime as a positive social event to interact and converse with others. This is conducive to maintaining the mealtime for a longer period of time, which helps to facilitate the client to eat more during the meal. C - The nurse should plan to have the client eat the largest meal at noon to aid in the digestive process and promote more restful sleep at night. D - The nurse should plan to have the client eat nutrient-dense foods first in order to consume the greatest amount of vitamins and calories.

A nurse is reinforcing teaching with the parent of a child who has hemophilia and is experiencing acute hemarthrosis. Which of the following instructions should the nurse include in the teaching? A - Provide the child with a high-calorie diet. B - Apply warm compresses to affected joints. C - Administer ibuprofen for reports of pain. D - Keep the affected joints immobilized.

D - Keep the affected joints immobilized.

A nurse is collecting data form a toddler at a well-child visit. Which of the following findings should the nurse identify as a possible indication of. child maltreatment? A - Diaper dermatitis B - Bruise on the front of the lower leg C - Inflamed unilateral conjunctiva D - Laceration on the side of the torso

D - Laceration on the side of the torso

A nurse is monitoring a client who recently had a cast placed on the right lower extremity for a bone fracture. Which of the following findings should the nurse recognize as abnormal? A - Report of a dull, throbbing pain B - Extremities that are cool bilaterally C - Capillary refill of 3 seconds in the nail beds of the toes D - Lack of sensation between the first and second toes

D - Lack of sensation between the first and second toes

A nurse is attempting to establish a therapeutic relationship with a newly admitted client. Which of the following actions should the nurse plan to take during the working phase of the nurse-client relationship? A - Summarize the goals achieved in the relationship. B - Plan for the initial interaction with the client. C - Establish the parameters of the relationship. D - Promote the development of problem-solving skills.

D - Promote the development of problem-solving skills.

A nurse is preparing to administer ceftriaxone IM to a client. Which of the following actions should the nurse take? A - Use a 5/8-inch needle for the injection. B - Inject the medication at a 45° angle. C - Administer the medication in the deltoid muscle. D - Locate the ventrogluteal muscle for the injection site.

D - Locate the ventrogluteal muscle for the injection site. Rational A - An IM injection of this medication is painful. The nurse should use a 1- or 1.5-inch needle to administer the medication into a deep, large muscle. B - The nurse should hold the syringe at a 90° angle to administer an IM injection. The nurse should use a 45° or 90° angle when administering a subcutaneous injection. C - The nurse should administer this medication into a deep, large muscle. The deltoid muscle is too small and is not an appropriate site. D - The nurse should administer this medication into a deep, large muscle, such as the ventrogluteal site. This is the safest injection site for an intramuscular injection.

A nurse observes a client who is lying in bed experiencing a tonic-clonic seizure. Which of the following actions should the nurse take? A - Lower the side rails of the client's bed. B - Apply wrist restraints to the client. C - Position the client in the semi-Fowler's position. D - Loosen clothing around the client's neck.

D - Loosen clothing around the client's neck.

A nurse is collecting data from a client whose laboratory report indicated a potassium level of 3.1 mEq/L. Which of the following findings should the nurse expect? A - Increased bowel sounds B - Hyperactive reflexes C - Bounding peripheral pulses D - Muscle weakness

D - Muscle weakness Rational A - The nurse should expect the client who has hypokalemia to have decreased bowel sounds and constipation. After reporting findings to the provider, the nurse should monitor the client and administer oral potassium if prescribed. B - The nurse should expect the client who has hypokalemia to manifest hypoactive deep-tendon reflexes. After reporting findings to the provider, the nurse should monitor the client and administer oral potassium if prescribed. C - The nurse should expect the client who has hypokalemia to have irregular and weak pulses. After reporting findings to the provider, the nurse should monitor the client and administer oral potassium if prescribed. D - A potassium level of 3.1 mEq/L is below the expected reference range of 3.5 to 5.0 mEq/L, indicating hypokalemia. The nurse should expect this client to manifest ascending muscle weakness that starts in the quadriceps and can eventually impair respirations. Additional expected findings include decreased bowel sounds, constipation, hypoactive reflexes, weak and irregular pulses, and cardiac dysrhythmias. After reporting findings to the provider, the nurse should monitor the client and administer oral potassium if prescribed.

A nurse is preparing to care for a group of clients following charge-of-shift report. Which of the following actions should the nurse plan to take first? A - Document the mood of a client who has seasonal affective disorder and began receiving light therapy 2 weeks ago. B - Assist a client who has major depressive disorder to identify coping skills. C - Reinforce teaching about adverse effects of clozapine with a client who has schizophrenia. D - Offer a high-protein beverage to a client who has bipolar disorder and is experiencing a manic episode.

D - Offer a high-protein beverage to a client who has bipolar disorder and is experiencing a manic episode. Rational A - The nurse should document the mood of a client who is receiving light therapy for seasonal affective disorder to determine the effectiveness of the treatment. However, there is another action the nurse should take first. B - The nurse should assist a client who has major depressive disorder to identify coping skills to help with prevention of self-destructive behaviors. However, there is another action the nurse should take first. C - The nurse should reinforce teaching about adverse effects of clozapine with a client who has schizophrenia to promote adherence to the treatment plan. However, there is another action the nurse should take first. D - When using Maslow's hierarchy of needs to provide client care, the nurse should identify that the first action to take is to address the client's physiological needs. A client who has bipolar disorder and is experiencing a manic episode is at risk for malnutrition due to an inability to remain still to eat. By offering high-calorie, high-protein beverages and nutritious finger foods, the client can eat while pacing, which helps to maintain calorie requirements.

A nurse is reinforcing teaching about breast self-examination (BSE) with a group of clients who are premenopausal. At which of the following times should the nurse recommend the clients perform BSE? A - At the onset of ovulation B - The last day of each month C - The first day of the menstrual cycle D - One week after the menstrual cycle begins

D - One week after the menstrual cycle begins Rational A - Before menses, hormone changes increase the texture and sensation of breast tissue. Breast engorgement begins before ovulation, does not subside until several days after menses ends, and can interfere with accurate palpation. If conception does not occur, the accumulated fluid drains away via the lymphatic network. B - This action could cause inaccurate assessment because examination could occur at different times of the menstrual cycle each month. The nurse should recommend performing the exam on the first day of the month for postmenopausal women. After menopause, adipose breast tissue atrophies and is replaced by connective tissue. Elasticity is lost, and the breast can droop and become pendulous. C - The average menstrual cycle is 28 days and is controlled by the endocrine system. In the luteal phase, just prior to menstruation, increased estrogen and progesterone cause fluid retention and discomfort, and stimulate the growth of milk ducts in the breasts. Breast engorgement and tenderness interferes with accurate palpation and examination of breast tissue. D - Premenopausal clients can perform BSE once per month, 4 to 7 days after the beginning of the menstrual cycle. At this time, hormonal influence on breast tissue is decreased, so fluid retention and tenderness are reduced. Clients should inspect the breast for symmetry, size, contour, skin changes (color, texture, and venous patterns), nipple changes, and lesions. One breast might be larger than the other, and inverted nipples are common. The client should ask their provider about whether they recommend a BSE.

A nurse is contributing to the development of a fall prevention policy for clients who have dementia. Which of the following sources of information should the nurse identify as the primary guideline for the creation of the policy? A - Clinical expertise of facility nurses B - Review of medical records C - Facility performance indicators D - Peer-reviewed nursing journals

D - Peer-reviewed nursing journals

A nurse is collecting data from a client who is experiencing opioid withdrawal. Which of the following manifestations should the nurse expect? A - Hypotension B - Bradycardia C - Hypothermia D - Pupillary dilation

D - Pupillary dilation Rational A -Hypertension is an expected manifestation for a client who is experiencing opioid withdrawal. B - Bradycardia is an expected manifestation for a client who is experiencing opioid intoxication. C - Hypothermia is an expected manifestation for a client who is experiencing opioid intoxication. D -The nurse should expect a client who is experiencing opioid withdrawal to have dilated pupils. Constriction of pupils is caused by anoxia. When clients begin the withdrawal process, the pupils relax, causing dilation to occur. This manifestation will gradually diminish over 5 to 10 days.

A nurse is reinforcing teaching with an adult client who has stress incontinence. Which of the following is an evidence-based intervention the nurse should include? A - Wearing a bladder incontinence pad B - Using intermittent bladder catheterization C - Limiting fluid intake during waking hours D - Performing exercises that strengthen pelvic floor muscles

D - Performing exercises that strengthen pelvic floor muscles Rational A - This intervention can help prevent urine leaking through clothing. However, it is not an evidence-based guideline for reducing stress incontinence. B - Intermittent bladder catheterization is not recommended for stress incontinence. The procedure puts the client at risk for infection and is unlikely to resolve the problem of stress incontinence. Bladder catheterization might be required for a client who has urinary retention. C - The nurse should encourage the client to take most fluids during the waking hours (between 0600 and 1800) to prevent nocturia and to schedule fluid intake around social occasions when possible. D - Evidence-based practice is the integration of current research with clinical skills to improve clients' outcomes. Stress incontinence can be reduced by using exercises that strengthen the pelvic floor muscles, such as Kegel exercises.

A nurse is caring for a client who delivered vaginally 6 hr ago. Which of the following findings should the nurse report to the provider? A - Labial edema B - Fundus firm at the umbilicus C - WBC count 15,000/mm3 D - Perineal pad soaked in 15 min

D - Perineal pad soaked in 15 min

A nurse is collecting data from a newly admitted client who has anorexia nervosa. Which of the following manifestations should the nurse expect? A - Tachycardia B - BMI of 22 C - Hypertension D - Peripheral edema

D - Peripheral edema

A nurse is preparing to remove a client's NG tube. Which of the following interventions should the nurse take to decrease the risk of aspiration? A - Instill 10 mL of air through the NG tube. B - Place the client in the supine position. C - Irrigate the NG tube. D - Pinch the NG tube.

D - Pinch the NG tube. Rational A - The nurse should instill 50 mL of air through the NG tube to remove mucus and gastric secretions from the tube and to prevent aspiration of these secretions. B - The nurse should place the client in a sitting position to prevent the risk of aspiration. C - The nurse should identify that irrigating the NG tube before removal can put the client at risk for aspiration and should be avoided. D - The nurse should pinch the NG tube to prevent secretions from draining into the client's throat, which can cause aspiration.

A nurse is caring for a newborn who is receiving phototherapy. Which of the following actions should the nurse take? A - Reposition the newborn every 4 hr. B - Feed the newborn 30 mL (1 oz) of glucose water four times per day. C - Apply a thin layer of lotion to the newborn's skin. D - Place an opaque mask over the newborn's eyes.

D - Place an opaque mask over the newborn's eyes.

A nurse is caring for a client who has paraplegia following a stroke. When of the following actions should the nurse take to prevent deep vein thrombosis (DVT)? A - Massage the client's lower extremities daily. B - Apply cold packs to the client's calves three times a day. C - Elevate the head of the client's bed. D - Place sequential compression devices on the client's lower legs.

D - Place sequential compression devices on the client's lower legs. Rational A - A client who has limited mobility is at an increased risk for the development of DVT. The nurse should not massage the lower extremities as this can dislodge a thrombus, leading to an embolism. B - A client who has limited mobility is at an increased risk for the development of DVT. Application of cold packs causes vasoconstriction, which increases the risk for venous stasis. If the client develops a thrombus, the nurse should apply warm, moist heat to the area. C - A client who has limited mobility is at an increased risk for the development of DVT. Following a stroke, the nurse should elevate the head of the client's bed to a 30° angle to reduce intracranial pressure and to promote venous drainage. Elevating the client's legs promotes venous return and helps prevent the development of DVT. D - Prevention of DVT includes client education, leg exercises, early ambulation, adequate hydration, graduated compression stockings, and intermittent pneumatic compression, such as sequential compression devices (SCDs) and venous plexus foot pumps. The use of SCDs helps prevent venous stasis by promoting circulation.

A nurse is reviewing the laboratory report of a preschooler who has a Wilms' tumor and is scheduled to begin treatment with an antineoplastic medication regimen. Which of the following laboratory results should the nurse report to the provider? A - BUN 16 mg/dL B - WBC 5,500/mm3 C - Serum glucose 98 mg/dL D - Platelet count 70,000/mm3

D - Platelet count 70,000/mm3

A nurse is reviewing the laboratory results for a client who is receiving heparin. Which of the following laboratory values should the nurse report to the provider? A - Hematocrit 45% B - aPTT 60 seconds C - INR 1.0 D - Platelets 74,000/mm3

D - Platelets 74,000/mm3 Rational A - This hematocrit (Hct) level is within the expected reference range of 42% to 52% for male clients and 37% to 47% for female clients. An Hct level below the expected reference range in a client who is receiving heparin therapy can indicate bleeding. B - This aPTT level is within the expected reference range of 1.5 to 2.5 times the control value of 30 to 40 seconds for a client who is receiving heparin therapy. The nurse should identify that if the aPTT is not within the therapeutic anticoagulant range of 50 to 100 seconds, the client's dose of heparin should be adjusted. C - This INR level is within the expected reference range of 0.8 to 1.1. Clients who are taking warfarin should have INR testing to measure anticoagulant effects. D - The nurse should identify that this platelet count is below the expected reference range of 150,000 to 400,000/mm3, which could indicate heparin-induced thrombocytopenia and should be reported to the provider. This disorder is an adverse effect of heparin that causes the activation of platelets, resulting in widespread clot formation and platelet depletion.

A nurse is collecting data from a client who has paranoid personality disorder. Which of the following manifestations should the nurse expect? A -Preoccupied with details B - Uses attention-seeking behaviors C - Exploitive of others D - Projects blame onto others

D - Projects blame onto others Rational A - The nurse should expect a client who has obsessive-compulsive behavior to have manifestations of being preoccupied with details. B - The nurse should expect a client who has histrionic personality disorder to have manifestations of attention-seeking behaviors. C - The nurse should expect a client who has antisocial behavior to have manifestations of exploiting others. D - The nurse should expect clients who have paranoid personality disorder to project blame onto others rather than taking responsibility for their own actions.

A nurse in an urgent care facility is caring for a client who is receiving care following a sexual assault. Which of the following actions should the nurse plan to take? A - Allow the client to shower upon arrival to facility. B - Discourage the client from making negative statements. C - Recommend the client take legal action. D - Reassure the client that they are in a safe place.

D - Reassure the client that they are in a safe place. Rational A - Allowing the client to shower prior to the forensic examination can decrease the ability to collect and preserve evidence. A shower and change of clothes should be offered to the client after the forensic examination is completed. B - The nurse should encourage the client to voice emotions using reflection and other therapeutic communication techniques. C - The nurse should not offer personal advice to clients because this decreases the client's sense of autonomy and is nontherapeutic. D - The nurse should reassure the client that they are in a safe place because this can help the client feel more secure. Clients who have experienced sexual assault are often frightened or in shock.

A nurse is caring for a client who had a lumbar laminectomy and has a prescription for an oral opioid analgesic. Which of the following actions should the nurse take? A - Ask the client after 15 min if the pain medication is effective. B - Administer the medication after the client ambulates. C -Instruct the client to increase oral fluids to 1,500 mL daily. D - Recommend the client increase their intake of fiber-rich foods.

D - Recommend the client increase their intake of fiber-rich foods. Rational A - The nurse should ask the client if the oral pain medication is effective after 30 to 45 min and every 1 to 2 hr thereafter because oral medications have a slower onset than other routes of medications, such as intravenous. The nurse should also use nonpharmacological methods to manage the client's discomfort, such as cold or heat therapy. Relaxation, guided imagery, or music can also be used to manage the client's discomfort. B - The nurse should administer the medication 30 to 45 min before the client ambulates or before painful procedures or dressing changes. This will ensure the client will have less pain upon ambulation and when performing other activities. C - The nurse should encourage the client to increase intake of fluids, when possible, to 2,000 mL daily. Increased fluid intake keeps the feces moist and soft. Constipation is an adverse effect of opioid analgesia because the medication slows peristalsis. The feces dry out and become compacted from the prolonged time in the colon. The nurse should also encourage activity within the client's limitations to stimulate peristalsis. D - The nurse should encourage the client to increase intake of fiber-rich foods to improve the consistency of the feces and to ease bowel movements. Constipation is an adverse effect of opioid analgesia because the medication slows peristalsis. The feces dry out and become compacted from the prolonged time in the colon. The nurse should administer stool-softeners or laxatives if needed to alleviate this adverse effect.

A nurse is reinforcing discharge teaching about wound care with a family member of a client who is postoperative. Which of the following should the nurse include in the teaching? A - Administer an analgesic following wound care. B - Irrigate the wound with povidone iodine. C - Cleanse the wound with a cotton-tipped applicator. D - Report purulent drainage to the provider.

D - Report purulent drainage to the provider.

A nurse in a long-term care facility is collecting data from a client who reports fullness in the rectum and abdominal cramping. Which of the following findings should indicate to the nurse that the client might have a fecal impaction? A - Halitosis B - Hemorrhoids C - Rebound tenderness D - Small liquid stools

D - Small liquid stools

A nurse is reviewing prescriptions for a client who has a serum potassium level of 5.3 mEq/L. The nurse should identify that which of the following medications can cause hyperkalemia? A - Simvastatin B - Diltiazem C - Carvedilol D - Spironolactone

D - Spironolactone Rational A - Simvastatin is an HMG-CoA reductase inhibitor used in the treatment of hypercholesterolemia. Hyperkalemia is not an adverse effect of simvastatin. B - Diltiazem is a calcium channel blocker used in the treatment of angina, hypertension, and cardiac arrhythmias. Hyperkalemia is not an adverse effect of diltiazem. C - Carvedilol is a beta blocker used in the treatment of hypertension. Hyperkalemia is not an adverse effect of carvedilol. D - Spironolactone is a potassium-sparing diuretic used in the treatment of edema and hypertension. Hyperkalemia is a potential adverse effect of spironolactone. The nurse should withhold the medication and contact the provider.

A nurse arrives for her shift and is assigned more clients than she feels is safe. The charge nurse states there are no other options due to a shortage in nursing staff. Which of the following actions should the nurse take? A - Request to float to another unit. B - Refuse the assignment and leave the unit. C - File an incident report with the risk manager. D - Submit a written complaint to the nursing supervisor.

D - Submit a written complaint to the nursing supervisor.

A nurse in a clinic is collecting data from an adolescent who has received all recommended immunizations through the age of 6 years. Which of the following immunizations should the nurse plan to administer? A - Haemophilus influenza type b (Hib) B - Rotavirus (RV) C - Polio (IPV) D - Tetanus, diphtheria toxoids, and acellular pertussis (Tdap)

D - Tetanus, diphtheria toxoids, and acellular pertussis (Tdap) Rational A - The Hib immunization series is administered by 18 months of age. B - The RV immunization series is administered by 6 months of age. C - The IPV immunization series is administered by 6 years of age. D - The Tdap vaccine is recommended between the ages of 11 and 12 years. Therefore, this adolescent should receive the Tdap vaccine now.

A nurse is collecting data from a client who has post traumatic stress disorder (PTSD) due to a childhood maltreatment. Which of the following manifestations should the nurse expect? A - The client exhibits grandiose behaviors. B - The client shows an obsessive interest in performing daily activities. C - The client talks continually about the traumatic event. D - The client demonstrates hypervigilant behavior.

D - The client demonstrates hypervigilant behavior. Rational A - The nurse should expect a client who is experiencing mania to exhibit grandiosity, which is an exaggerated positive opinion of oneself. The nurse should expect a client who has PTSD to have negative thoughts about themselves and report feelings of guilt and shame. The nurse should encourage the client to identify their own strengths. B - The nurse should expect a client who has obsessive-compulsive disorder to show an obsessive interest in performing daily activities. The nurse should expect a client who has PTSD to be apathetic and show a decreased interest in daily activities. Clients who have PTSD often have difficulty developing meaningful relationships with others. The nurse should spend time with the client to build a trusting nurse-client relationship. C - The nurse should expect a client who has PTSD to avoid talking about the traumatic event and their emotions associated with it. The nurse should encourage the client to share their feelings about the event. D - The nurse should expect a client who has PTSD to be hypervigilant and easily startled. Other common manifestations of PTSD include irritability, anger, depression, and feelings of detachment. These feelings can result in thoughts of self-harm; therefore, the nurse should collect data regarding the client's thoughts and recommend interventions to ensure client safety.

A nurse is using a critical pathway while providing care to a client who is 3 days postoperative. Which of the following events should the nurse document as a variance? A - The nurse on the prior shift administered the client's twice-daily antibiotic 1 hr after it was due. B - The nurse hears two assistive personnel discussing the client in a public elevator. C - The client reports she does not have an advance directive. D - The client has a circular area of nonblanchable redness on her left heel.

D - The client has a circular area of nonblanchable redness on her left heel. Rational A variance occurs when expected outcomes of the critical pathway are not met. The nurse should document that the client has a circular area of nonblanchable redness on her left heel as a variance because this indicates the initial stage of a pressure ulcer and is not an expected outcome.

A nurse is participating in the unit's performance improvement program. The nurse should recognize that which of the following is a quality indicator? A - The unit sets a goal to ask all clients upon admission if they have a living will. B - An additional 15 wall-mounted hand sanitizers are installed on the unit. C - A new standardized form is developed for peer reviews facility-wide. D - The facility-wide fall injury rate for the previous quarter is 3%.

D - The facility-wide fall injury rate for the previous quarter is 3%.

A charge nurse is observing a newly licensed nurse preparing to administer an intermittent tube feeding via an NG tube to a client. For which of the following actions should the charge nurse intervene? A - The nurse initiates the feeding after obtaining 50 mL residual. B - The nurse flushes the tube with 30 mL of air before aspirating gastric fluid. C - The nurse administers the feeding through a syringe barrel by gravity. D - The nurse allows the client to sleep in supine position during the feeding.

D - The nurse allows the client to sleep in supine position during the feeding. Rational A - Prior to administering an intermittent gastric tube feeding, the nurse should check the stomach for residual volume. The nurse should check the facility policy if a single residual check is more than 100 mL or more than half the last feeding. If a single check is more than 500 mL, or two consecutive checks are 250 mL or more, the nurse should withhold the feeding. B - The nurse should flush the tube with 30 mL of air before aspirating gastric fluid to open and clear the feeding tube. After medication administration, the nurse should flush the NG tube with 30 to 100 mL of water to prevent clogging. The nurse should remember to note the amounts of water on the I&O record. C - The nurse can administer intermittent feedings by using a large barrel syringe or feeding bag. The nurse should fill the syringe with formula and hold it high enough for the formula to empty gradually via gravity. The nurse should continue to refill the syringe until the prescribed amount is administered. Following the feeding, the nurse should flush the tube with 30 mL water. D - The nurse should keep the head of the bed positioned at a minimum of 30° (preferably 45°) to prevent aspiration from reflux during feedings and for at least 30 min afterward. The nurse can place the client in reverse Trendelenburg if unable to elevate the head of the client's bed.

A nurse is reinforcing teaching with a client who is starting to take levothyroxine for hypothyroidism. The nurse should instruct the client to monitor and report which of the following as an indication of an overdose of this medication? A - Lethargy B - Weight gain C - Constipation D - Tremors

D - Tremors Rational A - The nurse should instruct the client to monitor for insomnia, which is a manifestation of levothyroxine overdose or thyrotoxicosis. Lethargy is a manifestation of hypothyroidism. The nurse should instruct the client to balance activity and rest to promote comfort. B - The nurse should instruct the client to monitor and report weight loss, an adverse effect of levothyroxine, to the provider. Weight gain is a manifestation of hypothyroidism. C - The nurse should instruct the client to monitor and report diarrhea and abdominal cramps, which are adverse effects of levothyroxine. Constipation is a manifestation of hypothyroidism. D - The nurse should instruct the client to monitor and report manifestations of levothyroxine overdose or thyrotoxicosis, such as tachycardia, angina, tremors, insomnia, and sweating. The nurse should instruct the client to have thyroid function tests performed yearly to monitor thyroid levels.

A nurse is participating on a committee that is revising the facility's policies and procedures for infection control. Which of the following statements should the nurse recommend to include in the facility's infection control manual? A - Double-bag linens prior to removing them from the client's room. B - Place sterile objects within 1.3 cm (0.5 in) inside the edge of a sterile field. C - Apply a surgical mask to a client on contact isolation prior to transporting to radiology for an x-ray. D - Use a 1:10 bleach solution to clean blood spills.

D - Use a 1:10 bleach solution to clean blood spills.

A nurse is reviewing notes from change-of-shift report regarding the need to remove a client's NG tube. Which of the following actions should the nurse take first? A - Disconnect the tube from the wall suction. B - Perform hand hygiene before donning gloves. C - Provide mouth care for the client. D - Verify the provider's prescription to discontinue the tube.

D - Verify the provider's prescription to discontinue the tube. Rational A - One purpose of NG tubes is to provide gastric decompression by using suction connected through a drainage canister. Disconnecting the tube from the wall suction prior to removal will prevent damage to the gastric mucosa during tube removal; however, another action is the priority. B - Performing hand hygiene and donning gloves is important to decrease the risk of transmitting micro-organisms to and from the client; however, another action is the priority. C - Providing mouth care for the client maintains and promotes hygiene and comfort following the removal of the NG tube; however, another action is the priority. D - The greatest risk to this client is injury from early removal of the NG tube. Early NG tube removal can cause altered nutrition, fluid and electrolyte imbalance, and possible abdominal discomfort or aspiration if the tube is used for decompression. Reinserting the tube due to an error might cause the client discomfort. Therefore, the priority action the nurse should take is to verify the provider's order to discontinue the tube.

A nurse is checking the meal tray of a school-age child who has celiac disease. Which of the following foods should the nurse remove? A - Creamed corn B - Steamed white rice C - Baked potato D - Wheat bread

D - Wheat bread Rational A - The nurse should limit the child's intake of foods that contain significant amounts of gluten. Corn is an acceptable substitute grain for a child who has celiac disease. B - The nurse should limit the child's intake of foods that contain significant amounts of gluten. Steamed white rice is an acceptable substitute grain food for a child who has celiac disease. C - The nurse should limit the child's intake of foods that contain significant amounts of gluten. A baked potato is an acceptable food for a child who has celiac disease. D - The nurse should identify that wheat bread contains gluten, which should be eliminated from the child's diet. Therefore, the nurse should remove it and contact the facility's dietary department to request a substitute for the child to maintain caloric intake. Foods containing rye and barley should also be eliminated from the child's diet. Consuming these foods can exacerbate manifestations of celiac disease, including diarrhea, steatorrhea, and abdominal distention and pain.

A nurse is reinforcing teaching with the guardian of child who has scabies and a new prescription for permethrin 5% cream. Which of the following information should the nurse include? A - "Apply the cream to only the affected areas twice daily." B - "Wash off the cream 4 to 6 hours after application." C - "The provider will monitor for liver toxicity while your child is taking the medication." D -"The medication will eliminate your child's itching within 2 to 3 weeks."

D -"The medication will eliminate your child's itching within 2 to 3 weeks."

A nurse overhears two assistive personnel (AP) discussing the details of a client's diagnosis and treatment plan in the hospital cafeteria. Which of the following actions should the nurse take? A - Report the situation directly to the facility's risk manager. B - Determine if both of the APs are directly involved in the client's care. C - Place a completed incident report in the client's medical record. D -Inform the APs that the conversation violates the client's confidentiality.

D -Inform the APs that the conversation violates the client's confidentiality.

A nurse is assisting with the care of a 3-year-old child who is prescribed a lumbar puncture. Which of the following actions should the nurse take to prevent complications? A - Stabilize the child in a prone position during the procedure. B - Apply temporary elbow restraints during the procedure. C - Implement NPO status for 12 hr after the procedure. D -Maintain the child in a flat position after the procedure.

D -Maintain the child in a flat position after the procedure.

A charge nurse in a long-term care facility is monitoring the activities of an assistive personnel (AP). Which of the following actions by the AP indicates that the charge nurse should intervene? A - Returns unopened supplies from a client's room to the storage room B - Obtains assistance when lifting an object that weighs 18.1 kg (40 lb) C -Double-bags a biohazard bag that is contaminated on the outside D -Stands with feet close together while transferring a client from the bed to a chair

D -Stands with feet close together while transferring a client from the bed to a chair

A nurse is collecting data from a client who is 32 hr postpartum. Which of the following findings should the nurse expect? A - Saturation of one perineal pad every 15 min B - Fundus 2 cm above the umbilicus C - Temperature of 39° C (102.2° F) D -Urine output of 3,000 mL in 24 hr

D -Urine output of 3,000 mL in 24 hr

A nurse is collecting data from a client who has heart failure and is taking digoxin. Which of the following outcomes from the medication should the nurse expect? A - Increased weight B - Increased heart rate C - Decreased urinary output D- Decreased shortness of breath

D- Decreased shortness of breath

A nurse is caring for a client who has been receiving potassium chloride after being admitted with a serum potassium of 3.1 mEq/L. Which of the following findings should the nurse identify as a therapeutic responses to treatment? A - Decreased bowel sounds B - Positive Trousseau's sign C - Positive Chvostek's sign D- Increased muscle strength

D- Increased muscle strength Rational A - Decreased bowel sounds are a manifestation of hypokalemia. The client might also have a paralytic ileus with a low potassium level. Therefore, this is not a therapeutic response to potassium supplements. B - A positive Trousseau's sign is a manifestation of hypocalcemia or hypomagnesemia, rather than a therapeutic response for a client who is receiving potassium supplements. A positive Trousseau's sign is manifested by palmar flexion and spasm when a blood pressure cuff is inflated above the client's systolic blood pressure for 3 min. C - A positive Chvostek's sign is a manifestation of hypocalcemia or hypomagnesemia, rather than a therapeutic response for a client who is receiving potassium supplements. A positive Chvostek's sign is manifested when the facial nerve is tapped anterior to the earlobe and the face begins to twitch. D - Hypokalemia causes muscle weakness. A therapeutic response to potassium supplements would be an increase in muscle strength.

A nurse is reinforcing teaching with a client who is on a low-sodium diet and asks how to improve the taste of bland food. Which of the following should the nurse recommend? A - Ketchup B - Mayonnaise C - Soy sauce D- Lemon juice

D- Lemon juice

A nurse is assisting with the care of a client who is receiving 0.9% sodium chloride by continuous IV infusion. The client reports pain and swelling at the IV site. In which order should the nurse perform the following steps? A - Notify the charge nurse B - Stop the infusion C - Elevate the affected arm D - Withdraw the IV catheter E - Check the IV site

E - Check the IV site B - Stop the infusion D - Withdraw the IV catheter C - Elevate the affected arm A - Notify the charge nurse

A nurse is providing postmortem care to an adult client. Which of the following actions should the nurse take? Select all A - Place the client in a side-lying position. ​B - Determine whether an autopsy has been ordered. ​C - Cover the client's body with a sheet. D - Ask the client's loved ones about their religious rituals. E - Remove the client's dentures.

​B - Determine whether an autopsy has been ordered. ​C - Cover the client's body with a sheet. D - Ask the client's loved ones about their religious rituals. Rational A - Place the client in a side-lying position is incorrect. The nurse should place the client in a supine position with their head on a pillow. B - Determine whether an autopsy has been ordered is correct. The nurse should determine whether an autopsy has been ordered prior to performing any care to the body.​ C - Cover the client's body with a sheet is correct. The nurse should cover the client's body with a sheet and provide privacy for the family to view the body. D - Ask the client's loved ones about their religious rituals is correct. The nurse should ask the client's family or loved ones about any religious or cultural practices that might need to be included in the postmortem care of the client's body. E - Remove the client's dentures is incorrect. The nurse should ensure that the client's dentures remain in place to maintain the client's normal facial appearance.

A nurse is reinforcing discharge instructions with a client who has a permanent tracheostomy. Which of the following equipment should the nurse verify is available at the client's home? select all A - ​Stethoscope ​B - Oxygen ​C - Suction machine D - ​Portable ventilator ​E - Replacement cannula

​B - Oxygen ​C - Suction machine ​E - Replacement cannula Rational A - Stethoscope is incorrect. The nurse should reinforce teaching about indications of airway obstruction to report to the provider immediately. However, the client is not required to have a stethoscope in the home. B - Oxygen is correct. The nurse should reinforce that the client needs portable oxygen in the home care setting, even if continuous oxygen therapy is not required. C - Suction machine is correct. The nurse should reinforce the availability of suction equipment in the home so the client is able to clear airway secretions effectively. D - Portable ventilator is incorrect. There is no indication of a need for a portable ventilator for this client. E - Replacement cannula is correct. The nurse should reinforce teaching about having a replacement cannula available in the home in case dislodgment of the tracheostomy cannula occurs.

A nurse is collecting data from a client who is 2 weeks postpartum and breastfeeding their newborn. Which of the following findings indicates mastitis? A - WBC count 9,500/mm3 ​B - Firm breasts and areolae ​C - Red and painful area in one breast D - Hgb 12.0 g/dL

​C - Red and painful area in one breast Rational A - A WBC count of 9,500/mm3 is within the expected reference range of 5,000 to 10,000/mm3. A client who has mastitis can have an elevated WBC count. B - Firm breasts and areolae indicate engorgement. Engorgement usually occurs 3 to 5 days after delivery and lasts about 24 hr. It is a temporary condition and usually resolves with comfort measures. C - ​Mastitis often appears as a red, hard, and painful area in one or both breasts. Although mastitis can occur in both breasts, it is usually unilateral. It usually occurs within the first 6 weeks and might require antibiotics to resolve. D - An Hgb of 12 g/dL is within the expected reference range of 12 to 16 g/dL.


Conjuntos de estudio relacionados

Similarity, Similar Figures, Similar Figures

View Set

CH 7 - Escrows and Title Insurance

View Set

Sadlier-Oxford Vocab Level H - Unit 1

View Set

Security + 501 Chapter 5 Risk Management

View Set

Ch 39 Nursing assesement: GI practice questions

View Set

Amboss Rheumatology and Immunology

View Set

Chapter 30 - Listening Guide Quiz 20: Haydn: Symphony No. 94 in G Major (Surprise), II

View Set